CBSE previous Year Solved Papers Class 12 English Delhi 2011

CBSE previous Year Solved  Papers  Class 12 English  Delhi 2011

Time allowed : 3 hours                                                                                           Maximum Marks: 100
General Instructions :

  1. This paper is divided into three sections : A, B and C. All the sections are compulsory.
  2. Separate instructions are given with each section and question, wherever necessary. Read these instructions very carefully and follow them faithfully.
  3. Do not exceed the prescribed word limit while answering the questions.

SET I

SECTION – A
(Reading)

Question.1. Read the passage given below and answer the questions
that follow:

  1.  For many years now the governments has been promising the eradication of child-labour in hazardous industries in India. But the truth is that despite all the rhetoric, neither the government so far has succeeded in eradicating this evil, nor has been able to ensure compulsory primary education for every Indian child. Between 60 and 100 million children are still at work instead of going to school, and around 10 million are working in hazardous industries. India has the largest child population of 380 million in the world, plus the largest number of children who are forced to earn a living.
  2.  We have many laws that ban child-labour in hazardous
    industries. According to the Child-Labour (Prohibition
    and Regulation) Act 1986, the employment of children below the age of 14 in hazardous occupations has been strictly banned. But each state has different rules regarding the minimum age of employment. This makes implementation of these laws difficult.
  3.  Also, there is no ban on child-labour in non-hazardous occupations. The Act applies to the organised or factory sector and not the unorganised or informal sector where most children find employment as cleaners, servants, porters, waiters etc. among other forms of unskilled
    work. Thus, child-labour continues . because the
    implementation of the existing laws is lax.
  4.  There are industries, which have a special demand for child labour because of their nimble fingers, high level of concentration and capacity to work hard at abysmally low wages. The carpet industry in U.P. and Kashmir employs children to make hand-knotted carpets. There are 80,000 child workers in Jammu and Kashmir alone. In Kashmir because of the political unrest, children are forced to work while many schools are shut. Industries like gem cutting and polishing, pottery and glass want to remain competitive by employing children.
  5.  The truth is that it is poverty which is pushing children into the brutish labour market. We have 260 million people below the poverty line in India, a large number of them are women. Poor and especially woman-headed families, have no’option but to push their little ones in this hard life, in to hostile conditions, with no human or labour rights.
  6.  There is a lobby which argues that there is nothing wrong
    with children working as long as the environment for i work is conducive to learning new skills, but studies have
    shown that the children are made to do boring, repetitive and tedious jobs and are not taught new skills as they grow older. In these, hell-holes like the sweet shops for the old there is no hope.
  7.  Children working irr hazardous industries are prone to debilitating diseases which can cripple them for life. By sitting in cramped, damp and unhygienic spaces, their limbs become deformed for life, inside matehstick, fire¬works and glass industries they are victims of bronchial diseases andT.B. Their mental and physical development is permanently impaired by long hours of work. Once trapped, they can’t get out of this vicious circle of poverty. They remain uneducated and powerless. Finally, in later years, they too are compelled to send their own children to work. Child-labour perpetuates its own nightmare.
  8.  If at all the government was serious about granting children their rights, an intensive effort ought to have been made to implement the Supreme Court’s Directive of 1997, which laid down punitive action against employers of child-labour. Only compulsory primary education can eliminate child-labour,
  9.  Surely, if 380 million children are given a better life and elementary education, Indians human capital would be greatly enhanced. But that needs, as former President Abdul Kalam says, “a second vision”.

(a) (i) On which two accounts has the government not succeeded so far in respect of children ?
Answer : The government has not succeeded so far in respect of children on account of eradication of child-labour in hazardous industries in India and to ensure compulsory primary education for every Indian child.
(ii) What makes the implementation of child-labour law
difficult ? ,
Answer : Each state has different rules regarding the minimum age of employment. Also, there is no ban on child-labour in non-hazardous occupation, The Act does not apply to the unorganized or informal sector. This makes the implementation of child-labour law difficult.
(iii) Why do the industries prefer child-labour ?
Answer : There are industries which have a special demand for child-labour, it is because of their nimble fingers, high level of concentration and capacity to work hard at low wages.
(iv) What are the adverse effects of hazardous industries
on children ? Give any two.
Answer: Children working in hazardous industries are prone to debilitating diseases which can cripple them for life. By . sitting in cramped, damp and unhygienic spaces, their limbs become deformed.
Secondly, they become victim of bronchial diseases by working in matehstick, fire-works and glass industries.
(v) What does the Supreme Court’s Directive of 1997 provide ?
Answer : The Supreme Court’s Directive of 1997 laid down punitive action against employers of child-labour. Only compulsory primary education can eliminate child-labour.
(b) Find words from the passage which mean the same as the following ?
(i) risky/dangerous (para 1)
Answer: hazardous .
(ii) very unfriendly (para 5)
Answer: hostile
(iii) intended as punishment (para 8)
Answer: punitive

Question.2. Read the passage given below and answer the questions that follow:
There is nothing more frustrating than, when you sit down at your table to study with the most sincere of intentions and instead of being able to finish the task at hand you find your thoughts wandering. However, there are certain techniques that you can use to enhance your concentration. “Your concentration level depends on a number of factors,” says Samuel Ghosh, a social counsellor. “In order to develop your concentration span, it is necessary to examine various facets of your physical and internal environment,” she adds.
To begin’ with one should attempt to create the physical environment that is conducive to focussed thought. Whether it is the radio, TV or your noisy neighbours identify the factors that make it difficult for you to focus. For instance, if you live in a very noisy neighbourhood, you could try to plan your study hours in a nearby library.
She disagrees with the notion that people can concentrate or study in an environment with distractions like a loud television, blaring music etc. “If you are distracted when you are attempting to focus, your attention and retention powers do not work at optimum levels,” cautions Ghosh. “Not more than two of your senses should be activated at the same time,” she adds. What that means is that music that sets your feet tapping is not the ideal accompaniment to your books’?
Also do not place your study table or desk in front of a window. “While there is no cure for a mind that wants to wander, one should try and provide as little stimulus as possible. Looking out of a window when you are trying to concentrate will invariably send your mind on a tangent,” says Ghosh.
The second important thing, she says, is to establish goals for oneself instead of setting a general target and then trying to accomplish, what you can in a haphazard fashion. It is very important to decide what you have to finish in a given span of * time. The human mind recognises fixed goals and targets and appreciates schedules more than random thoughts”. Once your thoughts and goals are in line, a focused system will follow.
She recommends that you divide your schedule into study and recreation hours. When you study, choose a mix of subjects that you enjoy and dislike and save the former for the last so that you have something to look forward to. For instance, if you enjoy verbal skill tests more than mathematical problems, then finish Maths first. Not only will you find yourself working harder, you will have a sense of achievement when you wind up.
Try not to sit for more than 40 minutes at a stretch. Take a very short break to make a cup of tea or listen to a song and sit down again. Under no circumstances, should one sit for more than one and a half hours. Short breaks build your concentration and refresh your mind. However, be careful not to overdo the relaxation. It may have undesired effects.
More than anything else, do not get disheartened. Concentration is merely a matter of disciplining the mind. It comes with practice and patience and does not take very long to become a habit for life.
(a) On the basis of your reading of the above passage, make notes on it in points only using abbreviations wherever necessary. Supply a suitable title.
1. Notes
(i) Tech, to enhance concentration
(a) create phy envirmnt.
(b) identify the factors resp
(c) music not the ideal accomp
(ii) Recommendations
(a) dvd schedule
(b) choose sub that you like or dislike
(c) don’t sit for more than 40 min
(iii) More suggestions
(a) don’t get dishrtnd
(b) concentrn is merely a discipline of mind
(c) rqurs practice and patience

Abbreviations used
Tech                technique
Phy                  physical
Envirmnt       environment
Resp                responsible
Accomp          accompaniment
Dvd                 divide
Dishrtnd        disheartened
Concentrn     concentration
Rqurs              requires
Title : Concentration – Key to Success (or) Techniques to Enhance Concentration
(b) Write a summary of the above passage in about 80
words.

SUMMARY

According to Samuel Ghosh, a social counsellor, study needs concentration which depends on various factors. These include physical environment and avoiding distractions.
Music should not be listened as it lessens the ‘attention and retention power. One should -not keep study table or desk in front of a window to provide little stimulus to mind. Another important thing is to set goals. Once thoughts and goals are in line, a focused system will flow. Ghosh recommended that the schedule to study and recreation hours should be divided. Choose the subject first which is disliked. Do not sit for more than 40 minutes, take a break in between and do not get disheartened.

SECTION B
(ADVANCED WRITING SKILLS)

Question.3. You are Srinivas/Srindhi of D.P. Public School, Nagpur. As Student Editor of your school magazine, draft a notice in not more than 50 words for your school notice board inviting articles/sketches from students of all classes.
OR
You want to rent out your newly constructed flat in the heart of the city. Draft an advertisement in not more than 50 words to be published in ‘The Deccan Herald’, Bangalore under classified columns. Give all the necessary details. You are Mohan/Mahima of Jayanagar, Bangalore.
Answer:
cbse-previous-year-solved-papers-class-12-english-delhi-2011-7
cbse-previous-year-solved-papers-class-12-english-delhi-2011-8

Question.4. You are cultural secretary of PND Xavier School, Jamshedpur. Your school organized a debate on the topic, ‘The impact of reality shows on the younger generation.’ Write a report in 100-125 words to be published in ‘The Times of India’, Jamshedpur.
OR
A major bus mishap which left several people seriously injured took place at Nicholas Road, Nungambakkam, Chennai. Luckily no life was lost. Collect the information from the eye witnesses and send a report in 100-125 words to ‘The Nungambakkam Times. You are Vinod/Vinodini, reporter.
Answer:
‘The Impact of Reality Shows on the
Younger Generation’
Pnd Xavier School, Jamshedpur’
By : Xyz (Cultural Secretary)
‘The Times Of India’, Jamshedpur 30th January, 2014.
PND Xavier School, Jamshedpur hosted the inter-school debate competition in which 27 schools participated. The panel of judges was formed by special correspondent, NDTV,
Chief Justice Mr. Verma and English Lecturer of Jamshedpur College. Coordinator CBSE board was* the’ chief guest. Our Principal welcomed the guest with garland and inspirational speech on the influence of media on children. The first round consisted of 16 students from different schools. In the second round there were shortlisted 8 students who presented their views on the impact of reality shows. They told that reality shows are the latest rage on the television, that are challenging, thrilling and exciting. Above all they have made a star of the common man. Finally 4 students were left, out of whom, 3 were awarded the winners and 1st and 2nd runners up awarded prizes. The winner was given the prize on the basis of concluding lines in which he told that reality shows prompt the younger generation to do stunts at home, thus putting their life in danger, so they should be watched under the vigilance of parents.
OR
A Major Bus Mishap
Report By: Vinodini ‘The Nungambakkam Times 20th August, 2014
It’s alarming that road accidents have been taking place almost daily. The rising toll of accidents indicates, that it is only due to violation of traffic rules and absolute negligence. A similar accident took place at Nicholas Road, Nungambakkam, Chennai, according to an eye witness, a bus going towards the main city, collided with a truck laijen with steel rods. Before the truck driver could do anything, the truck rammed into the bus from the right side and damaged the whole side. The seats were completely torn and the passengers were thrown out of their seats. Fortunately all the passengers were saved but were badly injured. The witness called the police helpline and within few minutes the patrolling police arrived with three ambulances. The injured were immediately rushed to the hospital where a team of expert doctors treated them. The police registered a case against the driver for careless and negligent driving.

Question.5. Write a letter to the Dean, D.P. I School of Management, Mumbai requesting him to apprise you of the details such as eligibility criteria, fees, hostel facility, prospects of placement etc. for admission to P.G. Diploma in HRM. You are Ram/Ramola of 21 Civil Lines, Bareilley.
OR
Write a letter to the Editor, Deccan Times’, Bangalore about the inadequate parking facilities in the Commercial Street, M.G. Road which is causing a lot of inconvenience to the people. Offer your sugeestions. You are Anoop/Ritu, 24 Hennus Road, Bangalore.
Answer:
21, Civil Lines ,
Bareilley
25th April, 2014
The Dean
D.P. I School of Management
Mumbai
Subject: Enquiry about the HRM course Dear Sir,
I have come to know that your institute provides excellent training in Post Graduate Diploma in ‘Human Resource’ Management. I came to know about this from the National newspaper’s classified columns and thus developed a keen interest in doing the course. In this connection, I would like to put forth that I have done my schooling from CBSE board in the stream of Commerce with 92 percent and graduated from the eminent city college with 87 percent.
It would be great on your part if you could kindly let me know the eligibility criteria, course fee, duration of the course and timings. Beside that, I am interested to know about the hostel facility as, I am an outsider and would be requiring hostel to stay in.
I would be highly obliged if you could please send me the prospectus at the earliest, so that I can avail this opportunity. Yours truly,
Ramola
OR
24, Hennus Road Bangalore,
12th November, 2014 The Editor Deccan Times,
Bangalore
Subject: Inadequate Parking Facility Dear Sir,
Through the columns of your esteemed newspaper, I would like to express my views on the inadequate parking facilities in the commercial street, M.G. Road which is causing a lot of incoveince to the people. There is not a single day when the street is without chaos. It is due to lack of space as well as lack of sense of responsibility. As it is a very busy area, vehicles are parked all over in a haphazard manner. Even the shopkeepers have to manoeuvre their way to make a safe passage. It is -» necessary to start an awareness campaign to enlighten the sense of parking in addition to the proper parking area and the Municipal Corporation should assign the duty to responsible people who could control the traffic properly.
Hope my letter will find place in your newspaper.
Thanking You
Yours Sincerely
Ritu

SECTION-C

Question.8. Answer any three of the following in 30-40 words each :
(a) Do you think the poet Pablo Neruda advocates total inactivity and death ? Why ? Why not ?
Answer : No, the poet Pablo Neruda doest not advocate total inactivity and death. He only wants us to remain quiet for sometime. So, that we can asses ourselves and stop destructive activity in favour of mankind. Then there would be no war or violence.
(b) Why and how is Grandeur associated with the mighty dead ?
Answer: Grandeur is associated with the mighty dead because the huge tombs and buildings are erected in the memory of great personalities and they become more important than they were when alive. Thus Grandeur is associated with their death.
(c) X
(d) How do denizens and ‘chivalric’ add to our understanding of the tigers’ attitude?
Answer : These two words mean inhabitants and fearless which are applicable to the tigers with reference to the poem. The tigers are the king of jungle and they are suppossed to be the bravest species on earth. They have a respectable position in the animal kingdom. So they are fearless inhabitants which also symbolizes Aunt Jennifer’s attitude.

Question.9. Answer the following in 30-40 words each :
(a) What was Franz expected to be prepared with for the school that day ?
Answer : Franz was expected to be prepared with rules for the use of participles, but he came to the school thinking that he would be punished as he had not learnt his lesson.
(b) Why was Douglas determined to get over his fear of
water ?
Answer : Douglas could not enjoy water sports like canoeing, boating and swimming because of his fear of water; therefore he wanted to overcome his fear. Although he wanted to get into the waters of cascades but his fear held him back. He felt paralyzed at the very thought of water, so he decided to overcome it.
(c) Why was the crofter so talkative and friendly with the peddler ?
Answer : Crofter was alone in his house and he needed a company desperately. So, when the peddler came to spend the night in his house, he wanted to share his feelings with him and peddler had sufficient time to listen to him.
(d) *
(e) Why didn’t Sophie want Jansie to know about her story with Daney ?
Answer: In Sophie’s view, Jansie was a stupid girl who had no sense of keeping any secret. If she had told her about meeting with Daney, there was possibility that Jansie would tell it to everybody and her father would be angry.

Question.10. Answer the following in 125-150 words :
Give a brief account of the life and activities of the people like Saheb-e-Alam, settled in Seemapuri.
Answer : Saheb, was a poor boy whose name contradicted his status which meant the lord of the world. He was from a poor
family from Bangladesh. In order to earn livelihood his family
Question is not included due to change in present syllabus. shifted to Delhi where they settled in the Seemapuri area. But when they found no work to do they became rag pickers. They had to search the garbage like others for something to make their livelihood. Saheb, whose age was to enjoy the life, go to school and live his childhood, also looked into the heaps of garbage for some coins. This was a tedious task as they had to leave their houses barefoot early in the morning, with a bag on their shoulders. They did not have chappals, so they were not in a habit to wear them. -Since their fields and homes were swept away in the flood in Bangladesh, they came to the big city with a hope. But here’also they led a miserable life.

Question.11. Answer the following in 125-150 words :
The lesson ‘On The Face of It’, is an apt depiction of the loneliness and sense of alienation by people on account of a disability. Explain.
OR
What precautions were taken by the prison authorities to ensure that the German exam was conducted smoothy and also under strict security?
Answer : The lesson ‘On the Face of It’ is a heart touching story about two different people who had entirely different attitudes towards life. Both were handicapped and were leading a secluded life but they looked at the world with different angles. Derry, a young boy of fourteen had a burnt face whereas Mr. Lamb had a tin leg. Derry was withdrawn from the society as he had faced people who were scared of him due to his ugly face or felt disgusted. Therefore, he began to avoid people and locked himself up in a room. Similarly Mr. Lamb led a lonely life after he lost his leg in war. Children called him ‘Lamey Lamb’, but he adjusted himself with this impairment. He even supported Derry and gave him courage and confidence to face his handicap boldly.
Thus it is true that society is more inclined to the healthy person and they outcaste the one who is disabled. Physical pain is much more bearable than the pain of alienation by the society.
OR
The Governor of Oxford Prison took all the necessary precautions to conduct the exam. It was arranged in the cell only and the doors were closed a day before. All the possible actions were taken so that Evans, who had escaped earlier too should not be able to do this time. Jackson, the senior prison officer had himself searched all the places. Evans nail scissors and nail file had been removed from the cell to avoid any risk. After he had done his shaving, his razor had also been removed away from there. The Governor, could listen to all the conversation going on in the cell through a device that was kept above the door. He remained in the office to listen to the conversation so that no mishap took place. Stephen, was appointed at the door of the cell to peep through the hole and Rev. McLeery was. thoroughly searched before the commencement of the examination. Everything was properly checked and everyone was perfectly positioned.

Question.12. Answer the following in 30-40 words :
(a) **
(b) How did Dr. Sadao ensure that, the American sailor left his house but he himself remained safe and secure?
Answer : Dr. Sadao was a docor by profession, so it was his duty to save a person’s life whether he was an enemy or not. He treated the American war prisoner as a doctor and on the other hand informed the General as a true patriot. He sought his help to get rid of him but then let him escape.
(c) What part of story did Jack himself enjoy the most and why ?
Answer. Jack used to tell stories to Joe, his daughter before she retired to bed. The basic story remained the same but
the hero was changed. He enjoyed the part only when Joe listened carefully without disturbing him. Otherwise he always got irritated.
(d) What did Zitkala-Sa feel when her long hair was cut ?
Answer : Zitkala-Sa belonged to one of the tribes, of native Americans and their tradition was to keep long and heavy hair. When her long hair was cut, Zitkala-Sa felt exteremely hurt. Keeping short hair at that time meant the sign of mourning and also the hair of prisoners of war shingled by captors.

SET II

Note s Except for the following questions, all the remaining questions have been asked in previous sets.
7. Read the extract give below and answer the questions that follow:
…………..but soon
Put that thought away, and Looked out at young
Trees sprinting, the merry children spilling
Out of their homes,………..
(a) Which thought did the poet put away?
Answer : The poet put away the thought of looking at her mother.
(b) What do the ‘sprinting trees’ signify?
Answer : The sprinting trees in the poem signify the young age. They appear to be running fast when seen from the vehicle in motion. Similarly, the young age of poet’s mother is also passing fast.
(c) What are ‘the merry children spilling out of their homes’ symbolic off ?
Answer : In order to avoid looking at her mother’s pale face, the poetess looks outside the car. She watches the young and happy children coming out of their homes in open to play. These children symbolize young age, energy and life.

Question.5.You are Rohit/Yasmin, a resident of 10, Civil Lines, Saharanpur. During your college life, you participated in various curricular and co-curricular activities with meritorious performance. Write a letter to the Principal of your college i.e. Gurunanak College, Ambala requesting him to issue you a testimonial including the details of your performance.
OR
 As Cultural Secratary of G.D. Birla School, Jamshedpur, you are. organizing an inter school poster marking competition, in connection with the Diamond Jublee Celebratins of your school. Draft a circular letter to be . sent to various schools in your city requesting their participation. Invent details.
Answer:
10, Civil Lines
Saharanpur .
14th August, 2014 The Principal
Gurunanak College,
Ambala
Subject: Issuing of testimonials Dear Sir,
This is to inform you that I have been a student of your college in the year 2007, and passed out in the year 2010. During my college life, I took part in several curricular and co- curriculared activities and achieved a great amount of success. I represented the college in the swimming competition on the regional level and won a gold medal for the college. The State—level tennis championship was also won by me. In the inter-college debate competition too, I was awarded the first prize and won the trophy. My photos also appeared in ‘The Express’. Now, I have to apply for the job and I hope these credentials will be helpful for me.
I would be highly obliged if you could kindly issue me the testimonials, including the details of performances achieved by me.
Thanking You Yours Sincerely,
Yasmin
G.D. Birla School, Jamshedpur We are pleased to inform all the schools of the city that G.D. Birla School, Jamshedpur is organizing an inter-school poster making competition, in connection with the Diamond Jublee Celebrations of the school on Friday, 28th August 2014 at the school playground from 10 a.m. to 2 p.m. The theme of the competition is ‘Green India, Clean India’. There is no participation fee. The chief guest for the function is Police Commissioner of the city, Mr. Jay Sharma. The prize distribution ceremony will be followed by an inspiring speech by our honorable Principal Sir.
The Management, staff and students of the school cordially invite you to this colourful event. Hope, you will provide full support by joining this event and make it more exciting.

6. Corruption has become the order of the day. It has entered all walks of life. Write an article for your school magazine in 150-200 words on ‘The Role of Youth in Combating Corruption5. You are Vijay/ Vineeta.
OR
You are Sanjay/ Geetha, a student of St. Martha’s Sr. Sec. School, Ghaziabad. You have read the news about the inhuman treatment meted out to the new entrants by the seniors in educational institutions. You are shocked to read about it. Write an article in 150-200 words on ‘The Evil of Ragging in Educational Institutions’ to be published in ‘The Hindu5, Delhi.
Answer:
‘The Role of Youth in Combating Corruption’
By Vineeta
We know that the problem of corruption is spreading everywhere like mushrooms and these create economic crisis. Ultimately, the poor get poorer and the rich get richer. If these problems cannot be solved then the fate of country will be doomed. People are becoming greedy for the need of more and more money and for this they are using all the unfair means. It- is high time that the young people should come forward and take some action. This is their duty to care about their future. It is said that “Youth is a spark which can either burn or lighten the country.” Today there is tremendous pressure on the youth of our country in almost every field. It is their duty to carry a nation forward. Although it is the duty of every citizen but the eyes are predominantly on the youth. They should root out the corruption keeping in mind that no one is above law. If one thinks about the success of the country, then automatically corruption will be rooted out. It is up to the youth to derive inspiration from their ideals high. The higher the goal, the greater will be the potential to do the work and fruitful will be the results.
OR
‘The Evil of Ragging in Educational Institutions’
By Geetha (Student, St. Martha Sr. Sec. School)
The Hindu ‘
Ragging in educational institutions, is a distorted form of introduction of the juniors by their seniors. It involves insults in physical as well as verbal terms. There are many complex activities which are supposed to be done by the ‘ juniors which make them feel humiliated. It has become increasingly unpopular due to several complaints of serious injuries to the victims and strict laws regarding ragging. In other words, we can say that ragging is an extreme form of bullying that is omnipresent not only in schools but social
areas also. Few days ago, I came across a news in which a group of seniors, from a reputed college ragged their juniors savagely and they carried on their activities of hooliganism, extortion, drug peddling etc. throughout Although an FIR was lodged against these rogues, but the torture suffered by the new entrants broke them completely before their college life began. It is not a mere news but, it gives clear message about the evil of ragging -that is flourishing in educational institutions. We must be responsible and sensible enough to think hard as to how the educational institutions be made an altar of study, and not, the breeding ground for criminals. As parents, teachers and citizens, we should take a step forward to stop such inhuman activities. .

Question.9. Answer the following in 30-40 words each :
(a) What had been put up on the bulletin board?
Answer : The bulletin board contained an order from Berlin, that from then onwards, German will be taught in the schools of Alsace and Lorraine.
(b) What did Douglas experience when he went down to the bottom of the pool for the first time?
Answer : When Douglas went down to the bottom of the pool for the first time, he jumped. He did not spring upwards to the surface, but surfaced gradually. His eyes and nose were out of water. He tried to catch a rope but failed.
(c) Why was the crofter so talkative and friendly with the peddler?
Answer: Crofter was alone in his house and he was in need of a companion desperately. So when the peddler came to spend the night in his house, he wanted to share his feelings with him and peddler had sufficient time to listen to him.
(d) Did Geoff keep his promise to Sophie? How do you know?
Answer : Yes, Geoff kept his promise as he talked very less. Most of the time he remained silent and also knew that his sister had big dreams of becoming a manager, and she used to form wild stories, so he never shared anything with anybody. Thus, we know that he had kept his promise.

12. Answer the following in 30-40 words each:
(a) What sort of brightness and silence prevailed in Antarctica during summer?
Answer : During the summer there was a surreal 24-hour austral summer light, and a universal silence, that was interrupted only by the occassional avalanche or calving ice sheet.
(b) Why did Hana wash the wounded soldier herself?
Answer : Hana had to wash the wounded man herself as her domestic maid Yumi refused to do so because the man, was an enemy. Not only this, but all the servants were against Dr. Sadao and his wife Hana in this respect, so they all left the house.

SET III

Note : Except for the following questions, all the remaining questions have been asked in previous sets.

Question.5.Judith Public School Pune has recently introduced Diploma Course in Lab Technology under vocational stream. The prospects of this course are not clear to your mind. Write a letter to the Principal of the school inquiring about the future prospects of the course and requesting him to send prospectus if any. You are Cham an/ Chanchal, 25 Ashirwad Apartments, Kolhapur.
OR
Write a letter to the Municipal Chairman of New Town, North Arcot District about the dilapidated condition of public monument requesting him to take urgent steps for the preservation of this valuable heritage. You are Anar kali/ Akbar, 10, Pudupet, New Town.
Answer:
25, Ashirwad Apartment Kolhapur
15th April, 2014 The Principal
Judith Public School,
Pune
Dear Sir,
Subject : Inquiring about the Diploma Course in Lab Technology
I, have come to know that your institution has recently introduced Diploma course in Lab Technology under vocational stream. I saw your advertisement in the newspaper and I am Very much interested in doing this course. I have completed my schooling from Central Government School this year and have secured a good percentage in the Commerce stream. Since I don’t plan on sitting idle long, hence I went through the details provided and I have enountened a few doubts regarding the future prospects of the course.
I woukLbe obliged, if you could provide me clear details on the prospects of the course along with the duration, charges, timings and the date of the commencement of the course and send the prospects on the above given address, so that I can avail the opportunity of this course.
Thanking You Yours Sincerely „ Chanchal
OR
10, Pudupet
New Town
24 th December, 2014
The Municipal Chairman New Town,
North Arcot District
Subject: Dilapidated condition of Public Monument Dear Sir,
This is to bring to your notice that our Cultural Heritage is taking a bad shape. India is known for its varied and vast culture and therefore, it is the best counted tourist destination among the other countries. Since ages, the monuments like Sanchi Stupa, Nalanda, Mahabalipuram Rathas, Khajuraho Temples, Hampi are bearing their own grand history. Many folktales are connected with these Forts, but these are not properly conserved and protected. The situation is alarming as the structures in the city are in dilapidated condition. City’s unprotected monuments are dying a slow death. This pathetic situation’ can be overcome only when the Department of Archaeological Survey of India would undertake this responsibility. In addition to it, if the respected Chairman takes some steps to prevent deterioration of these monuments urgently.
Hope, my letter would help in awakening the consciousness of the Government as well as the related authorities. Thanking You
Your Sincerely .
Anarkali

Question.6.There is a growing trend among the youth to participate in adventurous activities. Write an article in 150-200 words for your school magazine on the topic, ‘Why would you prefer to have an adventurous life?’
OR
. Children up to the age of 12 years have been participating in TV programmes, bearing a lot of stress and neglecting their studies at such a prime time of their life. Write an article in 150-200 words on ‘How far is it justified for children to participate in TV programmes?’ You are Sukra/ Sukanya.
Answer:
‘Why would you prefer to have
an adventurous life?’
Adventurous life has attracted youth a lot and it has become the upcoming trend. That is why adventure tourism has grown in decades. Adventurous life excites and rejuvenates a man, hence more and more youth are fascinated by it. Experiencing the life in the lap of nature rock, climbing, river rafting, jungle safari, skiing etc. is totally a different experience. As the lifestyle is getting more stressful, people want a vacation that helps them to relax. Now a days
adventure tour operators are ready with exciting packages. 9. Earlier, trekking and mountaineering Were supposed to be done by the NCC Cadets only, but the boundaries are now broken and open for a common man. Adventure in life is a mix of holiday along with the beauty of nature thus making it an adventurous life. It is a getaway from the tiresome life of the city and find something thrilling that rejuvenates both mind and body. State tourism offers variety of adventure holidays. Besides, there are challenging safaris to support the interests of the youth.
“How far is it justified for children to
participate in TV programmes?”
By: Sukanya
The unbearable pressure of competitions, has provoked the , children to do something ay a very early stage in their life. Taking part in the television programmes is the result of such unhealthy competitions. This is the time when they should concentrate more on studies but their time is consumed or rather wasted in such participations. The children are so obsessed with the programmes, that they do not pay least attention towards their books. They are occupied with their schedule so much that after returning home they have least 12. stamina to study. Thus, the golden time of their life is invested in doing TV programmes and entertainment. Others enjoy their act but how many of us bother about their prospects?
What would they become when they are grown up, as it is not always possible for a child artisr-to gain same success on maturity. People should first prioritize their basic studies and then come into this line. Therefore, in my opinion it is not justified for children below the age of 12 years to be involved in TV programmes.

Question.9.Answer the following in 30-40 words each :
(a) Who did M. Hamel blame for the neglect of learning on the part of boys like Franz?
Answer: Mr. M. Hamel blamed the parents for the neglect of learning on the part of boys like Franz who got their children engaged in the work of farming. He too assigned them the work of gardening, so boys couldn’t learn French.
Question.(b) Why did Douglas go to lake Wentworth in New Hampshire?
Answer : Douglas went to Fake Wentworth in New Hampshire to overcome his fear of water, as he was not sure that terror had left him even after the training. He made it a point by swimming two miles in the lake all alone to get rid of his fear.
(e) How did Sophie’s father react when Geoff told him about her meeting with Daney Casey?
Answer : Sophies father was not a day dreamer like her. He knew that his daughter had a habit of weaving wild stories but he was a realist. When Geoff told him about her meeting with Daney Casey, he did not show any concern. He was just worried that this would land Sophie in trouble one day.

12.Answer the following in 30-40 words each :
(a) Chapter deleted
(b) How does the writer indicate that Dr. Sadao’s father was a very traditional and conventional man?
Answer: Dr. Sadao’s father was a traditional and conventional man as he thought that Japan’s future depended on the islands. His only aim was to educate his son, so he was serious with him everytime. Although he was a strict father but he cared for his son a lot. Moreover, he had strong belief in arranged marriages.

Solved CBSE Sample Papers for Class 12 Political Science Set 9

Solved CBSE Sample Papers for Class 12 Political Science Set 9

[Time Allowed : 3 hrs.]                                                                                              [Maximum Marks] : 100

General Instruction:

  1. All Questions are compulsory.
  2. Question numbers 1-5 are of 1 mark each. The answers to these questions should not exceed 20 words each.
  3. Question numbers 6-10 are of 2 marks each. The answers to these questions should not exceed 40 words each.
  4. Question numbers 11-16 are of 4 marks each. The answers to these questions should not exceed 100 words each.
  5. Question numbers 17-21 are of 5 marks each. The answers to this question should not exceed 150 words.
  6. Question numbers 22-27 are of 6 marks each. The answers to this question should not exceed 150 words.

Question.1. Define ‘Shock-Therapy’.
Answer. The Shock-Therapy was the state of affairs which signifies the collapse of communism followed by a painful process of transition from an authoritarian socialist system to a democratic capitalist system.

Question.2. Why is human security more important in the contemporary world than territorial security ?
Answer. Human Security is more important in the contemporary world than territorial security because this concept of security goes beyond military threats to include a wide range of threats and dangers affecting human existence.

Question.3. Why were the States reorganised on linguistic basis in India in 1956 ?
Answer.

  • The states were reorganised on linguistic basis to accommodate plurality and adopt a flexible approach in dealing with the demands of the regions.
  • It also reduced the threat of division and separatist attitude among states.

Question.4. When and where was the 1st non-Congress state government formed after India’s independence ?
Answer.

  • In March 1957 in Kerala the 1st non-Congress state government was formed after Independence.
  • After the defeat of Congress, CPI formed the coalition government.

Question.5. In which year did the Congress Party win 415 Lok Sabha seats ? Who became the Prime Minister at that time ?
Answer. The Congress party won 415 seats in the Lok Sabha election in 1984. Rajiv Gandhi became the Prime minister.

Question.6. Why is it said that the collapse of Berlin Wall signified the collapse of the bi-polar world ? 
Answer. The Berlin Wall which symbolised the division between capitalist and the communist world was demolished by the people on 9th November, 1989.
This marked the unification of Germany and the beginning of the end of the communist bloc and led to the end of the bi-polar politics.

Question.7. State any one measure to deal with terrorism as a new source of threat to security.
Answer. In the case of terrorism military should be engaged for human security mission because terrorism, which is political violence, targets civilians deliberately and indiscriminately. For example, terrorist attack on US World Trade Center in New York on 11th September 2001.

Question.8. Why do indigenous people need special protection to protect their rights ?
Answer. Indigenous people are the people who help to bring the issues of environment resources and politics together. The UN defines Indigenous populations as comprising the descendants
of peoples who inhabited the present territory of a country at the time when persons of a different culture or ethnic origin arrived there from the other parts of the world and overcame them. . ..

  1. These people need protection because issues related to the rights of the indigenous communities have been neglected in domestic and international politics for very long.
  2. During the 1970s growing international contacts among indigenous leaders from around the world led to a sense of common concern and shared experiences. The World Council of Indigenous Peoples was formed in 1975.

Question.9. Enumerate any two principles of Nehru’s Foreign Policy.
Answer.

  1. The first Prime Minister of India Pt. Jawaharlal Nehru was the “chief architect” of India’s foreign policy.
  2. The three major objectives of Nehru’s foreign policy were to :
    (a) preserve the hard earned sovereignty
    (b) protect territorial integrity, and
    (c) promote rapid economic development.

Question.10. Why did the Communist Party of India split in 1964 ?
Answer. The Communist Party went through a major split in 1964 following an ideological rift between th6 Soviet Union and China.
The pro-Soviet factors remained as the CPI, while the opponents formed the CPI (M) i.e. Marxist. Both these parties continue to exist to this day significantly in the states of Kerala, West Bengal, Punjab, Andhra Pradesh and Tamil Nadu.

Question.11. What is the rationale of the Non-Aligned Movement after the end of Cold War?
Answer. Non-alignment as a strategy evolved in the Cold War context, but the end of Cold War and emergence of a unipolar world did not see the end of the non-alignment.
Non-alignment still contains some core values and end-using ideas. These are :

  1. Its emphasis has shifted from “political issues to economic issues”.
  2. Liberalisation of third world economies for rapid development of the countries of south now remains the main concern of NAM.
  3. Issues like democracy, disarmament, human rights and neo-colonialism are as relevant
    today as earlier.
  4. With the disintegration of the erstwhile USSR, there is only one super power, therefore,
    it is very essential for NAM to make sincere efforts to check USA from taking unilateral decisions.
  5. Now its emphasis has also shifted towards poverty alleviation, New International Economic Order based on equality, equity and justice and promotion of industrialisation. NAM must make efforts to reshape and democratise the United Nations so that the domination of powerful countries is checked.
    Besides, NAM must ensure that in the era of globalisation, liberalism and explosion of Information Technology (IT), the developed and developing nations derive the maximum benefit and are not allowed to be exploited.
    Therefore, the concept of non-alignment is applicable even in normal situation. Its essence is that every international issue is discussed on merit. This is an attitude which must be reflected in international affairs.

Question.12. Why was the US attacked by the terrorists on 11th September, 2001 ? How did the US react to it?
Answer.

  1. The 9/11 attacks were one of the major human disasters. On 11 September 2001, nineteen hijackers hailing from a number of Arab countries took control of four American commercial aircraft shortly after takeoff and flew them into important buildings in the US.
    Two airliners crashed into the North and South Towers of the World Trade Center in New York.
    The third Aircraft crashed into the Pentagon building in Arlington where the US Defence Department is headquartered.
    The fourth aircraft, presumably bound for the capital building of the US Congress came down in a field in Pennsylvania.
    The 9/11 attacks killed nearly three thousand people. The shocking part for the Americans was that they have been compared to the British burning of Washington DC in 1814 and the Japanese attack on Pearl Harbor in 1941. However in terms of loss of life, 9/11 was the most severe attack on US soil since the founding of the country in 1776.
  2. The US response to 9/11 was swift and ferocious. The then President Bush had a much harder view of US interests and of the means by which to advance them.
    (a) As part of its global war on terror, the US launched Operation Enduring Freedom against all those suspected to be behind this attack, mainly Al-Qaeda and the Taliban regime in Afghanistan.
    (b) The US forces made arrests all over the world, often without the knowledge of the government of the persons being arrested, transported these persons across countries and detained them in secret prisons.
    (c) Some of the prisoners were kept at Guantanamo Bay, a US naval base in Cuba where prisoners did not enjoy the protection of International law.

Question.13. “Global commons have not only common, but differentiated responsibilities.” Do you agree with this view ? Why ?
Answer. Common but differentiated responsibility signifies that states have common but differentiated responsibilities to check Global environmental degradation. The developed countries acknowledge the responsibility that they bear in the international pursuit of sustainable development in view of pressures their societies place on the global environment and the technological and financial resources they command.
Conventions and Declarations on Differentiated Responsibilities.
We can implement the ideas with the help of Conventions and Declarations like :

  1. The Rio Declaration of 1992 convened on Environmental issues says that “states shall ‘ cooperate in the spirit of global partnership to conserve, protect and restore the health and integrity of the Earth’s ecosystem.
  2. The 1992 United Nations Framework Convention on Climatic Change (UNFCCC) also provides that the parties should act to protect the Climate system on the basis of equity and in accordance with their common but differentiated responsibilities and respective capabilities.
  3. The Kyoto Protocol is an international agreement setting targets for industrialised countries to cut their greenhouse gas emissions. The protocol was agreed to in 1997 in Kyoto in Japan, based on principles set out in UNFCCC.

Question.14. What is meant by globalisation ? Explain any three cultural consequences of globalisation.
Answer. Globalisation means integrating the economy of a country with the economies of other countries in the process of free flow of trade and capital.
In another way Globalisation means integrating our economy with the World Economy.
As far as cultural consequences are concerned, it would be a mistake to assume that cultural consequences of globalisation are only negative. Culture is not a static thing. All cultures accept outside influences all the time. Some external influences are negative because they reduce dur choices.
But some times external influences simply enlarge our choices and modify our culture without overwhelming the traditional norms. For example burger is no substitute for a masala dosa and therefore does not pose any real challenge.
In the same way blue jeans can go well with a homespun khadi kurta. Here the outcome of outside influences is a new combination, that is unique. This clashing combination has been exported back to the country.
So we can safely say that globalisation broadens our cultural outlook and promotes cultural homogenisation.
The cultural globalisation leads to a fear that this process poses a threat to cultures in the world. The rise of a uniform culture is not the emergence of globalisation or a global culture. What we have in the name of a global culture is the imposition of western culture on the rest of the world.

  1. The culture of the politically and economically dominant society leaves its imprint on a less powerful society, and the world begins to look more like a dominant power wishes it to be.
  2. This is dangerous not only for the poor countries but for the humanity on the whole for it leads to the shrinking of the rich cultural heritage of the entire globe.

Question.15. Highlight the developments in India’s nuclear programme.
Answer. India’s Nuclear policy advocates ‘no first use’ and reiterates India’s commitment to global verification on non-discriminatory nuclear disarmament leading to a nuclear weapons free world.
(a) The nuclear programme in India was initiated in the late 1940s under the guidance of H J. Bhabha.
(b) When China conducted a nuclear test in 1964, India realised its strategic importance.
(c) India’s first nuclear experiment was conducted in 1974 and India declared that it was only for peaceful purposes.
(d) At the same time India refused to sign the CTBT in 1995 because of its discriminatory nature.
(e) India also conducted a series of nuclear tests in May 1998, demonstrating its capacity to use nuclear energy for military purposes.
Our former President A.P.J. Abdul Kalam had written about our nuclear strategy in his book India’s Vision-2020. .

Question.16. Explain the impact of coalition governments on Indian politics.
Answer. With the election of 1989, a long phase of coalition governments began in India. Since then there have been nine governments at the centre, all of which have either been coalition governments or minority governments supported by other parties which did not join the government.
In this new phase any government could be formed only with the participation or support of many regional parties.
The above statement is justified because in the new era of coalition politics, the emphasis of political parties is on pragmatic considerations rather than an ideological positions and political alliance without ideological agreement. For instance :

  1. Coalition politics has shifted the focus of political parties from ideological differences to power sharing arrangements.
  2. Thus most parties of the NDA did not agree with the ‘Hindutava’ ideology of the BJP, yet they came together to form a government and remained in power for a full term.

Question.17. In India hero worship plays a part in its politics unequalled in magnitude by the part it plays in the politics of any other country…. but in politics,……  hero worship is a sure road to degradation and eventual dictatorship. Analyse the statement.
                                                                                                                                          Dr. Babasaheb Ambedkar.
Answer. The above statement of Dr. Babasaheb Ambedkar states that the leaders of Independent India took up the more difficult path to accommodate social diversities. In this way they were deeply committed to the idea of democracy. Leaders did not see politics as a problem; they saw it as a way of solving the problems.
But in society there are always different policy alternatives to choose from. There are different groups with different and conflicting aspirations.
All these differences led to the degradation of the society.

Question.18. “Tomorrow we shall be free from~the slavery of the British domination, but at midnight India will be partitioned. Tomorrow will thus be a day of rejoicing as well as mourning.” Analyse.
                                                                                                    [Mahatma Gandhi 14th August 1947, Kolkata]
Answer.Through the above statement Mahatma Gandhi wanted to express that the partition of the country appeared to prove everyone’s worst fears and there were serious questions about the future of India, i.e., would India survive as a unified country.
Hence, it is well said that 15th August was a day of rejoicing as well as mourning.

Question.19. Read this extract from the Rio Declaration and answer the following questions: “States shall cooperate in the spirit of global partnership to conserve, protect and restore the health and integrity of the earth’s ecosystem. In view of the different contributions of the global environmental degradation, states have common but differentiated responsibilities.”
(a) Give two examples of the ecosystem being talked about here.
(b) Which part of the world has greater responsibility for environmental protection ? And why ?
(c) To what extent was this spirit followed by the states in their action since the Rio Summit ?
Answer.
(a) Climatic change (global warming), Bio-diversity, Global commons, relationship between economic development and environmental management.
(b) The developed countries have greater responsibility for environmental protection because ecological degradation of today in the world is the outcome of their industrial production and their scientific experiments.
(c) Since the Rio summit, most of the states are making efforts to protect the environment.
Forest policy and other environmental programmes have been started. International organisations undertook many projects to encourage such activities and created awareness in cooperation with the concerned states.
Use of non-conventional resources, protection of flora and fauna which are specific to the regions are given importance.

Question.20. Study the political outline map of the world given below in which six countries have been marked as (1), (2), (3), (4) and (5). Identify these countries and name them. Also classify them as First World, Second World and Third World countries. Write your answer in the Answer-Book as per the following format:
No. of the Country
Name of the Country
First World/Second World/Third World
solved-cbse-sample-papers-for-class-12-political-science-set-9-1
Answer.
solved-cbse-sample-papers-for-class-12-political-science-set-9-2

Question.21.What does the cartoon indicates
solved-cbse-sample-papers-for-class-12-political-science-set-9-3
solved-cbse-sample-papers-for-class-12-political-science-set-9-4

  1. What led to the revival of the Congress and its eventual transformation into a mass political party ?
  2. Describe any two reasons for Congress’ clear majority in 1980.

Answer.

  1. The given cartoon indicates the sweeping majority of the Congress party in general and victory of India Gandhi in particular.
  2. The political crises i.e. inability of the party in power to incorporate the aspirations
    of the people.
    The party in power after 1977 election had absolute majority and yet its leadership decided to suspend the democratic process.
  3. The Congress party now (after 1977 political crisis) identified itself with the popular
    ideology-claiming to be the only socialist and pro-poor party.
    The charismatic personality of Indira Gandhi as people appealed for one leader who could provide safeguard to the nation.

Question.22. “Examine any six factors which led to the disintegration of the former Soviet Union.
Or
‘It is not enough to have representative form of democracy. It is necessary to participate in popular movements to make democracy a success.’ Do you agree with this view ? Why ?
Answer. The sudden disintegration of Soviet Union, the second world power was the matter of shock or surprise to the world. The causes and reasons for the collapse were very much inherent in the Soviet system and the Soviet Socialist Bloc. Undoubtedly it is said that the internal weaknesses of Soviet political and economic institutions, which failed to meet the aspirations of the people, were responsible for the collapse of the system.
Economic stagnation for many years led to serve consumer shortages and a large section of Soviet society began to doubt and question the system openly.
The causes of the disintegration of Soviet Union can be analysed as under:
solved-cbse-sample-papers-for-class-12-political-science-set-9-5
During the cold war era the Soviet Union gradually became stagnant in an administrative and political sense due to various factors like

  1. the communist party that had ruled the Soviet Union for over 70 years was not accountable to the people.
  2. ordinary people were alienated by slow and stifling administrati 1, rampant corruption, the inability of the system to correct mistakes it had made, the unwillingness to allow more openness in government and the centralisation of authority in a vast land.
  3. worse still, the party bureaucrats gained more privileges than ordinary citizens.
  4. people did not identify with the system and with rulers and the government increasingly lost popular backing.
    All these factors are responsible for the stagnant administrative and political structure. Mikhail Gorbachev was the last leader of the Soviet Union. He introduced economic and political reform policies of Perestroika. The main features of his reforms are as follows:
    (a) Reform polices were based on restricting of administrators system and openness of economic affairs.
    (b) Reform policy also aimed at keeping information and technological development in the USSR abreast of the information and technological revolutions taking place in the west.
    (c) Gorbachev introduced the democratisation of Soviet Union policy with the aim to normalise relations with the west and democratise the Soviet Union to tow free economic affairs in world arena.

In this way, Gorbachev’s reforms promised to deal with the existing problems of economic stagnation and administrative system.
The rise of nationalism and desire for sovereignty in various republics, like Estonia, Latvia and Lithuania, Ukraine, Georgia and others proved to be the final and most immediate cause for the disintegration of the USSR. Nationalist urges and feelings were very much at work throughout the history of the Soviet Union.
However nationalist dissatisfaction with the Soviet Union was strongest in the more “European” and prosperous Russian and the Baltic areas as well as Ukraine and Georgia. Ordinary people IK re felt alienated from central Asians and from each other and also concluded that they were paying too high an economic price to keep the more backward areas within the Soviet Union.

Or
Yes, to some extent movements and protests in a country strengthen democracy. We have mixed reactions, both for and against.
Arguments in favour : The history of movements and protests help us to “understand better the nature of democratic politics”.

  1. We have observed that these non-party movements like Anti-Arrack Movement, Chipko Movement, NBA are neither sporadic in nature nor are these a problem.
  2. These movements came to rectify some problems in the functioning of party politics and should be seen as integral part of our democratic politics.
  3. Popular movements ensured effective representation of diverse groups and their demands. This reduced the possibility of deep social conflicts and disaffection of these groups from democracy.
  4.  Besides, popular movements suggested new forms of active participation and broadened the idea of participation in Indian democracy, e.g., Anti-Arrack movement and “Dalit Panthers Movement”.

Arguments Against

  1. Critics of these movements often argue that collective actions like strikes, sit-ins and rallies disrupt the functioning of the government, delay decision making and destabilise democracy.
  2. The frequency and the methods u_,ed by the movements suggest that the routine functioning of democracy did not have enough space for the voices of these social groups.
    That is perhaps why these groups turned to mass actions and mobilisations outside the election arena. This can be seen in the recent case of the new economic policies.
  3. Yet the real impact of these movements on the nature of public policies seems to be very limited. This is partly because most of the contemporary movements focus on a single issue and represent the interest of one section of society, thus it is possible to ignore their reasonable demands.
  4. Democratic politics requires a broad alliance of various disadvantaged social groups. Such an alliance does not seem to be shaping under the leadership of these movements.
  5. Political parties are required to bring together different sectional interests, but they also seem to be unable to do so. Parties do not seem to be taking up issues of marginal social groups.
  6. Thus, the relationship between popular movements and political parties has grown weaker over the years, creating a vacuum in politics. And in recent years, this has become a major problem in Indian politics.

Conclusion 
Keeping in view both negative and positive arguments, while concluding we can sum up that movements are not only about collective assertions or only about rallies or protests.
They involve a gradual process of coming together of people with similar problems, similar demands and similar expectations.
Movements are also about making people aware of their rights and the expectations that they can have from democratic institutions. Social movements in India have been involved in these educative tasks for a long time and have thus contributed to expansion of democracy rather than causing disruptions. The struggle for the right to information is a case in point.

Question.23. Explain the three types of U.S. hegemony and give suitable examples for each.
Or
‘India and China are emerging as great economic powers.’ Do you agree ? Justify your answer with any three arguments.
Answer. The word Hegemony implies the leadership or predominance of one state. The roots of the word hegemony lie in classical Greece. At that time it was used to denote the preponderant position of Athens vis-a-vis the other city of Ancient Greece.
Presently, hegemony means, world power in the form of military domination, economic power, political clout and cultural superiority.
I. Hegemony as Hard Power : The very (first) nature of American hegemony advocates relations, patterns and balance of military capability between states. It is this notion of Hegemony which signifies military status that is especially applicable to the current position and role of the US in world politics.

  1. The very essence of contemporary US power lies in the overwhelming superiority of its military power. American military dominance today is both absolute and relative. In absolute terms, the US today has military capabilities that can reach any point on the planet accurately and no other power today can remotely match it.
  2. The US military dominance is based on both the higher military spend and on a qualitative gap i.e.a technological know-how.

II. Hegemony as structural Power: signifies the Economic Prospects of the hegemon power. This notion emerges from a particular understanding of the world economy.

  1. The basic idea is that an open world economy requires a hegemon or dominant power to support its creation and existence.
  2. The hegemon must possess both the ability and the desire to establish certain norms for order and must sustain the global structure.
  3. Hegemony in this sense is reflected in the role played by the US in providing global public goods which signifies those goods that can be consumed by one person without reducing the amount of the goods available for someone else.
  4. A classical example of the structural power of the US is the academic degree called the Master’s in Business Administration (MBA). The idea behind this was that business is a profession that depends upon skills that can be taught in a university which is uniquely American.

III. Hegemony as soft Rower: The US hegemony is not only purely military or economic in nature of status but it has its cultural dimension also which specifies the nature of “Hegemony as soft power.”

  1. Here, this sense of Hegemony implies class ascendancy in the social, political and particularly ideological spheres.
  2. In the field of world politics, this notion of hegemony suggests that a dominant power deploys not only military power but also ideological resources to shape the behaviour of competing and lesser powers.
  3. Here, consent goes hand in hand with and is often more effective than coercion. For example, the predominance of the US in the world today is based not only on its military power and economic powers but also on its cultural presence, like most of the dreams of individuals and societies across the globe, are dreams churned out by practices prevailing in twentieth-century America.
    Thus, this third sense of hegemony as soft power is about the capacity to manufacture consent.

Or
China has emerged as the third alternative since its Economic reforms of 1978. China has been the fastest growing economy since the reforms first began there.
(a) Now it is projected to overtake the US as the world’s largest economy by 2040.
(b) Its economic integration into the region makes it the driver of East Asian growth, giving it enormous influence in regional affairs.
(c) Besides, the strength of its economy, together with other factors such as population, land mass, resources, regional location and political influence have added to its power in a significant way and made it the third alternative power in the world sphere.
For India
The same is true regarding India. After the 1991 economic reforms India too has been the fastest growing economy along with China.

  • Its increasing Foreign Direct Investment and foreign exchange makes it a leading global trade partner.
  • Besides, the economic growth, the other factors like human resource development, Infrastructure, government policy, provision of SEZ have added feathers to its economic power.

Question.24. Assess the outcome of planned development towards the foundation of India’s economic growth and land reforms.
Or
Explain any two causes of the partition of India in 1947. Analyse its any four major consequences.
Answer. The term development refers to the process of improving living standards and attaining an economically viable level of industrial production.
It has to be judged by the improvement it makes in the quality of life of the people, creation of a harmonious equality based society and a self-reliant nation.
During the planning years the foundations of India’s future economic growth were laid down :

  1. Some of the largest developmental projects in India’s history were undertaken during this period. These included mega dams like Bhakhra-Nangal and Hirakud for irrigation and power generation.
  2. Some of the heavy industries in the public sector steel plants, oil refineries, manufacturing units, defence production, etc. were started during this period.
  3. Infrastructure for transport and communication was improved substantially. Agrarian sector witnesed a serious attempt at land reforms. Its main features were as follows:
  • The most significant and successful of these was the abolition of the colonial system of zamindari. This has released land from the clutches of a class that had little interest ’ in agriculture and reduced the capacity of the landlords to dominate politics.
  • Attempts were made at consolidation of land i.e. bringing small pieces of land together in one place so that the farm size could become viable for agriculture.

Drawbacks of Land Reforms
But the other two components of land reforms were much less successful.

  1. Though the laws were made to put an upper limit or ‘ceiling’ on how much agricultural land one could own, people with excess land managed to evade or violate the law.
  2. Similarly, the tenants who worked on some one else’s land were given greater legal security against eviction, but this provision was rarely implemented.
    It was not easy to turn these well meaning policies on agriculture into genuine and effective action. This could happen only if the rural, landless poor were mobilised. But the landowners were very powerful and wielded considerable political influence. Therefore, many proposals for land reforms were either not translated into laws, or when made into laws, they remained only on paper. This shows that economic policy is part of the actual political situation in the society.

Or
(a) In the Indian context the word partition signifies the division of British India into India and Pakistan. The process of partition started in 1940 — when the Muslim league propounded the “Two-Nation Theory”.
According to this theory India consisted of not one but two “people”, Hindus and Muslims. Several political developments in 1940s, the political competition between the Congress and the Muslim league and the British role led to the decision for the creation of Pakistan. The partition year 1947 was the year of one of the largest, most abrupt, unplanned and tragic transfers of population that human history has known.
(b) Communal Riots: In the name of religion people of one community ruthlessly killed and maimed people of the other community. There were killings and atrocities on both sides of the border. Cities like Lahore, Amritsar and Kolkata became divided into “communal zones”.
(c) Social Sufferings : People went through immense sufferings. They were forced to abandon their homes and move across borders. Minorities on both sides of the border fled their homes and often secured temporary shelter in “refugee camps”. Thousands of woman were abducted on both sides of the border. In many cases women were killed by their own family members to preserve the “family honour”.Many children were separated from their parents. Those who did manage to cross the border found that they had no home. Hence, for lakhs of these “refugees” the country’s freedom meant life in ‘refugee camps’.
(d) Administrative concerns and Financial strains: The partition saw not merely a division of properties, liabilities and assets or a political division of the country and the administrative apparatus, what alsd got divided were the financial assets and things like tables, chairs, typewriters, paper-clips, books and also musical instruments of police band.

  • The employees of government and the railways were also divided.
  • Above all, it was a violent separation of communities who had hitherto lived together as neighbours. It is estimated that the partition forced about 80 lakh people to migrate across the new border. Between five to ten lakh people were killed in partition related violence.
  • Besides, the partition had also created severe conflict between the two communities, i’.e., Hindus and Muslims. Keeping in view all this, writers, poets and film makers in India and Pakistan have expressed the ruthlessness of the killings and the sufferings of displacement and violence in their novels, short stories, poems and films. They coined a phrase to describe partition – as a “division of hearts”.

Question.25. Why was the national emergency declared in India on June 25, 1975 ? Analyse any three consequences of this emergency.
Or
Evaluate any three factors which were responsible for Indira Gandhi’s achieving a thumping majority in 1971 Lok Sabha elections.
Answer. The Congress Government declared an emergency in response to Jayaprakash Narayan’s nationwide Satyagraha for the resignation of Indira Gandhi. Observing the situation the government decided that a grave crisis had arisen which made the proclamation of a state of emergency necessary.
Thus, on 25 June 1975, the government declared that there was threat of internal disturbances and therefore, it invoked Article 352 of the constitution. Under the provision of this article the government could declare a state of emergency on grounds of external threat or a threat of internal disturbances.

  1. Effect on civil liberties of citizens
    During Emergency the government made extensive use of preventive detention. Using this provision, the government made large scale arrests.
  2. Effect on relationship between the executive and judiciary.
    As the impact of emergency, the Parliament also brought in many new changes in the constitution. In the background of the ruling of the Allahabad High Court in the Indira Gandhi case, an amendment was made declaring that elections of Prime Minister, President and Vice-president could not be challenged in the court.
  3. Effect on the function of Mass Media
    Deciding to use its special powers under the emergency provisions, the government suspended the freedom of the press. Newspapers were asked to get prior approval for all material to be published, i.e. “Press consorship”. For example, apprehending social and communal disharmony, the government banned Rashtriya Swayamsevak Sangh (RSS) and Jamait-e-Islami. Protests and strikes and public agititations were also disallowed.

Or
In the early of 1970s the government of Indira Gandhi gained popularity due to various factors such as:

  1. During this period the government made conscious attempts to project its socialist credentials.
  2. Indira Gandhi vigorously campaigned for implementing the existing land reform laws and undertook further land ceiling legislation.
  3. Not only this, in order to end her dependence on the other political parties, strengthen her party’s position in the Parliament and seek a popular mandate for her programmes, Indira Gandhi’s government recommended the dissolution of the Lok Sabha in December 1970.
  4. The crisis in East Pakistan and the Indo-Pak war leading to the establishment of Bangladesh added one more feather to the popularity of Indira Gandhi.
  5. In this way, Indira Gandhi and her government was seen not only as the protector of the poor and the underprivileged but also as a strong government.
  6.  The Congress was now in power in almost all the states and restored its dominance. It was also popular across different social sections.

Question.26. What is meant by Chipko Movement ? When did it start and where ? What is the significance of this movement in the conservation of the environment ?
Or
‘Towards the end of the 1980s, five major changes took place in Indian political system. In the light of this statement, examine any three changes.
Answer. The Chipko Movement began in two or three villages of Uttarakhand when the forest department refused permission to the villagers to fell ash trees for making agricultural
tools. However, the forest department allotted the same patch of land to a sports goods manufacturer for commercial use.
This enraged the villagers and they protested against the move of the government.

  1. This was a yery unusual form of collective action in which men and women from a village were engaged in early 1973. These villagers were protesting against the practices of commercial logging that the government had permitted.
  2. They used a novel tactic for their protest that of hugging the frees to prevent them from being cut down.

Impact of Chipko Movement
The Chipko Movement soon spread across many parts of Uttarakhand region. Larger issues of ecological and economic exploitation of the region were raised.

  1. The movement achieved a victory when the government imposed a ban on felling of trees in the Himalayan region for fifteen years, until the green cover was fully restored.
  2. Besides, women’s active participation in the Chipko agitation was a very novel aspect of the movement.
  3. But more than that the Chipko Movement which started over a single issue, became a symbol of many such popular movements emerging in different parts of the country during the 1970s and later.

Or
The end of 1980s witnessed major developments which made a long lasting impact on Indian politics. Among the five major changes as follws : three changes were remarkable like

  1. Mandal Issue : The year 1989 marked the end of ‘Congress System’ and the National Front led by V.P. Singh witnessed the rise of ‘Mandal Issue’ in national politics.
    The decision of National Front to implement the recommendation of the Mandal Commission led to violent ‘Anti-Mandal’ protests in different parts of the country.
  2.  New Economic Policy : In 1991, the Congress led by Narasimha Rao, introduced the ‘New Economic Policy’ with liberalisation and globalisation as its main planks. Even though it was criticised by various movements and organisations it was followed by the later governments.
  3. Changes in the Political System : The era of coalitions in 1989 symbolically indicated the decline of Congress, i.e., the end of Congress dominance over the Indian Party system.
    The decade of 1990s saw an era of multi-party system. It also saw the emergence of – powerful parties and movements that represented the dalits and backward castes.
    The role of left parties changed with time regarding alliances. For example, in 1996 they supported BJP then the Congress government and now again they oppose Congress. Since 1990, a long phase of coalition politics began in India. There have been nine governments at the centre, all of which have either been coalition governments (or) minority government supported by other parties. This could be seen as National Front (1989), the United Front (1996) and (1997), the NDA (1999) and the UPA (2004 and 2009).

Question.27. What are the constraints on American hegemony today ? Which one of these do you expect to get more important in the future ?
Or
Suggest any four reforms required to make the United Nations more effective.
Answer. As history reveals every empire declines because of its weaknesses inherent in itself, so the biggest constraints to American hegemony lie within the heart of hegemony itself. Moreover, we can identify “three constraints on American power”, which were actually not in operation in the years following 9/11. Recently all these constraints are slowly beginning to operate.
Institutional Architecture : The very first constraint lies in the institutional architecture of the American state. A system of division of powers between the three branches of government places significant brakes upon the unrestrained and immoderate exercise of America’s military power by the executive branch.
Open nature of American society possesses constraint : The second constraint on American hegemony is also domestic in nature and stems from the open nature of American society. In spite of mass media’s promotion or imposition of a particular perspective on domestic opinion in the US, there is nevertheless a deep scepticism regarding the purposes and methods of government in American political culture.
And this factor, in the long run is a huge constraint on US military action overseas, i.e. towards the “Invasion Policy of America.”
NATO as a constraint on American hegemony: The most important constraint on American hegemony is possessed by NATO. It is the only organisation in the international system that could possibly moderate the exercise of American Hegemony today.
Actually the US has an enormous interest in keeping the alliance of democracies that follow the market economics alive and therefore it is possible that its allies in the NATO will be able to moderate the exercise of US hegemony through their liberal economic policy.
Or
In the era of decolonisation, end of the cold war and globalisation, the NAM members are making demands for more democratisation of the United Nations. Some of the suggestions leading to reforms in the United Nations can be summed up as under:

  1. Role of Third world countries : With the process of decolonisation the newly independent third world countries should be given a major role.
  2. Tenure should be fixed/limited for the permanent members : In the era of globalisation the membership of the Security Council should not be permanent. The maximum tenure should be for a period of ten years.
  3. Strengthened position of the General Assembly: The position of the General Assembly to be strengthened. To avoid overcrowded agenda in the General Assembly, its agenda needs to be shortened.
  4. Reform in the Veto Power : The veto of the permanent Security Council members should be remodelled so as to help Security Council work effectively.
  5. Significant powers to the Secretary General: The Secretary General’s position should be strengthened since he emodies the principles and is the executive arm of the UN. But at the same time the post of the Secretary General of the UN should not be given, twice to the same individual.
  6. Restructuring the Security Council: The UN Security Council should be restructured and democratised, especially its permanent members, so as to give adequate representation to Afro-Asian countries.

CBSE Solved Papers For Class 12 Computer Science (C++) Paper 4

CBSE Solved Papers For Class 12 Computer Science (C++) Paper 4

SECTION ‘A’

Question 1:
(a) Find the correct identifiers out of the following, which can be used for naming variable, constants or functions in a C + + program :
While, FLOAT, Switch, – 123, + Two, Add5, typedef, First_Name
(b) Read the following code and answer the questions (i) and (ii).
void main ()
{
char name [20];
int Roll no =20;
gets (name);
puts (name);
cout«Roll no;
}
What are the header files to be included?
(c) Rewrite the following program after removing the syntactical error(s) if any Underline each correction.
#include<iostream.h>
const int Dividor 5;
void main ()
{
Number =15;
for (int count =1 count =<5; count++, Number -=3)
if (Number %Dividor==0)
cout «nuraber/Dividor «endl;
else
cout« Number+Dividor « endl;
}
(d) Find the output of the following C+ + program: [2]
#include<iostream.h>
void repch(char s [] )
{
for(int i=0;s [i] ! =’\0′;i++)
{
A( ( (i%2) !=0) &&(s [i] !=s [i + 1]))
{
S [ i ] = ‘ @’ ;
cout << “Hello”;
}
else if (s [i]==s [i + 1])
{
S [i + 1]=’ ! ‘ ;
i++;
}
}
}
void main ();
{
char str[]=”SUCCESS”;
cout<<“Original String”<<str<<endl; repch(str);
cout<<“Changed String”<<str;
}
(e) Find and write the output of the following C++ program code : [2]
Note :
Assume all required header files are already included in the program.
typedef char STRING[80]; .
STRING MIXITNOW(STRINGS)
{
int SIZE=strlen (S);
for(int I=0;I<size-1;I+=2)
{
char WS=S [I];
S[I]=S[I+1];
S[I+1]=WS;
}
for (I=1; I<size; I+=2)
if(S[I]>=’M’&& S[I]<=’U’)
S[I]=’@’;
}
void main()
{
STRING Word = “CRACKAJACK”;
STRING Str;
Str = MIXITNOW(Word);
cout<<Str<<endl;
}
(f) In the following program, find the correct possible output(s) from the options: [2]
#include<stdlib.h>
#include<iostream.h>
void main ()
{
randomize ();
char City [ ] [10]={“DEL”, “CHN”, “KOL”, “BOM”, “BNG”};
int Fly;
for (int I=0; I<3; I+ +)
{
Fly=random(2) + I;
cout«City [Fly]«”:”;
}
}
Outputs:

  1. DEL:CHN:KOL:
  2. CHN:KOL:CHN:
  3. KOL:BOM:BNG:
  4. KOL:CHN:KOL:

Solution:
(a) Adds, First_Name
(b) #include<iostream.h>
#include<stdio.h>
(c) #include<iostream.h>
const int Dividor=5;
void main ()
{
int Number = 15;
for (int count=l;
count<=5; count ++) Number-=3;
if (Number % Dividor= = 0)
{
cout«number/Dividor«endl;
}
else
cout<<Number+Dividor <<endl;
(d) Original String SUCCESS <- Hellos@C!ES!
(e) RCCAAKAJCK
(f) The possible out puts of the above program are :
CHN : KOL : CHN :
KOL : CHN : KOL :

Question 2:
(a) What is Encapsulation? Explain it with an example?
(b) Answer the questions
(i) and  (ii) after going through the following class:
class Student {
int class;
char subject [20];
public:
Student () //Function 1
{ class=12;
strcpy (subject, “C++”);
Student (char sub [J]) //Function 2
{
class=12;
strcpy (subject, Sub);
}
Student (int C) //Function 3
classic;
strcpy (subject, “C++”);
Student (char sub [ ], int c) //Function 4
{
class=c;
strcpy (subject, Sub);
};

  1. Write statements in C++ that would execute Function 3 and Function 4 of class Student. [1]
  2. Which feature of Object Oriented Programming is demonstrated using Function 1, Function 2, Function 3 and Function 4 in the above Class Student? [1]

(c) Define a Class Student with the following specifications:

roll-No. integer
name 20 characters
class 8 characters
marks [5] integer
percentage float

Calculate () a function that calculates overall percentage of marks and returns the percentage of marks. [Assume total marks as 500 i.e., max. marks per subject is 100].
Public Members:
Readmarks () a function that read marks and invokes the calculate function, displaymarks () a function that prints the marks.
(d) Answer the questions (i) to (iv) based on the following code: class Trainer
{
char TNo [5], TName [20], Specialization [10];
int Days;
protected:
float Renumeration;
void AssignRem (float) ;
public:
Trainer ();
void TEntry ();
void TDisplay ();
}
class Learner
{
char Regno [10], LName [20], Program [10]; protected:
int Attendence, grade;
public:
Learner ();
void LEntry ();
void LDisplay ();
};
class Institute : public Learner, public Trainer
{
char ICode [10], IName [20];
public:
Institute ();
void TEntry ();
void TDisplay ();
} ;

  1. Which type of Inheritance is depicted by the above example? [1]
  2. Identify the member function(s) that cannot be called directly from the object of class Institute from the following: [1]
    TEntry ()
    LDisplay ()
  3. What will be the size of object of class Institute ? [1]
  4. If class Institute was derived privately from class Learner and privately from class Trainer, then name the member function(s) that could be accessed through, object of class Institute.

Solution:
(a) Data Encapsulation: Wrapping up of data and functions together in a single unit is known as Data Encapsulation.
Example:
class Item /* Class wraps Data & Functions together in a single unit. */.
{
int Ino: char Desc [20] ;
public: void Purchase ();
void Sale ();
};
(b)

  1. For function 3 : student S(10);
    For function 4 : Student C(“cluestech”, 100);
  2. Function overloading or constructor overloading or polymorphism.

(c) class Student {
private: int roll_no;
char name [20];
char clhss [8];
int Marks [5];
float percentage;
float Calculate ()
{
int sum=0;
for (int i=0; i<5; i++)
sum=sum+marks [i];
return (sum/5);
}
public:
void Readmarks ()
{
cout<<“Enter the student’s name”;
gets(name);
cout<<“\n Enter the roll no”;
cin>>roll_no;
cout«”\n Enter the student’s class”;
cin»class;
cout«”\n Enter 5 marks”;
for (int i=0; i<5; i++)
cin>>marks [i];
percentage=calculate();
}
void displaymarks ()
{
cout«”\n Name entered is”; puts (name) ;
cout«”\n Class of the student is”<<class;
cout«”\n Roll number of the student is” <<roll_no;
cout«”\n Marks entered are”;
for (int i=0; i<5; i++)
cout<<marks [i ] «” “;
cout«”\n percentage of the student is”«percentage;
}
};
(d)

  1. Multiple Inheritance is depicted by the given example.
  2. Both the given member functions can be called directly from the objects of the class Institute.
  3. 37 bytes.
  4. The member functions are:
    Institute()
    TEntry()
    TDisplay()

SECTION ‘B’ 

Question 3:
(a) An array A[10] [20] is stored in the memory along the row, with each of the element occupying 2 bytes, find out the memory location for element A[2] [5], if an element A[5] [10] is stored at the memory location 3020. [3]
(b) Write definition for a function SHOWMID (int P[][5], int R, int C) in C++ to display the elements of middle row and middle column from a two dimensional array P having R number of rows and C number of columns.
For example, If the content of array is as follows:

115 112 116 101 125
103 101 121 102 101
185 109 109 160 172

The function should display the following as output:
103 101 121 102 101 116 121 109
(c) Top is a pointer variable pointing to the top element of a Stack, with each node having the following structure declaration:
struct Stack {int Data; Stack*Next;| considering the above explanation, what does the following code do?
int count=0, Sum=0;
Stack*Temp = Top;
while (Temp —» Next!= NULL)
{ count++;
Sum+=Temp -+Data;
Temp=Temp —>Next;
}
count«Sum/count ;
Also find output if stack contains data as 10, 20, 9, 10.
(d) Given a class declaration as below to implement a Queue using a circular array. Complete the class definition with all member functions. [4]
class Queue
{
int F,R;
int Element [100];
Queue (); //To initialize F and R
void Addition (); //this function should check over flow Condition before adding elements
void Deletion () ; //this function should check empty Queue condition before deleting elements
(e) Convert the expression (x*3+y*3+z*3) / (x + y + z) into postfix expression. Show the content of the stack during the conversion.
Solution:
(a) Array is of A [10] [20].
Total rows, m = 10.
Total columns, n = 20 Width of elements, W = 2 bytes.
If it is stored along row
Formula : Address of A [I] [J] = B + W ((I-Ir)n + (J-Jc)) row = 0 & Jc = Lowest number & column = 0 Position of A [5] [10] = 3020 Therefore, 3020 = B + 2 (5 x 20 + 10)
3020 = B + 2 (110)
3020 = B + 220
B = 2800. [IV*]
Now, position of A [2] [5]
2810 + 2 (2 x 20 + 50)
2810 + 2 (45)
2810 + 90
2900.
(b) void SHOWMID (int P[][5],int R,int C)
{
If (R%2 ! =0);
int X = R + 1;
else
X = R;
If (C%2!=0)
int Y = C+1;
else
Y = R;
forCintJ=Q;J<C;J++)
cout«P[X/2] [J]«””;
cout«endl;
for(intl=0;I<R;I++)
cout«P[I] [Y/2]«””;
(c) It will calculate the average of stack values. Output will be 9.
(d) class Queue {
int F,R;
int Element [100];
Queue ()
{
R=Null;
F=Null;
}
void Addition ();
void Deletion ();
} ;
void Queue :: Addition ()
{
int val;
if ((R+l) % 100 = =F)
cout «”Queue is full”;
else
{
R=(R+l)%Element;
Element [R] =Val;
}
}
void Queue :: Deletion ()
{
int val;
if (F!=R)
{
F=(F+1)%100;
val=Element [F];
cout<< “Removed value =” <<val;
}
else
{
cout<< “Queue is empty”;}
}
}
(e) (x*3 + y*3 + z*3)/ (x+y+z)

Symbol Scanned Stack Expression
( (
( ((
x (( x
* ((* x
3 ((* x
+ ((+ x*
y ((+ x* y
* ((+* x* y
3 ((+* x* y
+ ((+ x* y*+
z ((+ x* y*+z
* ((+* x* y*+z
3 ((+* x* y* + z3
) ( x* y* z3 *+
/ (/ x* y* z3 *+

 

Symbol Scanned Stack Expression
( (/( x* y* z*+
x (/( x* y* z*+x
+ (/(+ x* y* z*+x
y (/(+ x* y* z*+xy
+ (/(+ x* y* z*+xy+
z (/(+ x* y* z*+xy+z
) (/ x* y* z*+xy+z+/
) x* y* z*+xy+z+/

Question 4:
(a) Observe the program segment carefully and answer the question that follows :
class Labrecord
{
int. Expno;
char Experiment[20];
char Checked;
int Marks;
pubilc:
void Enter_Exp();
void Show_Exp();
char RChecked()
{
return Checked;
}
void AssignMark(int M)
{
Marks = M;
}
};
void Modify Marks()
{
fstream File;
File.open(“Marks.Dat”,ios::binary:ios::in|ios::out);
Labrecord L;
int Rec=0;
while(File.read((char*)&L, sizeof(L)))
{
if(L.Rchecked()==’N’)
L.Assignmark(0);
else
L.AssignMark(10);
——————–//Statement 1
——————–//Statement 2
Rec ++;
}
File.Close ();
If the function ModifyMarks() is supposed to modify marks for the records in the file MARKS.DAT based on their status of the member checked (containing value either ‘Y or ‘N’). Write C++ statements for the statement 1 and statement 2 where statement 1 is required to position the file write pointer to an appropriate place in the file and statement 2 is to perform the write operation with the modified record.
(b) Write a function in C++ to count the number of uppercase alphabets present in a text file [2] “Article.TXT”
(c) Given a binary file DATABASE.DAT, containing records of the following structure type : [3]
struct product
{
int prod_id;
char grade;
float price;
};
Write a function in C++ that would read contents from the file ‘DATABASE.DAT and creates a file named ‘IMPORTED.DAT’ copying only those records from DATABASE.DAT whose price ranges from 2000 to 5000.
Solution:
(a) Statement 1: File Seekp (size of (L));
Statement 2: File write ((clear*)&L, size of (L));
(b) void Countupper()
{ int count=0; char c’h;
infstream fin (“Article . txt”) ;
while (Ifin.eof () )
{
fin»ch;
if(isupper(ch))
count ++;
}
cout «”No of uppercase characters = “«count;
fin. close () ;
}
(c) void create filed
{
ifstream fin;
ofstream fout;
fin.open (“DATABASE.DAT”, ios : :binary) ;
fout.open(“IMPORTED.DAT”, ios::binary);
product p;
while (fin.read(( char*)&p, sizeof (p) )
{
if(p.price>=2000&& p.price<=5000)
fout.write ((char*) & p, size of (p));
}
fin. close () ;
fout. close ();
}

SECTION ‘C’

Question 5:
(a) What do you understand by primary key and candidate key? [2]
(b) Consider the following tables Item and Customer. Write SQL commands for the statement (i) to (iv) and give outputs for SQL queries (v) to (viii).
Table: Item

I_Id Manufacturer Price Item Name
PC01 ABC 35000 Personal Computer
LC05 ABC 55000 Laptop
PC03 XYZ 32000 Personal Computer
PC06 COMP 37000 Personal Computer
LC03 PQR 27000 Laptop

Table: Customer

C_ID City IJD Customer Name
01 Delhi LC03 N Roy
06 Mumbai PC03 H Singh
12 Delhi PC06 R Pandey
15 Delhi LC03 C Sharma
16 Banglore PC01 K Agarwal
  1. To display the details of those customers whose city is Delhi. [1]
  2. To display the details of Items whose price is in the range of 35000 to 55000 (Both values included) [1]
  3. To display the customer Name, City from table customer and Item Name and Price from table Item, with their corresponding matching I_Id. [1]
  4. To increase the price of all Items by 1000 in the table Item. [1]
  5. SELECT DISTINCT City FROM CUSTOMER; [1/2]
  6. SELECT Item Name, MAX (Price)
    FROM ITEMGROUP BY Item Name;
  7. SELECT Customer Name, Manufacturer [1/2]
    FROM ITEM, CUSTOMER WHERE ITEM.IJd = CUSTOMER.I_Id;
  8. SELECT Item Name, Price *100 VM
    FROM ITEM WHERE Manufacturer = ‘ABC’

Solution:
(a) Primary key: Primary key is the key that uniquely identifies a particular record in a file. i.e. in a table.
Candidate key:

  1. If a relation scheme has more than one primary key, each is called a candidate key.
  2. One of the candidate keys is arbitrarily designated to be the primary key and the others are called secondary keys.

(b)

  1. SELECT * FROM Customer WHERE City = ‘Delhi’; [1]
  2. SELECT * FROM Item WHERE Price > = 35000 and Price < = 55000;
  3. SELECT Customer Name, City FROM Customer, Item WHERE Customer.I_Id=Item. I_Id;
  4. Update Table Item Set Price=Price+1000;

City

Delhi
Mumbai
Banglore

6.

Item: Name Price
Personal Computer Laptop 37.000

57.000

7.

Customer Name Manufacturer
K Agarwal H Singh
R Pandey
C Sharma N. Roy
ABC
XYZ
COMP
PQR
PQR

8.

Item Name Price
Personal computer Laptop 35.0. 00
55.0. 00

Question 6:
(a) State and define principle of duality. Why is it so important in Boolean Algebra ?
(b) Write the equivalent Boolean Expression for the following Logic Circuit: [2]
cbse-solved-papers-for-class-12-computer-science-c-paper-4-3
(c) Derive a Canonical SOP expression for a Boolean function F, represented by the following truth table:

   p     Q     R    P(P,Q,R)
0 0 0 1
0 0 1 0
0 1 0 1
0 1 1 0
1 0 0 0
1 0 1 0
1 1 0 1
1 1 1 1

(d) Reduce the following Boolean expression using K-map: F(a,b,c,d) = Σ(0,2,3,8,10,11).
Solution:
(a)
Principal of duality : Daulity principle states that from every boolean relation another boolean . relation can be derived by :

  • Changing each or sign (+) to an AND sign (-).
  • Changing each AND sign (-) to an or sign (+) ex. Daul of A + A’B = A.(A’+B)

Importance in Boolean Algebra : The principle of daulity is an important concept in boolean algebra to prove various theorems.
(b) (A+B). (A+B’)
(c) F (P, Q, R) = (P+Q+R).(P+Q’+R’).(P’+Q+R).(P’+Q+R’) OR
F(P,Q,R) = π(0,3,4,S)
Note:
Deduct ½ mark if wrong variable names are used.
(d) F (a,b,c,d) = Σ(0,2,3,8,10,11)
The K-map is:
cbse-solved-papers-for-class-12-computer-science-c-paper-4-1
Hence F(a,b,c,d)= b’c+b’d’ + b’c = b’c + b’d’.

Question 7:
(a) Out of the following which is the fastest

  1. wired and
  2. wireless medium of communication.
    Infrared, Coaxial cable, Ethernet cable, Microwave, Optical fiber. [2]

(b) Expand the following terminologies: [1]

  1. SMS
  2. VOIP.

(c) What is MODEM? [1]
(d) Describe the following in brief: [2]

  1. MOSAIC
  2. Usenet

(e) ABC SWITCH GEARS LTD in Srinagar is setting up the network between its different departments located in different wings. There are 4 wings named as Manufacturing (M). Research (R), Administration (A) and Personnel (P).
Distance between various wings are :

Wing A to wing M 100 m
Wing A to wing R 200 m
Wing A to wing P 400 m
Wing M to wing R 300 m
Wing M to wing P 100 m
Wing R to wing P 450 m

Number of Computers:

Wing M 15 m
Wing R 100 m
Wing A 50 m
Wing P 150 m
  1. Suggest a suitable Topology for networking the computers of all wings.
  2. Name the wing where the server is to be installed. Justify your answer.
  3. Suggest the placement of Hub/Switch in the network.
  4. Mention an economic technology to provide Internet Accessibility to all the wings.

Solution:
(a)

  1. Wired : Optical Fiber
  2. Wireless : Infrared (or Microwave)

(b)

  1. SMS : stands for Short Message Service.
  2. VOIP : Voice-Over-Internet-Protocol.

(c) Modem : (Modulator Demodulator)

  1. It is a device which converts analog signal to digital signals & vice versa.
  2. It is a device used to connect and communicate with other computers.

(d)

  1. MOSAIC: It is an easy method to navigate through internet. It was developed by Mark Anderson in 1993, at NCSA (National Center of Super-Computing Applications) at University of Illinois.
  2. Usenet: It is a group of individuals sharing a particular interest to discuss the views regarding their interest i.e newsgroups.

(e)

  1. Suitable network topology for all wings is BUS Topology.
    cbse-solved-papers-for-class-12-computer-science-c-paper-4-2
  2. Wing P should be used to install server as it has maximum number of computers.
  3. The Hub /Switch should be installed in all the four wings.
  4. The economic technology to provide Internet Accessibility to all wings is co-axial.
    Cable Network or Broadband Network.

Solved CBSE Sample Papers for Class 12 Political Science Set 11

Solved CBSE Sample Papers for Class 12 Political Science Set 11

[Time Allowed : 3 hrs.]                                                                                              [Maximum Marks] : 100

General Instruction:

  1. All Questions are compulsory.
  2. Question numbers 1-5 are of 1 mark each. The answers to these questions should not exceed 20 words each.
  3. Question numbers 6-10 are of 2 marks each. The answers to these questions should not exceed 40 words each.
  4. Question numbers 11-16 are of 4 marks each. The answers to these questions should not exceed 100 words each.
  5. Question numbers 17-21 are of 5 marks each. The answers to this question should not exceed 150 words.
  6. Question numbers 22-27 are of 6 marks each. The answers to this question should not exceed 150 words.

Question.1. What was New International Economic Order ?
Answer.

  1. The New International Economic Order (NIEO) stands for attaining economic development and political independence for the least developed countries.
  2. The idea of NIEO originated from the sustainable economic development of the least need for developed countries of NAM.

Question.2. Mention the full form of IMF. How many member States does it have ?
Answer. The full form of IMF is the International Monetary Fund. It is an international organisation which commands or subordinates all those financial institutions and legislation that act at the international level. Presently it has 184 members.

Question.3. Differentiate between ‘regionalism’ and ‘separatism’.
Answer. The term Regionalism implies expression of regional aspirations and regional identity like North Eastern states. On the other hand separatism signifies separatist politics and state autonomy. For example —separatists who want a separate Kashmiri nation.

Question.4. Correct the following statement and rewrite:
Answer. Goa was not merged with Maharashtra as the central government did not want it. Ans. Goa was not merged with Maharashtra because it was under the possession of the Portuguese and was treated as the acquired territory of India.

Question.5. What is meant by Chipko Movement ?
Answer. The Chipko movement was a protest against the commercial logging that the government had permitted. In this movement villagers, both men and women, used a novel tactic for their protest by hugging tjie trees to prevent them from being cut down.

Question.6. Why is there need for an International Organisation ?
Answer. The need for an International Organisation is very much justified with the quotation of the former UN secretary general, Dag tfammarskjold that “the international Organisation (UN) was not created to take humanity to heaven but to save it from hell.”
International Organisation, thus plays a very significant role in the world full of antagonism, differences and conflicts.
(a) International Organisation provides a “common platform” where the countries can discuss contentious issues and find peaceful solutions.
(b) In the era of Globalisation and mutual cooperation the “International Organisation” provides mechanisms, rules and a bureaucracy to help the nations about howto cooperate and have more confidence on global issues like global warming, eradication of some diseases, etc.
(c) “International Organisations” are not an answer to everything but they are important. “International Organisation” helps in matters of war and peace.

Question.7. What is meant by geo-politics ?
Answer. Resource Geopolitics is the geographical political affairs concerned with the allocation and distribution of natural resources among the nation-states of Global arena.

  1. In another way resource geopolitics is all about who gets what, when, where and how.
  2. Here, they have also been the focus of inter-state rivalry and western geopolitical thinking about resources which has been dominated by the relationship of trade, war and power, at the core of which were overseas resources and maritime navigation.
  3. For instance critical importance of ensuring uninterrupted supply of strategic resources, in particular oil was well established both during the First World War and the Second World War.

Question.8. How has globalisation involved greater trade in commodities across the globe ?
Answer. Globlalisation has involved greater trade in commodities across the globe in the following ways:
(i) The restrictions imposed by different countries on allowing the imports of other countries have been reduced.
(ii) Secondly, the restrictions on movement of capital across countries have also been
reduced.

Question.9. Mention any two examples of international issues where India took an independent stand.
Answer. India took an independent stand on :

  1. The issue of joining power blocs: After independence, India wanted to formulate an independent foreign policy and keep aloof from power politics of super powers. Thus, India adopted Non-alignment policy to protect national identity and sovereignty as India did not want to enter into military alliances in the cold war politics.
  2. International treaties like NPT : India is opposed to the international treaties aimed at Non-proliferation since they are discriminatory in nature and selectively applicable to the non-nuclear powers, they legitimise the monopoly of the five nuclear weapon powers.

Question.10. Why did India decide to go Nuclear ?
Answer. India is opposed to the international treaties aimed at non-proliferation since they are selectively applicable to the non-nuclear powers. India conducted a series of nuclear tests demonstrating its capacity to use nuclear energy for military purposes.
India’s nuclear doctrine of credible minimum nuclear deterrence professes “no first use” and reiterates its commitment to global, verifiable and non-discriminatory nuclear disarmament leading to a weapons-free world.

Question.11. Describe any four consequences of the disintegration of Soviet Union.
Answer. The disintegration of the Soviet Union as the second world power and the collapse of socialist systems in Eastern Europe had profound consequences for world politics in general and Asian countries, like India, in particular. The consequences of the disintegration can be analysed as follows:

  1. The disintegration of Soviet Union meant the end of the cold war confrontation. The end of cold war signifies collapse of the ideological conflict between the socialists and the capitalists. Now there is no alliance system based on ideology. As the alliance system led to the formation of military blocs, so the end of confrontation demanded the end of the arms race and restoration of peace.
  2. With the disintegration of USSR, the end of the cold war left open only two possibilities like either the remaining superpower would dominate and create a “unipolar system” or different countries or groups of countries could become important players in the international system, thereby bringing in a ‘multipolar system’, where no one power could dominate.
  3. As it turned out, the US became the sole super power. Backed by the power and prestige of the US, the ‘capitalist economy’ was now the dominant economic system internationally.
  4. Institutions like the World Bank and International Monetary Fund became powerful advisors to all these countries since they gave them loans for their transition to capitalism. Politically, the notion of liberal democracy emerged as the best way to organise political life.

Question.12. Do you justify the attack on Iraq in the name of ‘attack on terrorism’ ? Support your answer with any two arguments.
Answer. The US attack on Iraq was not at all justified in the wake of terrorist attack on the WTC on September 11, 2001 because the main objective of the attack was to control Iraqi oil fields and to install a regime friendly to the US.
Operation Iraqi Freedom was the code name given by US when it launched the invasion of Iraq on 19th March, 2003. More than forty countries joined in the US-led Coalition after the UN refused to give its mandate to the invasion.
Aims and Objectives

  1. The purpose of the invasion was to prevent Iraq from developing weapons of mass . destruction [WMD].
  2. But this was an eyewash, because no evidence of WMD has been unearthed in Iraq, so it is being speculated all over the world that the invasion was motivated by other objectives such as controlling Iraqi oil fields and installing a regime friendly to the US.

Outcome.
The outcome of Iraqi invasion was very complex and contradictory.

  1. Although the government of Saddam Hussien fell swiffly, the US has not been able to
    pacify Iraq.
  2. A full-fledged insurgency against US occupation was ignited in Iraq.
  3. Iraqi casualties are very much higher than US. The US has lost over 3,000 military personnel in the war.
  4. It is conservatively estimated that 50,000 Iraqi civilians have been killed since the US led invasion.
    On the whole it is now widely recognised that the US invasion of Iraq was, in some crucial respects, both a military and political failure. (Any two)

Question.13. How can the Euro pose a danger to the US Dollar ?
Answer. European Union was established in 1992 for a common foreign and security policy, cooperation on justice and the creation of a single currency, i.e., Euro.
But European Union’s currency Euro can pose a threat to the dominance of the US dollar because of following reasons :

  1. Euro is the standard currency of European Union and its share of world trade is three times larger than that of the United States.
  2. European Union’s economic power influences its closest neighbours as well as Asia and Africa. Euro is also getting importance over dollar.
  3. As the European Union functions as an important bloc in economic organisation, such as
    World Trade Organisation (WTO), so the Euro also has an important place in WTO.
    (iv) Euro poses a threat to US doflar because EU is the world’s biggest economy with a GDP of more than $12 billion in 2005, slightly larger than that of the United States

Question.14. Explain any two political and economic consequences each of globalization.
Answer. Economic Consequences of Globalisation :

  1. Globalisation has involved greater trade in commodities across the globe.
  2. The restrictions imposed by other countries on allowing the imports have been reduced.

Political Consequences:

  1. In place of welfare state, it is the market that becomes the prime determinant of economic and social priorities.
  2. The entry and increased role of MNCs all over the world leads to a reduction in the capacity of governments to take decisions on their own. (one point each)

Question.15. What is meant by SAARC ? How can peace and co-operation be enhanced through it ?
Answer. SAARC stands for South-Asian Association for Regional Co-operation. It is a major regional initiative by the South Asian states to evolve co-operation through multilateral means.
Its main aim is to have a common programme for regional development.
SAARC members signed an agreement in 2001 called SAFTA (South Asian Free Trade) to promote trade. According to this agreement a free Trade Zone is to formed for the whole South Asia.
It also aims to reduce or lower the Trade Tariffs by 20 per cent in 2007.
India believes that there are real economic benefits for all members from SAFTA because free trade will help members to cooperate better on political issues.

Question.16. Describe how the princely states of Manipur and Junagadh acceded to India.
Answer. As part of integration of Princely States, a few days before independence the Maharaja of Manipur Bodhachandra Singh signed the Instrument of Accession with the Indian Government on the assurance that internal autonomy of Manipur would be maintained.

  1. As a follow up the Maharaja held elections in Manipur in June 1948 and the state became a ‘Constitutional Monarchy’. Thus Manipur became the first state to hold an election based on Universal Adult Franchise.
  2. But in the Legislative Assembly there were sharp differences over the question of merger of Manipur with India. While the state Congress wanted the merger, other political parties were opposed to this.

Question.17. Read the following passage and answer the question below.
“In the history of nation-building only the soviet experiment bears comparison with the Indian. There too, a sense of unity had to be forged between diverse ethnic groups religious, linguistic communities and social classes. The scale— geographic as well as demographic— was comparably massive. The raw material the state had to work with was equally unpropitious : a people divided by faith and driven by debt and disease.”
                                                                                                                                         — Rama Chandra Guha
(a) List the commonalities that the author mentions between India and Soviet Union and give one example for each of these from India.
(b) The author does not talk about dissimilarities between the two experiments. Can you mention two dissimilarities ?
(c) In retrospect which of these two experiments worked better and why?
Answer.
(a) The above two statements of M.K. Gandhi and Jawaharlal Nehru reveal or suggest the agenda such as:

  • “Enlightened and secular democracy” to accommodate social diversity and to promote free and fair electoral politics.
  • Economic and technological development of India to promote welfare motive.
  • Both nations shaped their nations on the linguistic basis.
  • Both India and Soviet Union shared same geographical and demographical reasons for the division of states.

(b) The two dissimilarities which the authors do not mention are Soviet Union divided into 15 independent countries by compromising its unity and integrity which is not in India’s case.
(c) The “Indian experiment worked better because it promoted “linguistic and cultural plurality” of the country without affecting the “unity of the nation”.

Question.18. “Indian policy makers made a mistake by emphasising the role of state in the economy. India could have developed much better if private sector was allowed a free play right from the beginning”. Give arguments for or against this proposition.
Answer. No, the above mentioned statement is not fully true because the role of state in the Indian economy was very much required to regulate our economy immediately after independence. Later on, when our economy got stabilised and regulated the Indian policy makers introduced New Economic Policy in 1991 to liberalise our economy.
In the beginning India did not follow any of the two known paths. It did not accept the capitalist model of development in which development was left entirely to the private sector, nor did it follow the socialist model in which private property was abolished and all the production was controlled by the state.
But the elements from both these models were taken together in India /.e. “Mixed Economy”. This concept of Mixed Economy was open to criticism both from the left and the right.

  1. Critics argued that planners refused to provide the private sector with enough space and stimulus to grow
  2. According to them them the enlarged public sector created enough hurdles for private capital, in the way of installing systems of licenses and permits for investment.
  3. The state controlled more things than were necessary and this led to inefficiency and corruption.

Arguments in favour of state control

  1. On the other hand, there were Critics. who thought that the state did not do enough. It intervened only in those areas where the private sector was not prepared to go. Thus the state helped the private sector to make profit.
  2. Also, instead of helping the poor, the state intervention ended up creating a new middle class that enjoyed the privileges of high salaries without much accountablity.
    Thus, we can safely say the role of state in Indian Economy was beneficial in the early years both for the public as well as the private sector. And this led India towards development.

Question.19. Read the passage and answer the questions that follow :
…nearly all ‘new social movements’ have emerged as corrective to new maladies
— environmental degradation, violation of the status of women, destruction of
tribal cultures and the undermining of human rights — none of which are in and by themselves transformative of the social order. They are that way quite different from revolutionary ideologies of thfc past. But their weakness lies in their being so heavily
fragmented — a large part of the space occupied by the new social movement
seem to be suffering from …… various characteristics which have prevented them from being relevant to the truly oppressed and the poor in the form of a solid unified movement of the people. They are too fragmented, reactive, ad hocish, providing no comprehensive framework of basic social change. Their being anti-this or that (anti-West, anti-capitalist, anti-development, etc.) does not make them any more coherent, any more relevant to oppressed and peripheralized communities. — Rajni Kothari
(a) What is the difference between new social movements and revolutionary ideologies ?
(b) What according to the author are the limitations of social movements ?
Answer.
(a) The main difference between new social movements and revolutionary ideologies is that like revolutionary ideologies, none of new social movements are in and by themselves “transformative of the social order” but they emerged as corrective of new maladies.
(b) According to the author the social movements are fragmented, reactive, ad hocish and not relevant to oppressed communities

  • They never provide a comprehensive framework of basic social change.

Question.20. Identify five states which became separate states by 1986.
solved-cbse-sample-papers-for-class-12-political-science-set-11-1
Answer.
(i) Arunachal Pradesh (ii) Mizoram (iii) Manipur
(iv) Tripura (v) Nagaland

Question.21. Look at the picture given below and answer the following questions.
solved-cbse-sample-papers-for-class-12-political-science-set-11-2

  1. The above picture is related to which country ?
  2. What does the picture indicate ? Support your answer with two examples.

Answer.

  1. The Great Wall and Dragon are two symbol most commonly associated with China.
  2. This cartoon depicts China’s economic rise. China followed its own path in introducing a market economy. The Chinese economy did not go for ‘Shock Therapy’ but opened their economy step by step as under:
  • The privatisation of agriculture in 1982 was followed by the privatisation of industry in 1998.
  • Trade barriers were eliminated only in Special Economic Zone where foreign investors could set up enterprises.
  • China’s accession to WTO in 2001 has been further step in its opening to the outside world.

Question.22. Analyse the different ways in which American hegemony could be overcome.
Or
How far was the US attack on Iraq justified in the wake of terrorist attack on the World Trade Center on September 11, 2001 ?
Answer. As we are living in a global village so we all are the neighbours of the village headman. If the behaviour of the headman becomes intolerable and we do not have the option of leaving the global village, resistance will then be the only option available. The same is applicable to US Hegemony.
Now, the question arises who and how. There is no world government. There are some rules and norms called the laws of war that restrict but do not prohibit war.

  1. We must recognise that no single power, anywhere in the world is near balancing the US militarily.
    Therefore, we can discuss some strategies developed by the political analysts.
  2. ‘Bandwagon’ strategy : some analysts argue that it is strategically more prudent to take advantage of the opportunities that hegemony creates like :
    (a) raising economic growth rate requires increased trade,
    (b) Transfer of technology and investment. Thus, it is suggested by the analysts that
    instead of engaging in activities opposed to the hegemon power, it may be advisable to extract benefits by operating within the hegemonic system.
  3. Hide strategy: This strategy implies staying as far removed from the dominant power as possible. For example China, Russia, the European Union-all of them in different ways – are seeking to stay below the radar, as it were, and not overly and unduly antagonise the US.
    But it is also not a feasible strategy. While it may be an attractive, viable policy for small states, it is hard to imagine mega-states like China, India and Russia or a huge European Union being able to hide for any substantial length of time.
  4. Non-state and Third estate strategy : The given proposition is to a large extent unrealistic from the theoretical point of view but very realistic from the practical point of view because non-state actors would challenge the US hegemony in a very active way.
    These challenges to American hegemony will emerge in the economic and cultural realms and will come from a combination of Non-governmental organisations [NGOs], social movements, and public opinion.
    The challenge may arise from sections of the media and intellectuals, artists and writers. These various actors may well form links across national boundaries, including Americans, to criticise and resist US policies.

Or
The US attack on Iraq was not at all justified in the wake of terrorist attack on the WTC on September 11, 2001 because the main objective of the attack was to control Iraqi oil fields and to install a regime friendly to the US.
Operation Iraqi Freedom was the code name given by US when it launched invasion of Iraq on 19th March, 2003. More than forty countries joined in the US-led Coalition after the UN refused to give its mandate to the invasion.
One of the reasons for the invasion was to prevent Iraq from developing weapons of Mass Destruction [WMD]. But this was an eyewash, because no evidence of WMD has been unearthed in Iraq, so it is being speculated all over the world that the invasion was motivated by other objectives such as, controlling Iraqi oil fields and installing a regime friendly to the US.
The outcomes of Iraqi invasion also reflected that US attack was just to show its hegemonic aspirations nothing else.
Although the government of Saddam Hussien fell swiftly, the US has not been able to pacify Iraq.
A full-fledged insurgency against US occupation was ignited in Iraq.
Iraqi casualties are very much higher than US. The US has lost over 3,000 military personnel in the war.
It is conservatively estimated that 50,000 Iraqi civilians have been killed since the US led invasion.
On the whole it is now widely recognised that the US invasion of Iraq was, in some crucial respects, both a military and political failure.

Question.23. Examine India’s changing relationship with post-communist Russia.
Or
What led to the emergence of bi-polar world ? What were the arenas of cold war between the two blocs ?
Answer. Indo-Russian relations are embedded in a history of trust and common interests and are matched by popular perceptions.
Common view on the multipolar world order: Russia and India share a vision of multipolar world order. For both these countries a multipolar world order is the co-existence of several powers in the international system, collective security, greater regionalism, negotiated
settlement of international conflicts, an independent foreign policy for all countries and decision making through bodies like the UN that should be strengthened, democratised and empowered.
India’s stand towards Russia : India gets meaningful benefits for having healthy relations with Russia on the issues like Kashmir, energy supplies, sharing information on international terrorism, access to central Asia, and balancing its relationship with China.
Russia’s stand towards India : Like India, Russia stands to benefit from this relationship because India is the’second largest arms market for Russia.
Besides, Indian military gets most of its hardware from Russia. Since India is an oil importing nation, so Russia is important to India and has repeatedly come to its assistance during its oil crisis.
In order to meet the demands of energy India is trying to increase it energy imports from Russia and the republics of Kazakihstan and Turkmenistan. This also broadened the scope for partnership and investment in oilfields.
India has also strengthened its relations with Russia for her nuclear energy plans and space industry. India gets the cryogenic rocket from Russia whenever needed.
Thus, we can safely conclude that India has maintained good relations with all the post-communist countries. But the strongest relations are still those between Russia and India.
Or
The emergence of two power blocs can be traced back to some major developments in the Cold War between the two superpowers like
solved-cbse-sample-papers-for-class-12-political-science-set-11-3

  1. Expansionist policies of the superpowers: The two superpowers were keen on expanding their sphere of influence in different parts of the world. In a world sharply divided between two alliance systems or blocs, a state was supposed to remain tied to its protective superpower to limit the influence of the other superpower and its allies.
    The smaller states in alliances used the link to the superpowers for their own purposes.
  2. Military Alliances : During the Cold War era Europe, thus, became the main arena of conflict between the superpowers.
  3. Ideological Conflicts : Power rivalries, military alliances and the balance of power between the superpowers led to the division of Europe into two power blocs i.e., US Power bloc and USSR power bloc.

The Arenas of the Cold War refers to the areas where crises and war occurred or threatened to occur between the alliance systems but did not cross certain limits.

  1. We begin with the Cuban missile crises which was only one of the several crises that occurred during the Cold War but fortunately both sides, US and USSR, decided to avoid war.
  2. The Cold War also led to several shooting wars between the two superpowers which were poised for direct confrontation in
  3. Korea (1950-53)
  4. Berlin crisis (1958-62)
  5. The Congo Crisis (1960s)

Crises deepened as neither of the parties involved was willing to back down, but it is important to note that these crises and wars did not lead to another World War.

  1. A great many lives were lost in some of these arenas like Korea, Vietnam and Afghanistan, but the world was spared a nuclear war and global hostilities.
  2. In some cases, huge military build-ups were reported. In many cases, diplomatic communication between the superpowers could not be sustained and contibuted to the misunderstandings.
  3. Sometimes, countries outside the two blocs, for example the non-aligned countries, played a role in reducing Cold War conflicts and averting some grave crises.
    In this way, as the Cold War rolled from one arena to another, the logic of restraint was increasingly evident.

Question.24. “Indian policy makers made a mistake by emphasising the role of state in the economy. India could have developed much better if private sector was allowed a free play right from the beginning”. Give arguments for or against this proposition.
Or
Discuss the principles and difficulties involved in the process of partition.
Answer. The above mentioned statement is partly true. The role of state in the Indian economy was very much required to regulate our economy immediately after independence. Later on, when our economy got stabilised and regulated the Indian policy makers introduced New Economic Policy in 1991 to liberalise our economy.
In the beginning India did not follow any of the two known paths. It did not accept the capitalist model of development in which development was left entirely to the private sector, nor did it follow the socialist model in which private property was abolished and all the. production was controlled by the state.
But the elements from both these models were taken together in India i.e. “Mixed Economy”. This concept of Mixed Economy was open to criticism both from the left and the right.

  1. Critics argued that planners refused to provide the private sector with enough space
    and stimulus to grow.
  2. According to them the enlarged public sector created enough hurdles for private capital, by the way of installing systems of licenses and permits for investment.
  3. The state controlled more things than were necessary and this led to inefficiency and corruption.

Arguments in favour of state control

  1. On the other hand, there were critics who thought that the state did not do enough. It intervened only in those areas where the private sector was not prepared to go. Thus the state helped the private sector to make profit.
  2. Also, instead of helping the poor, the state intervention ended up creating a new middle class that enjoyed the privileges of high salaries without much accountability.
    Thus, we can safely say the role of state in Indian Economy was beneficial in the early years both for the public as well as the private sector. And this led India towards development.

Or
In the Indian context the word partition signifies the division of British India into India and Pakistan. The process of partition started in 1940 — when the Muslim League propounded the “Two-Nation Theory”.
According to this theory India consisted of not one but two “people”, Hindus and Muslims. Several political developments in 1940s, the political competition between the Congress and the Muslim League and the British role led to the decision for the creation of Pakistan.
Thus it was decidedthat India would be divided into two countries Inoia and Pakistan. Principle of religious majorities was followed for the division, i.e., the areas where the Muslims were in majority would make up the territory of Pakistan and the rest was to stay with India. But this principle of partition presented all kinds of difficulties such as :
Problems of East and West

  1. There was no single belt of Muslim majority areas in British India. There were two areas of concentration, one in the west and one in the east.
  2. Merger of NWFP: On the partition move not all Muslim majority areas wanted to be in Pakistan. Even Khan Abdul Ghaffar Khan. The undisputed leader of the North Western Frontier Province, was staunchly opposed to the two nation theory. But ultimately the NWFP was made to merge with Pakistan.
  3. Difficulties related to provinces of Punjab and Bengal:
    Punjab and Bengal were the two muslim majority provinces of British India. These provinces had very large areas where the non-muslims were in majority. Thus, it was decided that these two provinces would be bifurcated according to the religious majority. In this way, on 14-15 August 1947, not one but two nation-states came into existence, i.e. India and Pakistan.

Question.25. Describe the emergence of the coalition government in India.
Or
“The opposition to Emergency could keep the Janata Party together only for a while.” Evaluate.
Answer. Elections in 1989 led to the era of coalitions with the defeat of Congress party. What happened after 1989 was the emergence of several parties in such a way that no single party could get absolute majority in the Lok Sabha.
in this era, regional parties played a crucial role in forming the ruling alliance. Thus, with the election of 1989 a long phase of coalition politics began in India. Since then there have been nine governments at the centre all of which have either been coalition governments or minority governments supported by other parties which did not join the government. In this new phase any government could be formed only with the participation or support of many regional parties.
Or
Actually the opposition to emergency could keep the ‘Janata Party’ together only for a while. Janata Party lacked direction, leadership and a common programme, which led to mid-term elections in 1980. The 1977 elections turned into a referendum on the experience of the emergency. For the first time since independence the Congress Party was defeated and opposition came into power at the centre.

  1. The Janata Party made this election a referendum on the emergency. Its campaign was focused on the non-democratic character of the rule and on the various excesses that took place during this period.
  2. In the backdrop of arrests of thousands of persons and the censorship of the press, the public opinion was against the Congress.
  3. Besides, the formation of the Janata Party also ensured that non-Congress votes would not be divided.
  4. But, from the very beginning there was stiff competition among leaders for the post of Prime Minister which ultimately led to the mid-term poll of 1980.
  5. Janata Party government could .tot bring about a fundamental change in policies from those pursued by the Congress.
  6. Not only this there was split in the Janata Party and the government which was led by Morarji Desai lost its majority in less than 18 months.
  7. Hence, fresh Lok Sabha elections were held in January 1980 in which Janata Party suffered
    a comprehensive defeat. Congress Party led by Indira Gandhi came back to power by winning 353 seats. (Any six)

Question.26. ‘Coalition government is a bane or a boon for democracy in India.’ Explain any three arguments in support of your answer.
Or
‘All regional movements need not lead to the separatist demands.’ Explain the statement by giving suitable examples.
Answer. It is correct to say that coalition governments in India have helped in arriving at some consensus, like:
In the midst of severe competition and many conflicts, a consensus appears to have emerged amdng most parties. This consensus consists of four elements.

  1. Agreement on new economic policies : While many groups are opposed to the new economic policies, most parties believe that these policies would lead the country to prosperity and a status of economic power in the world.
  2. Acceptance of the political and social claims of the backward castes : Political parties have recognised that the social and political claims of the backward castes need to be accepted. As a result, all political parties now support reservation of seats for the backward classes in education and employment. Political parties are also willing to ensure that the OBCs get adequate share of power.
  3. Acceptance of the role of state level parties in the governance of the country: The distinction between state level and national level parties is fast becoming less important.
  4. Emphasis on pragmatic considerations rather than ideological positions and political alliances without ideological agreement: Coalition political shifted the focus of political parties from ideological differences to power sharing arrangements thus, most parties of the NDA did not agree with the Hindutva ideology of the BJP, yet, they came together to form a government and remained in power for a full term.
    To sum up, all these are momentous changes and are going to shape politics in the near future.

Or
All regional movements need not lead to separatist demands because regional aspirations are very much a part of democratic politics. Expression of regional issues is not an aberration or an abnormal phenomenon. Even in small countries like the United Kingdom, there are regional aspirations in Scotland, Wales and Northern Ireland.
For all regional movements the best way to respond is through democratic negotiations rather than through suppression.
For instance, look at the situation in the eighties. Militancy had erupted in Punjab. Problems persisted in the North-East; Students in Assam were agitating; Kashmir valley was on the boil. Instead of treating these as simple law and order problems, the Government of India reached negotiated settlements with regional movements. This produced a reconciliation which reduced the tensions-existing in many regions. The example of Mizoram shows how political settlement can resolve the problem of separatism effectively.
All these examples show that the meaningful and justified regional movements need not lead to separatist demands or encouraged to espouse separatism.

Question.27. Explain India’s improving relationship with China.
Or
What were the factors that led to the popularity of Indira Gandhi’s government in the early of 1970 ?
Answer. Our relations with China after independence started off very well due to a number of friendly gestures on the part of India. In 1954, India signed the famous Panchsheel, which started a new era of Sino-Indian friendship.
But after 1957 various “contentious issues” arose in Sino-Indian relations like :

  1. Tibet Problem
  2. Sikkim Issue
  3. Border Issue
  4. Chinese Attack -1962
  5. Chinese assistance to Pakistan
  6. Nuclear Test Issue.

From 1958 to 1975, China saw India as its major geo-strategic rival and wanted to keep India down.

  1. Attempts at normalisation of relations between the two were taken at the Indian initiative
    in 1976. Due to this, Sino-Indian diplomatic relations were restored with the exchange of ambassadors.
  2. JWG (Joint Working Group) was set up by both countries to find a solution to the border dispute.
  3. Both countries also decided to reduce forces along the Sino-Indian frontier and both pledged not to use or threaten the use of force against the other.
  4. The process of mending forces and emphasis on “friendly competition” has started. This could be done through mutual understanding and bilateral agreements. Both countries should come together to fight against the global challenges like terrorism, economic disparity and nuclear arms race.
  5. In the Unipolar World there have been significant changes in India China relations. Their relations now have a strategic as well as an economic dimension. Both countries view
    themselves as rising powers in global politics and both would like to play a major role in the Asian economy and politics.
    While concluding we can say that a long term stable relationship and peace between the two is important for the mutual benefit of both the countries. Increasing transportation and communication links, common economic interests and global concerns should help to establish a more positive and sound relationship between the two most populous countries of the world.
    More recently in 2006 both countries signed an agreement on cooperation when the Chinese President, Hu Jintao visited India.
    Both countries have agreed to cooperate with each other in areas that could otherwise create conflict between the two, such as bidding for energy deals abroad. At the global level, India and China thus, have adopted similar policies in international economic institutions like the World Trade Organisation.

Or
In the early of 1970s, the government of Indira Gandhi gained popularity due to various factors such as:

  1. During this period the government made conscious attempts to project its socialist credentials.
  2. Indira Gandhi vigorously campaigned for implementing the existing land reform laws and undertook further land ceiling legislation.
  3. In order to end her dependence on the other political parties, she strengthened her party’s position in the Parliament and sought a popular mandate for her programmes. Indira Gandhi’s government recommended the dissolution of the Lok Sabha in December 1970.
  4. The crisis in East Pakistan and the Indo-Pak War leading to the establishment of Bangladesh added one more feather to the popularity of Indira Gandhi.
  5. In this way, Indira Gandhi and her government was seen not only as the protector of the poor and the underprivileged but also as a strong government.
  6. The Congress was now in power in almost all the states and restored its dominance. It was also popular across different social sections.

CBSE Sample Papers for Class 12 Political Science Delhi – 2014

CBSE Sample Papers for Class 12 Political Science Delhi – 2014

Time Allowed: 3 Hours                                                                                          Maximum Marks: 100
General Instructions:

  1. All questions are compulsory.
  2. Question Numbers 1 to 10 are of one mark each. The answers to these questions should not exceed 20 words each.
  3.  Question Numbers 11 to 20 are of two marks each. The answers to these questions should not exceed 40 words each.
  4. Question Numbers 21 to 30 are of four marks each. The answers to these questions should not exceed 100 words each.
  5. Question Numbers 31 to 35 are of six marks each. The answers to these questions should not exceed 150 words each.
  6. Question Number 35 is based on the map. Write the answer in your Answer-Book.

SET -I

Question.1.Which incident was related to 9/11 ? 1
Question.2. Under which plan did USA extend financial support for reviving Europe’s economy after the Second World War ? 1
Question.3. Who is the present Secretary-General of the United Nations ? 1
Question.4. Define Security. 1
Question.5. Name the leader who played a historic role in negotiating with the rulers of princely states to join the Indian Union. 1
Question.6. Which political party laid emphasis on the idea of one country, one culture and one nation ? 1
Question.7. In which year was the fifth general election to Lok Sabha held ? 1
Question.8. Mention the main reason for the defeat “of Congress Party in the elections of 1977. 1
Question.9. Who represented Congress (O) and Congress (R) after the split of the Congress Party ? 1
Question.10. Which student’s group led the anti-foreigner movement in Assam ? 1
Question.11. What is meant by the Cold War ? 2
Question.12. State any two features of the Soviet System. 2×1=2
Question.13. Which two differences between India and China led to an army conflict in 1962 ? 2×1=2
Question.14. List any four principal organs of the United Nations. ‘ 4x%=2
Question.15. Suggest any one effective step which would limit war or violence between countries. 2
Question.16. Differentiate between the main objectives of the First and the Second Five Year Plans. 2
Question.17. Highlight any two features of the ideology of Bharatiya Jana Sangh. 2×1=2
Question.18. What is meant by Non-alignment ? 2
Question.19. What was the main demand of Chipko Movement ? 2
Question.20. Who was the chairperson of Mandat Commission ? State any one recommendation made by him/her. 1+1=2
Question.21. Describe any four consequences of Shock Therapy. 4×1=4
Question.22. Study the cartoon given below carefully and answer the questions that follow : l+l+2=4
cbse-sample-papers-for-class-12-political-science-delhi-2014-1
(a) The mighty soldier with weapons represents which country ?
(b) Why do names of various countries appear on his uniform ?
(c) What important message does the cartoon convey to the World ?
Question.23. What is meant by the ASEAN way ? Mention any two of its objectives. 2+2=4
Question.24. Explain any two points of conflict between India and Bangladesh. 2+2=4
Question.25. As decided by the member States in 2005, highlight any four steps to make the United Nations more relevant in the changing context. 4×1=4
Question.26. Mention any four political consequences of globalization. 4×1=4
Question.27. List any two merits and two demerits of the Green Revolution. 2+2=4
Question.28. Suppose you are looking after the foreign policy of India. Which four values will you like to integrate into the foreign policy ? 4×1=4
Question.29. Explain any two reasons for the popularity of Indira Gandhi during 1971 election. 2+2=4
Question.30. In the given political outline map of India, four places have been marked A , B , C and D.
Identify them with the help of the information given below and write their correct names in your answer book, along with their.serial numbers and the alphabet concerned. 4×1= 4
cbse-sample-papers-for-class-12-political-science-delhi-2014-2
(i)The state associated with Narmada Bachao andolan?
(ii) The State which merged with the Indian Union in 1975.
(iii) The State related to ‘Operation Blue Star’.
(iv) The State whose one of the important leader was Lai Denga.
Question.31. What was Cuban Missile Crises ? Describe its main events.6
Or
Explain any three reasons for the disintegration of the USSR.
Question.32. Explain the concept of ‘common but differentiated responsibilities’. How and where was it emphasized upon ? 3×2=6
Or
Explain any three benefits of globalization with examples.
Question.33. What forced the Union Government of India to appoint the States Reorganisation Commission in 1953 ? Mention its two main recommendations. Name any four new States formed after 1956. 2+2+2=6
Or
Describe the various steps taken to hold the first general elections in India. How far these elections were successful ? 4+2=6
Question.34. Examine the three main reasons responsible for the split in the Congress Party during 1969.
Evaluate any three consequences of the emergency imposed in 1975. 3×2=6
Question.35. Read the passage carefully given below and answer the questions that follow : l+2+3=6
The Assam Movement from 1979 to 1985 is the best example of such movements against ‘outsiders’. The Assame suspected that there were huge numbers of illegal Bengali Muslim settlers from Bangladesh. They felt that unless these foreign nationals are detected and deported, they would reduce the indigenous Assamese into a minority. There were other economic issues too. There was widespread poverty and unemployment in Assam despite the existence of natural resources like oil, tea and coal. It was felt that these were drained out of the State without any commensurate benefit to the people. 2+2+2=6
(i) Name the group that led the movement against outsiders in 1979.
(ii) Why did the Assamese seek the detection and deportation of the outsiders ?
(iii) What were the economic issues taken up as part of the movement ?
Or
This new challenge came to the force in the 1980s, as the Janata experiment came to an end and there was some political stability at the centre. This decade will be remembered for some major conflicts and accords in the various regions of the country, especially in Assam, the Punjab, Mizoram and the developments in Jammu and Kashmir.
(i) Explain the meaning of the phrase ‘Janata experiment came to an end.’
(ii) ‘There was some stability at the centre’. What does it imply ?
(iii) Highlight any two developments in Punjab in 1980s.

SET-II

Question.2. Name any two member states of the European Union who are permanent members of the U.N. Security Council. 1
Question.4. What is the main objective of he United Nations ? 1
Question.6. Name the leader of the freedom movement of India who was popularly known as Frontier Gandhi. 1
Question.8. Who was the official Congress candidate for the post of President of India in 1969 ? 1
Question.11. List any four member countries of NATO. 2
Question.13. State any two features of the European Union that make it an influential organisation. 2×1=2
Question.17. Explain the major difference of ideology between that of the Congress and the Jana Sangh. 2
Question.19. Highlight any two main demands of the anti-arrack movement. 2×1=2
Question.23. Explain any two strategies to overcome hegemony. 2×2=4
Question.26. Explain any two ecpnomic consequences of globalization. 2×2=4
Question.29. Explain the reasons for the students movement of 1974 in Bihar and the role played by Jai Prakash Narain in this movement. 2+2=4
Question.33. Explain any three challenges faced by India at the time of its independence. 3×2=6
Or
“For a long time, Congress Party had been a social and ideological coalition.” Justify the statement.

SET – III

Question.2.Name any two founder member-states of ASEAN. 1
Question.4.Mention any two agencies of the United Nations. 1
Question.6.Which political party of fndia had leaders like A.K. Gopalan, E.M.S. Namboordiripad and S.A. Dange ? 1
Question.8.What is meant by the term ‘Congress Syndicate’ ? 1
Question.11.What was the Cuban Missile Crisis ? 2
Question.13.State any two reasons for the instability of democracy in Pakistan. 2×1=2
Question.17.What is meant by the two nation theory ? 2
Question.19.Mention any two demands of Bhartiya Kisan Union. 2×1=2
Question.23.Explain any two reasons for the popular struggle in East Pakistan (now Bangladesh) against West Pakistan during 1971. 2×2=4
Question.26.What is meant by Common Property Resources ? Explain with examples. 2+2=4

CBSE Previous Year Solved Papers Class 12 Computer Science Outside Delhi 2011

CBSE Previous Year Solved  Papers  Class 12 Computer Science Outside Delhi 2011

Time allowed : 3 hours                                                                                           Maximum Marks: 70

General Instructions :

  1.  There are a total of 26 questions and five sections in the question paper, All questions are compulsory.
  2. Section A contains question number 1 to 5, Very Short Answer type questions of one mark each.
  3.  Section B contains question number 6 to 10, Short Answer type I questions of two marks each.
  4.  Section C contains question number 11 to 22, Short Answer type II questions of three marks each.
  5.  Section D contains question number 23, Value Based Question of four marks.
  6. Section E contains question number 24 to 26, Long Answer type questions of five marks each.
  7. There is no overall choice in the question paper, however, an internal choice is provided in one question of two marks, one question of three marks and all three questions of five marks. An examined is to attempt any one of the questions out of two given in the question paper with the same question number.

Question.1. (a) What is the difference – between Type Casting and Automatic Type Conversion ? Also, give suitable C++ code to illustrate both.
Answer:
Type Casting : It is an user defined explicit type conversion which operand in an expression is forced to be a specific data type.
SYNTAX:-
(type) expression
Automatic Type Conversion : In case of an expression in which operands of different type are mixed, an implicit type conversion is performed by the computer which automatically converts all operand upto the type of largest operand.
(b) Write the names of the header files, whichis/are essentially required to run/execute the following C++ code:
cbse-previous-year-solved-papers-class-12-computer-science-outside-delhi-2011-1
Answer: #include<iostream.h> and #include<ctype.h>
(c) Rewrite the following program after removing the syntactical errors (if any). Underline each correction.
cbse-previous-year-solved-papers-class-12-computer-science-outside-delhi-2011-2
cbse-previous-year-solved-papers-class-12-computer-science-outside-delhi-2011-3
(d) Find the output of the following program :
cbse-previous-year-solved-papers-class-12-computer-science-outside-delhi-2011-4
cbse-previous-year-solved-papers-class-12-computer-science-outside-delhi-2011-5
Answer:
cbse-previous-year-solved-papers-class-12-computer-science-outside-delhi-2011-6
(e) Find the output of the following program:
cbse-previous-year-solved-papers-class-12-computer-science-outside-delhi-2011-7
Answer:
cbse-previous-year-solved-papers-class-12-computer-science-outside-delhi-2011-8
(f) Go through the C++ code shown below, and find out the possible output or output from the suggested output options (i) to (iv). Also, write the least value and highest value, which can be assigned to the variable MyNum.
cbse-previous-year-solved-papers-class-12-computer-science-outside-delhi-2011-9
Answer:
cbse-previous-year-solved-papers-class-12-computer-science-outside-delhi-2011-10

Question.2. (a) Differentiate between Constructor and Destructor function with respect to Object Oriented Programming.
Answer:
cbse-previous-year-solved-papers-class-12-computer-science-outside-delhi-2011-11
(b) Write the output of the following C++ code. Also, write the name of feature of Object Oriented Programming used in the following prQgram jointly illustrated by the functions [I]to[IV].
cbse-previous-year-solved-papers-class-12-computer-science-outside-delhi-2011-12
cbse-previous-year-solved-papers-class-12-computer-science-outside-delhi-2011-13
Answer:
cbse-previous-year-solved-papers-class-12-computer-science-outside-delhi-2011-14
(c) Define a class Applicant in C++ with following description:
Private Members

  •  A data member ANo (Admission Number) of type long
  •  A data member Name of type string
  •  A data member Agg (Aggregate Marks) of type float
  • A data member Grade of type char
  •  A member function GradeMe( ) to find the Grade as per the Aggregate Marks obtained by a student. Equivalent Aggregate Marks range and the respective Grades are shown as follows:cbse-previous-year-solved-papers-class-12-computer-science-outside-delhi-2011-15

Public members

  • A function ENTER( ) to allow user to enter values for ANo, Name, Agg & call function GradeMeO to find the Grade.
  •  A function Result( ) to allow user to view the content of all the data members.

Answer:
cbse-previous-year-solved-papers-class-12-computer-science-outside-delhi-2011-16
cbse-previous-year-solved-papers-class-12-computer-science-outside-delhi-2011-17
cbse-previous-year-solved-papers-class-12-computer-science-outside-delhi-2011-18
cbse-previous-year-solved-papers-class-12-computer-science-outside-delhi-2011-19
(d) Answer the question (i) to (iv) based on the following:
cbse-previous-year-solved-papers-class-12-computer-science-outside-delhi-2011-20
cbse-previous-year-solved-papers-class-12-computer-science-outside-delhi-2011-21
cbse-previous-year-solved-papers-class-12-computer-science-outside-delhi-2011-22

  1. Write the names of member functions, which are accessible from objects of class course.
  2. Write the names of all the data members, which is /are accessible from member function commence of class course.
  3. Write the names of all the members, which are accessible from objects of class teacher.
  4.  Which type of inheritance is illustrated in the above C++ code?

Answer:

  1.  Result( ), Student( ), Enroll ( ), Display( ), TCode, TName, Salary
  2. Data member from Students, Private Members from Teacher
  3. TCode, TName, Salary
  4. Multiple Inheritance

Question.3.(a) Write a Get2Froml() function in C++ to transfer the content from one array ALL[ ] to two different arrays Odd[ ] and Even[ ]. The Odd[ ] array shotdd contain the values from odd position (1, 3, 5,….) of ALL[ ] and Even[ ] array should contain the value from even positions (0, 2, 4, ….) ofALL[].
Example:
cbse-previous-year-solved-papers-class-12-computer-science-outside-delhi-2011-23
Answer.
cbse-previous-year-solved-papers-class-12-computer-science-outside-delhi-2011-24
(b) An array G[50] [20] is stored in the memory along the row with each of its element occupying 8 bytes, find out the location of G[10] [15], if G[0] [0] is stored at 4200.
Answer:
cbse-previous-year-solved-papers-class-12-computer-science-outside-delhi-2011-25
(c) Write a function in C++ to perform Delete operation on a dynamically allocated Queue containing Members details as given in the following definition of NODE.
cbse-previous-year-solved-papers-class-12-computer-science-outside-delhi-2011-26
Answer:
cbse-previous-year-solved-papers-class-12-computer-science-outside-delhi-2011-27
(d) Write a DSUM () function in C++ to find sum of Diagonal Elements from NxN matrix.
(Assuming that the N is a odd number)
Answer:
cbse-previous-year-solved-papers-class-12-computer-science-outside-delhi-2011-28
(e) Evaluate the following postfix notation of expression: True, False, NOT, AND, True, True, AND, OR
Answer:
cbse-previous-year-solved-papers-class-12-computer-science-outside-delhi-2011-29

Question.4. (a) Observe the program segment given below carefully and fill the blanks marked as Statement 1 and Statement 2 using seekg( ), seekp( ), tellp( ) and tellg( ) functions for performing the required task.
cbse-previous-year-solved-papers-class-12-computer-science-outside-delhi-2011-30
cbse-previous-year-solved-papers-class-12-computer-science-outside-delhi-2011-31
Answer.
cbse-previous-year-solved-papers-class-12-computer-science-outside-delhi-2011-32
(b) Write a function in C++to count the no. of “He” or “She” words present in a text file “STORY.TXT”.  If the file “STORY.TXT” content is as follows:
He is playing in the ground. She is playing with her dolls.
The output of the function should be
Count of He/She in file :
Answer:
cbse-previous-year-solved-papers-class-12-computer-science-outside-delhi-2011-33
(c)Write a function in C++ to search for a laptop from a binary file “CAMERA.DAT” containing the objects of class CAMERA (as defined below). The user should enter the Model No and the function should search and display the details of the camera.
cbse-previous-year-solved-papers-class-12-computer-science-outside-delhi-2011-34
Answer:
cbse-previous-year-solved-papers-class-12-computer-science-outside-delhi-2011-35

Question.5. (a) What do you understand by Selection and Projection operations in relational algebra ?
Answer : A selection is a unary operation written a σa (R) whereα is a propositional formula that consists of atoms as allowed in the normal selection and the logical operator ∩(and), ∪ (or) and (negation). The selection selects all those tuples in R for which α holds.
A projection a unary operation written as π a1,a2,……a(R) where a1,a2,……an is a set of attribute name. The result of such projection is defined as the set that is obtained when all the tuples in R are restricted to the set. Consider the following tables EMPLOYEE and SALGRADE and answer (b) and (c) parts of this question:
Table: EMPLOYEE
cbse-previous-year-solved-papers-class-12-computer-science-outside-delhi-2011-36
(b) Write SQL commands for the following statements:
(i) To display the details of all EMPLOYEES, descending order ofDOJ.
(ii)To display NAME and DESIG of those EMPLOYEES, whose SGRADE is either S02 or S03.
(iii)To display the content of all the EMPLOYEES table, whose DOJ is in between ‘09-FEB-2006’ and ‘08-AUG-2009’.
(iv)To add a new row with the following:
(v) 109, ‘Harish Roy’, ‘HEAD-LF, ‘S02’. ‘09-SEP-2007, ‘21- APR-1983’
Answer:
cbse-previous-year-solved-papers-class-12-computer-science-outside-delhi-2011-37
(c) Give the output of the following SQL queries :
(i) SELECT COUNT (SGRADE), SGRADE FROM EMPLOYEE GROUP BY SGRADE;
(ii)SELECT MIN (DOB), MAX(DOJ) FROM EMPLOYEE;
(iii)SELECT Name, SALARY FROM EMPLOYEE E, SALGRAGE S WHERE E.SGRADE = S.SGRADE AND E.ECODE<103;
(iv) SELECT SGRADE, SALARY+HRA FROM SALGRADE WHERE SGARDE =‘S02’;
Answer:
cbse-previous-year-solved-papers-class-12-computer-science-outside-delhi-2011-38

Question.6.(a) Verify the following using Truth Table
X+Y.Z= (X+Y). (X+Z)
Answer:
cbse-previous-year-solved-papers-class-12-computer-science-outside-delhi-2011-39
(b) ’Write the equivalent Boolean Expression for the following – Logic Circuit.
cbse-previous-year-solved-papers-class-12-computer-science-outside-delhi-2011-40
Answer:F(P, Q, R)=P.Q+P.R
(c) Write the SOP form of a Boolean function F, which is represented in a truth table as follows:
cbse-previous-year-solved-papers-class-12-computer-science-outside-delhi-201141
Answer :
cbse-previous-year-solved-papers-class-12-computer-science-outside-delhi-2011-42
(d) Reduce the following Boolean Expression using K-Map:  F(A, B, C, D) = ∑ (0, 1, 2, 4, 5, 6, 8, 10)
Answer:
cbse-previous-year-solved-papers-class-12-computer-science-outside-delhi-2011-43

Question.7.(a) In networking what is WAN? How is it different from LAN?
Answer:
A Wide Area Network (WAN) is a network that covers a broad area (i.e., any telecommunications network that links across metropolitan, regional, national or international boundaries) using leased telecommunication lines.
LAN is completely different from WAN. LAN is only limited for connecting systems within the same building, whereas, WAN is extended for connection between countries, cities, etc.
(b) Differentiate between XML and HTML.
Answer:
cbse-previous-year-solved-papers-class-12-computer-science-outside-delhi-2011-44
(c) What is WEB2.0 ?
Answer : WEB2.0 is associated with web applications that facilitates participatory information sharing, interoperability, user centered design and collaboration on the World Web Wide. It allows user to interact and collaborate with each other in a Social Media dialogue as creators of user generates
content in a virtual community.
(d) Out of the following, identify client side script(s) and server side script(s)
(a) javascripy
(b) ASP
(c) vbscript
(d) JSP Answer:
JavaScript & VBScript are client side scripts. JSP & ASP are server side scripts.
(e) Great Studies University is setting up its Academic schools at Sunder Nagar and planning to set up a network. The university has 3 academic schools and one administration center as shown in the diagram below:
cbse-previous-year-solved-papers-class-12-computer-science-outside-delhi-2011-45
Number of Computers in each of the School / Center is follows :
cbse-previous-year-solved-papers-class-12-computer-science-outside-delhi-2011-46

  1.  Suggest the most suitable place (i.e. Schools / Center) to install the server of this university with a suitable reason.
  2.  Suggest an ideal layout for connecting these schools/ center for a wired connectivity.
  3.  Which device you will suggest to be placed/ installed in each of these schools / center to efficiently connect all the computers with in these schools / center.
  4.  The university is planning to connect its admission office in the closest big city, which is more than 350km from the university. WTiich type of network out of LAN, MAN or WAN will be formed? Justify your answer.

Answer:

  1.  Admin Center is suitable to install server because it has the maximum number of computers.
  2. cbse-previous-year-solved-papers-class-12-computer-science-outside-delhi-2011-47
  3.  Switch
  4.  WAN because given distance is more than the range of LAN and MAN.

(f)Compare open source software and proprietary software.
Answer: Open source software refers to a program or software in which the source code (the form of the program when a programmer writes a program in a particular programming language) is available to the general public for use and/or modification from its original design free of charge. Proprietary software is software that is owned by an individual or a company (usually the one that developed it). There are almost always major restrictions on its use, and its source code is almost always kept secret.
(g) What are cookies?
Answer : Cookies are a small piece of data sent from a website and stored in a user’s web browser while the user is browsing that website.

CBSE previous Year Solved Papers Class 12 Maths Outside Delhi 2014

CBSE previous Year Solved  Papers Class 12 Maths Outside Delhi 2014

Time allowed: 3 hours                                                                                          Maximum Marks : 100
General Instructions:

  1. All questions are compulsory.
  2. Please check that this question paper contains 26 questions.
  3. Questions 1-6 in Section A are very short-answer type questions carrying 1 mark each.
  4. Questions 7-19 in Section B are long-answer I type questions carrying 4 marks each.
  5. Questions 20-26 in Section C are long-answer II type questions carrying 6 marks each.
  6. Please write down the serial number of the question before attempting it.

SET I

SECTION – A

Question.1. If R = {(x, y): x + 2y = 8} is a relation on N, write the range of R.
Solution.
cbse-previous-year-solved-papers-class-12-maths-outside-delhi-2014-1

Question.2.
cbse-previous-year-solved-papers-class-12-maths-outside-delhi-2014-2
Solution.
cbse-previous-year-solved-papers-class-12-maths-outside-delhi-2014-3

Question.3. If A is a square matrix such that A2 = A, then write the value of 7A – (I + A)3, where I is an identity matrix.
Solution.
cbse-previous-year-solved-papers-class-12-maths-outside-delhi-2014-4
cbse-previous-year-solved-papers-class-12-maths-outside-delhi-2014-5

Question.4.
cbse-previous-year-solved-papers-class-12-maths-outside-delhi-2014-6
Solution.
cbse-previous-year-solved-papers-class-12-maths-outside-delhi-2014-7

Question.5.
cbse-previous-year-solved-papers-class-12-maths-outside-delhi-2014-8
Solution.
cbse-previous-year-solved-papers-class-12-maths-outside-delhi-2014-9

Question.6.
cbse-previous-year-solved-papers-class-12-maths-outside-delhi-2014-10
Solution.
cbse-previous-year-solved-papers-class-12-maths-outside-delhi-2014-11
cbse-previous-year-solved-papers-class-12-maths-outside-delhi-2014-12

Question.7.
cbse-previous-year-solved-papers-class-12-maths-outside-delhi-2014-13
Solution.
cbse-previous-year-solved-papers-class-12-maths-outside-delhi-2014-14

Question.8.
cbse-previous-year-solved-papers-class-12-maths-outside-delhi-2014-15
Solution.
cbse-previous-year-solved-papers-class-12-maths-outside-delhi-2014-16

Question.9.
cbse-previous-year-solved-papers-class-12-maths-outside-delhi-2014-17
Solution.
cbse-previous-year-solved-papers-class-12-maths-outside-delhi-2014-18

Question.10.
cbse-previous-year-solved-papers-class-12-maths-outside-delhi-2014-19
Solution.
cbse-previous-year-solved-papers-class-12-maths-outside-delhi-2014-20

SECTION – B

Question.11.
cbse-previous-year-solved-papers-class-12-maths-outside-delhi-2014-21
Solution.
cbse-previous-year-solved-papers-class-12-maths-outside-delhi-2014-22

Question.12.
cbse-previous-year-solved-papers-class-12-maths-outside-delhi-2014-23
Solution.
cbse-previous-year-solved-papers-class-12-maths-outside-delhi-2014-24
cbse-previous-year-solved-papers-class-12-maths-outside-delhi-2014-25
OR
cbse-previous-year-solved-papers-class-12-maths-outside-delhi-2014-26
Solution.
cbse-previous-year-solved-papers-class-12-maths-outside-delhi-2014-27

Question.13. Using properties of determinants, prove that:
cbse-previous-year-solved-papers-class-12-maths-outside-delhi-2014-28
Solution.
cbse-previous-year-solved-papers-class-12-maths-outside-delhi-2014-29

Question.14.
cbse-previous-year-solved-papers-class-12-maths-outside-delhi-2014-30
Solution.
cbse-previous-year-solved-papers-class-12-maths-outside-delhi-2014-31

Question.15.
cbse-previous-year-solved-papers-class-12-maths-outside-delhi-2014-32
Solution.
cbse-previous-year-solved-papers-class-12-maths-outside-delhi-2014-33

Question.16. Find the value (s) of x for which y = [x(x – 2)]2 is an increasing function.
Solution.
cbse-previous-year-solved-papers-class-12-maths-outside-delhi-2014-34
cbse-previous-year-solved-papers-class-12-maths-outside-delhi-2014-35
OR
cbse-previous-year-solved-papers-class-12-maths-outside-delhi-2014-36
Solution.
cbse-previous-year-solved-papers-class-12-maths-outside-delhi-2014-37
cbse-previous-year-solved-papers-class-12-maths-outside-delhi-2014-38

Question.17.
cbse-previous-year-solved-papers-class-12-maths-outside-delhi-2014-39
Solution.
cbse-previous-year-solved-papers-class-12-maths-outside-delhi-2014-40
cbse-previous-year-solved-papers-class-12-maths-outside-delhi-2014-41
OR
cbse-previous-year-solved-papers-class-12-maths-outside-delhi-2014-42
Solution.
cbse-previous-year-solved-papers-class-12-maths-outside-delhi-2014-43
cbse-previous-year-solved-papers-class-12-maths-outside-delhi-2014-44
cbse-previous-year-solved-papers-class-12-maths-outside-delhi-2014-45

Question.18. Find the particular solution of the differential equation
dy/dx= 1 + x + y + xy, given that y = 0 when x = 1.
Solution.
cbse-previous-year-solved-papers-class-12-maths-outside-delhi-2014-46

Question.19.
cbse-previous-year-solved-papers-class-12-maths-outside-delhi-2014-47
Solution.
cbse-previous-year-solved-papers-class-12-maths-outside-delhi-2014-48
cbse-previous-year-solved-papers-class-12-maths-outside-delhi-2014-49
cbse-previous-year-solved-papers-class-12-maths-outside-delhi-2014-50

Question.20.
cbse-previous-year-solved-papers-class-12-maths-outside-delhi-2014-51
Solution.
cbse-previous-year-solved-papers-class-12-maths-outside-delhi-2014-52
cbse-previous-year-solved-papers-class-12-maths-outside-delhi-2014-53
OR
cbse-previous-year-solved-papers-class-12-maths-outside-delhi-2014-54
Solution.
cbse-previous-year-solved-papers-class-12-maths-outside-delhi-2014-55
cbse-previous-year-solved-papers-class-12-maths-outside-delhi-2014-56

Question.21.
cbse-previous-year-solved-papers-class-12-maths-outside-delhi-2014-57
Solution.
cbse-previous-year-solved-papers-class-12-maths-outside-delhi-2014-58
cbse-previous-year-solved-papers-class-12-maths-outside-delhi-2014-59

Question.22. An experiment succeeds thrice as often as it fails. Find the probability that in the next five trials, there will be at least 3 successes.
Solution.
cbse-previous-year-solved-papers-class-12-maths-outside-delhi-2014-60

SECTION – C

Question.23. Two schools A and B want to award their selected students on the values of sincerity, truthfulness and helpfulness. The school A wants to award Rs x each, Rs y each and Rs z each for the three respective values to 3,2 and 1 students respectively with a total award money of Rs 1, 600. School B wants to spend Rs 2,3000 to award its 4,1 and 3 students on the respective values (by giving the same award money to the three values as before). If the total amount of award for one prize on each value is Rs 900, using matrices, find the award money f or each value. Apart from these three values, suggest one more value which should be considered for award.
Solution.
cbse-previous-year-solved-papers-class-12-maths-outside-delhi-2014-61
cbse-previous-year-solved-papers-class-12-maths-outside-delhi-2014-62
cbse-previous-year-solved-papers-class-12-maths-outside-delhi-2014-63
Apart from the three values, sincerity, truthfulness and helpfulness, another value for award should be discipline.

Question.24. Show that the altitude of the right circular cone of maximum volume that can be described in a sphere of radius r is 4r/3. Also show that the maximum volume 3 of the cone is 8/27 of the volume of the sphere.
Solution.
cbse-previous-year-solved-papers-class-12-maths-outside-delhi-2014-64
cbse-previous-year-solved-papers-class-12-maths-outside-delhi-2014-65
cbse-previous-year-solved-papers-class-12-maths-outside-delhi-2014-66

Question.25.
cbse-previous-year-solved-papers-class-12-maths-outside-delhi-2014-67
Solution.
cbse-previous-year-solved-papers-class-12-maths-outside-delhi-2014-68
cbse-previous-year-solved-papers-class-12-maths-outside-delhi-2014-69

Question.26. Using integration, find the area of the region bounded by the triangle whose vertices are (- 1, 2), (1,5) and (3,4).
Solution.
cbse-previous-year-solved-papers-class-12-maths-outside-delhi-2014-70
cbse-previous-year-solved-papers-class-12-maths-outside-delhi-2014-71

Question.27. Find the equation of the plane through the line of intersection of the planes x + y + z=1 and 2x+3y+4z=5 which is perpendicular to the plane x – y + z = 0. Also find the distance of the plane obtained above, from the origin.
Solution . Equation of any plane through the line of intersection of the planes.
cbse-previous-year-solved-papers-class-12-maths-outside-delhi-2014-72
OR
cbse-previous-year-solved-papers-class-12-maths-outside-delhi-2014-73
Solution.
cbse-previous-year-solved-papers-class-12-maths-outside-delhi-2014-74

Question.28. A manufacturing company makes two types of teaching aids A and B of Mathematics for class XII. Each type of A requires 9 labour hours of fabricating and 1 labour hour for finishing. Each type of B requires 12 labour hours for fabricating and 3 labour hours for finishing. For fabricating and finishing, the maximum labour hours available per week are 180 and 30 respectively. The company makes a profit of Rs 80 on each piece of type A and Rs 120 on each piece of type B. How many pieces of type A and type B should be manufactured per week to get a maximum profit ? Make it as an LPP and solve graphically. What is the maximum profit per week ?
Solution.
cbse-previous-year-solved-papers-class-12-maths-outside-delhi-2014-75
cbse-previous-year-solved-papers-class-12-maths-outside-delhi-2014-76
cbse-previous-year-solved-papers-class-12-maths-outside-delhi-2014-77

Question.29. There are three coins. One is a two-headed coin (having head on both faces), another is a biased coin that comes up heads 75% of the times and third is also a biased coin that comes up tails 40% of the times. One of the three coins is chosen at random and tossed, and it shows heads. What is the probability that it was the two-headed coin ?
Solution. Let A be the two headed coin, B be the biased coin showing up heads 75% of the times and C be the biased coin showing up tails 40% (i.e., showing up heads 60%) of the times.
cbse-previous-year-solved-papers-class-12-maths-outside-delhi-2014-78
cbse-previous-year-solved-papers-class-12-maths-outside-delhi-2014-79
OR
Two numbers are selected at random (without replacement) from the first six positive integers. Let X denote the larger of the two numbers obtained. Find the probability distribution of the random variable X, and hence find the mean of the distribution.
Solution.
cbse-previous-year-solved-papers-class-12-maths-outside-delhi-2014-80
cbse-previous-year-solved-papers-class-12-maths-outside-delhi-2014-81
cbse-previous-year-solved-papers-class-12-maths-outside-delhi-2014-82

SET II

Note: Except for the following questions, All the remaining question have been asked in previous set.

SECTION – A

Question.9.
cbse-previous-year-solved-papers-class-12-maths-outside-delhi-2014-83
Solution.
cbse-previous-year-solved-papers-class-12-maths-outside-delhi-2014-84

Question.10.
cbse-previous-year-solved-papers-class-12-maths-outside-delhi-2014-85
Solution.
cbse-previous-year-solved-papers-class-12-maths-outside-delhi-2014-86
cbse-previous-year-solved-papers-class-12-maths-outside-delhi-2014-87

SECTION – B

Question.19. Using properties of determinants, prove that
cbse-previous-year-solved-papers-class-12-maths-outside-delhi-2014-88
Solution.
cbse-previous-year-solved-papers-class-12-maths-outside-delhi-2014-89
cbse-previous-year-solved-papers-class-12-maths-outside-delhi-2014-90

Question.20.
cbse-previous-year-solved-papers-class-12-maths-outside-delhi-2014-91
Solution.
cbse-previous-year-solved-papers-class-12-maths-outside-delhi-2014-92

Question.21. Find the particular solution of the differential equation x(1 + y2) dx – y(1 + x2) dy = 0, given that y = 1 when x = 0.
Solution.
cbse-previous-year-solved-papers-class-12-maths-outside-delhi-2014-93
cbse-previous-year-solved-papers-class-12-maths-outside-delhi-2014-94

Question.22.
cbse-previous-year-solved-papers-class-12-maths-outside-delhi-2014-95
Solution.
cbse-previous-year-solved-papers-class-12-maths-outside-delhi-2014-96

SECTION – C

Question.28.
cbse-previous-year-solved-papers-class-12-maths-outside-delhi-2014-97
Solution.
cbse-previous-year-solved-papers-class-12-maths-outside-delhi-2014-98
cbse-previous-year-solved-papers-class-12-maths-outside-delhi-2014-99

Question.29. Prove that the height of the cylinder of maximum volume that can be inscribed in a sphere of radius R is 2R/√3. Also find the maximum volume.
Solution.
cbse-previous-year-solved-papers-class-12-maths-outside-delhi-2014-100
cbse-previous-year-solved-papers-class-12-maths-outside-delhi-2014-101
cbse-previous-year-solved-papers-class-12-maths-outside-delhi-2014-102

SET III

Note: Except for the following questions, All the remaining question have been asked in previous set.

SECTION – A

Question.9.
cbse-previous-year-solved-papers-class-12-maths-outside-delhi-2014-103
Solution.
cbse-previous-year-solved-papers-class-12-maths-outside-delhi-2014-104

Question.10.
cbse-previous-year-solved-papers-class-12-maths-outside-delhi-2014-105
Solution.
cbse-previous-year-solved-papers-class-12-maths-outside-delhi-2014-106
cbse-previous-year-solved-papers-class-12-maths-outside-delhi-2014-107

SECTION-B

Question.19. Using properties of determinants, prove that:
cbse-previous-year-solved-papers-class-12-maths-outside-delhi-2014-108
Solution.
cbse-previous-year-solved-papers-class-12-maths-outside-delhi-2014-109
cbse-previous-year-solved-papers-class-12-maths-outside-delhi-2014-110

Question.20.
cbse-previous-year-solved-papers-class-12-maths-outside-delhi-2014-111
Solution.
cbse-previous-year-solved-papers-class-12-maths-outside-delhi-2014-112
cbse-previous-year-solved-papers-class-12-maths-outside-delhi-2014-113

Question.21. Find the particular solution of the differential equation log (dx/dy) =3x + 4y given that y = 0 when x = 0.
Solution.
cbse-previous-year-solved-papers-class-12-maths-outside-delhi-2014-114
cbse-previous-year-solved-papers-class-12-maths-outside-delhi-2014-115

Question.22.
cbse-previous-year-solved-papers-class-12-maths-outside-delhi-2014-116
Solution.
cbse-previous-year-solved-papers-class-12-maths-outside-delhi-2014-117

SECTION-C

Question.28. If the sum of the lengths of the hypotenuse and a side of a right triangle is given, show that the area of the triangle is maximum, when the angle of between them is 60°.
Solution.
cbse-previous-year-solved-papers-class-12-maths-outside-delhi-2014-118
cbse-previous-year-solved-papers-class-12-maths-outside-delhi-2014-119
cbse-previous-year-solved-papers-class-12-maths-outside-delhi-2014-120

Question.29.
cbse-previous-year-solved-papers-class-12-maths-outside-delhi-2014-121
Solution.
cbse-previous-year-solved-papers-class-12-maths-outside-delhi-2014-122
cbse-previous-year-solved-papers-class-12-maths-outside-delhi-2014-123

CBSE Previous Year Solved Papers Class 12 Maths Outside Delhi 2013

CBSE Previous Year Solved  Papers Class 12 Maths Outside Delhi 2013

Time allowed: 3 hours                                                                                          Maximum Marks : 100
General Instructions:

  1. All questions are compulsory.
  2. Please check that this question paper contains 26 questions.
  3. Questions 1-6 in Section A are very short-answer type questions carrying 1 mark each.
  4. Questions 7-19 in Section B are long-answer I type questions carrying 4 marks each.
  5. Questions 20-26 in Section C are long-answer II type questions carrying 6 marks each.
  6. Please write down the serial number of the question before attempting it.

SET I

SECTION – A

cbse-previous-year-solved-papers-class-12-maths-outside-delhi-2013-1
cbse-previous-year-solved-papers-class-12-maths-outside-delhi-2013-2
cbse-previous-year-solved-papers-class-12-maths-outside-delhi-2013-3
cbse-previous-year-solved-papers-class-12-maths-outside-delhi-2013-4
cbse-previous-year-solved-papers-class-12-maths-outside-delhi-2013-5
cbse-previous-year-solved-papers-class-12-maths-outside-delhi-2013-6

SECTION – B

cbse-previous-year-solved-papers-class-12-maths-outside-delhi-2013-7
cbse-previous-year-solved-papers-class-12-maths-outside-delhi-2013-8
cbse-previous-year-solved-papers-class-12-maths-outside-delhi-2013-9
cbse-previous-year-solved-papers-class-12-maths-outside-delhi-2013-10
cbse-previous-year-solved-papers-class-12-maths-outside-delhi-2013-11
cbse-previous-year-solved-papers-class-12-maths-outside-delhi-2013-12
cbse-previous-year-solved-papers-class-12-maths-outside-delhi-2013-13
cbse-previous-year-solved-papers-class-12-maths-outside-delhi-2013-14
cbse-previous-year-solved-papers-class-12-maths-outside-delhi-2013-15
cbse-previous-year-solved-papers-class-12-maths-outside-delhi-2013-16
cbse-previous-year-solved-papers-class-12-maths-outside-delhi-2013-17
cbse-previous-year-solved-papers-class-12-maths-outside-delhi-2013-18
cbse-previous-year-solved-papers-class-12-maths-outside-delhi-2013-19
cbse-previous-year-solved-papers-class-12-maths-outside-delhi-2013-20
cbse-previous-year-solved-papers-class-12-maths-outside-delhi-2013-21
cbse-previous-year-solved-papers-class-12-maths-outside-delhi-2013-22
cbse-previous-year-solved-papers-class-12-maths-outside-delhi-2013-23
cbse-previous-year-solved-papers-class-12-maths-outside-delhi-2013-24
cbse-previous-year-solved-papers-class-12-maths-outside-delhi-2013-25
cbse-previous-year-solved-papers-class-12-maths-outside-delhi-2013-26
cbse-previous-year-solved-papers-class-12-maths-outside-delhi-2013-27
cbse-previous-year-solved-papers-class-12-maths-outside-delhi-2013-28
cbse-previous-year-solved-papers-class-12-maths-outside-delhi-2013-29

SECTION – C

cbse-previous-year-solved-papers-class-12-maths-outside-delhi-2013-30
cbse-previous-year-solved-papers-class-12-maths-outside-delhi-2013-31
cbse-previous-year-solved-papers-class-12-maths-outside-delhi-2013-32
cbse-previous-year-solved-papers-class-12-maths-outside-delhi-2013-33
cbse-previous-year-solved-papers-class-12-maths-outside-delhi-2013-34
cbse-previous-year-solved-papers-class-12-maths-outside-delhi-2013-35
cbse-previous-year-solved-papers-class-12-maths-outside-delhi-2013-36
cbse-previous-year-solved-papers-class-12-maths-outside-delhi-2013-37
cbse-previous-year-solved-papers-class-12-maths-outside-delhi-2013-38
cbse-previous-year-solved-papers-class-12-maths-outside-delhi-2013-39
cbse-previous-year-solved-papers-class-12-maths-outside-delhi-2013-40
cbse-previous-year-solved-papers-class-12-maths-outside-delhi-2013-41

27. In a hockey match, both teams A and B scored same number of goals up to the end of the game, so to decide the winner, the referee asked both the captains to throw a die alternately and decided that the team, whose captain gets a six first, will be declared the winner. If the captain of team A was asked to start, find their respective probabilities of winning the match and state whether the decision of the referee was fair or not. [6]
Solution: Probability of getting a six by the captains of both the teams A and B is
P(A) = 1/6
cbse-previous-year-solved-papers-class-12-maths-outside-delhi-2013-42
cbse-previous-year-solved-papers-class-12-maths-outside-delhi-2013-43

28.A manufacturer considers that men and women workers are equally efficient and so he pays them at the same rate. He has 30 and 17 units of workers (male and female) and capital respectively which he uses to produce two types of goods A and B. To produce one unit of A, 2 workers and 3 units of capital are required while 3 workers and 1 unit of capital is required to produce one unit of B. If A and B are priced at Rs 100 and Rs 120 per unit respectively how should he use his resources to maximize the total revenue ? Form the above as an LPP and solve graphically. 
Do you agree with this view of the manufacturer that men and women workers are equally efficient and so should be paid at the same rate ? [6]
Solution: Let x units of the goods A and y units of goods B be produced to maximize the total revenue. Then the total revenue is Z = 100x + 120y. This is a linear function which is to be maximized. Hence it is the objective function. The constraints are as per the following table:
cbse-previous-year-solved-papers-class-12-maths-outside-delhi-2013-44
cbse-previous-year-solved-papers-class-12-maths-outside-delhi-2013-45
cbse-previous-year-solved-papers-class-12-maths-outside-delhi-2013-46

29. The management committee of a residential colony decided to award some of its members (say x) for honesty, some (say y) for helping others and some others (say z) for supervising the workers to keep the colony neat and clean. The sum of all the awardees is 12. Three times the sum of awardees for cooperation and supervision added to two times the number of awardees for honesty is 33. If the sum of the number of awardees for honesty and supervision is twice the number of awardees for helping others using matrix method, find the number of awardees of each category. Apart from these values namely honesty, cooperation and supervision, suggest one more value which the management of the colony must include for awards.[6]
cbse-previous-year-solved-papers-class-12-maths-outside-delhi-2013-47
cbse-previous-year-solved-papers-class-12-maths-outside-delhi-2013-48
cbse-previous-year-solved-papers-class-12-maths-outside-delhi-2013-49

SET II

Note: Except for the following questions, All the remaining questions have been asked in previous set.

SECTION – A

cbse-previous-year-solved-papers-class-12-maths-outside-delhi-2013-50
cbse-previous-year-solved-papers-class-12-maths-outside-delhi-2013-51
cbse-previous-year-solved-papers-class-12-maths-outside-delhi-2013-52

SECTION – B

cbse-previous-year-solved-papers-class-12-maths-outside-delhi-2013-53
cbse-previous-year-solved-papers-class-12-maths-outside-delhi-2013-54
cbse-previous-year-solved-papers-class-12-maths-outside-delhi-2013-55

SECTION – C

cbse-previous-year-solved-papers-class-12-maths-outside-delhi-2013-56
cbse-previous-year-solved-papers-class-12-maths-outside-delhi-2013-57
cbse-previous-year-solved-papers-class-12-maths-outside-delhi-2013-58

SET III

Note: Except for the following questions, All the remaining questions have been asked in previous sets.

SECTION – A

cbse-previous-year-solved-papers-class-12-maths-outside-delhi-2013-59
cbse-previous-year-solved-papers-class-12-maths-outside-delhi-2013-60

SECTION – B

cbse-previous-year-solved-papers-class-12-maths-outside-delhi-2013-61
cbse-previous-year-solved-papers-class-12-maths-outside-delhi-2013-62
cbse-previous-year-solved-papers-class-12-maths-outside-delhi-2013-63
cbse-previous-year-solved-papers-class-12-maths-outside-delhi-2013-64
cbse-previous-year-solved-papers-class-12-maths-outside-delhi-2013-65
cbse-previous-year-solved-papers-class-12-maths-outside-delhi-2013-66
cbse-previous-year-solved-papers-class-12-maths-outside-delhi-2013-67
cbse-previous-year-solved-papers-class-12-maths-outside-delhi-2013-68
cbse-previous-year-solved-papers-class-12-maths-outside-delhi-2013-69
cbse-previous-year-solved-papers-class-12-maths-outside-delhi-2013-70

Solved CBSE Sample Papers for Class 12 Political Science Set 8

Solved CBSE Sample Papers for Class 12 Political Science Set 8

[Time Allowed : 3 hrs.]                                                                                              [Maximum Marks] : 100

General Instruction:

  1. All Questions are compulsory.
  2. Question numbers 1-5 are of 1 mark each. The answers to these questions should not exceed 20 words each.
  3. Question numbers 6-10 are of 2 marks each. The answers to these questions should not exceed 40 words each.
  4. Question numbers 11-16 are of 4 marks each. The answers to these questions should not exceed 100 words each.
  5. Question numbers 17-21 are of 5 marks each. The answers to this question should not exceed 150 words.
  6. Question numbers 22-27 are of 6 marks each. The answers to this question should not exceed 150 words.

Question.1.What is ‘Schengen Visa’ ?
Answer. The historical agreement that led to the formation of European Union (EU), which removed border restrictions, is known as “Schengen visa”.

Question.2.Partition of India was a result of which theory? Who advanced this theory ?
Answer. Two-nation theory and it was propounded by Muhammad Ali Jinnah.

Question.3.Fill in the blanks in the following sentence with apropriate words:
Answer. The United Nations was founded as a………to the League of Nations. It was establised in………….
The United Nations was founded as a successor to the league of Nations. It was establised in 1945.

Question.4.After the dominance of the Congress party for more than 15 years, there was an upset in the 1967 General Election. What was this upset ?
Answer. The results of 1967 election jolted the Congress both at the national and the states levels. The political observers described the election result as a “Political Earthquake”.

Question.5.What was the movement against outsiders in Assam ?
Answer. In Assam, local communities started anti-foreigner movement against the people who were seen as “outsiders” (or) migrants (Illegal Bengali Muslim settlers from Bangladesh).

Question.6.Describe any two military features of the Cold War.
Answer.

  1. Both USA and Soviet Union set up military blocs to show their supremacy.
    USA formed NATO, SEATO and CENTO USSR WARSAW Part.
  2. They formed alliances and were expected to behave in a rational and responsible manner. It means the allied Rations understood the risks in fighting wars, where super powers are involved.

Question.7.When were the Organisation for European Economic Cooperation and European Economic Community established respectively ?
Answer. Organisation for European Economic Cooperation was established 1948.
European Economic Community was established 1957.

Question.8.What was Afro-Asian unity ?
Answer. The Bandung Conference was convened to strengthen the Afro-Asian unity. It was held in the Indonesian city of Bandung in 1955.
This convention led to the establishment of NAM and marked the zenith of India’s engagement with the new independent Asian and African nations.

Question.9.”Sardar Patel played a historic role in negotiations and solved the problems being faced by India after independence.” With whom did he negotiate and what was the outcome ?
Answer. Sardar Vallabbhai Patel was negotiating with the rulers of Princely States and merged most of them into the Indian Union.
For example, there were 26 small states in today’s Odisha and 14 big states and 119 small states in today’s Gujarat.

Question.10. Highlight any two environmental concerns of global politics.
Answer. There is a Great Global concern on environment in the following aspects:

  1. Agricultural land is losing fertility due to overuse of fertilizers and grasslands have been overgrazed. Food production has been severely affected because of polluted waterbodies.
  2. The ozone layer depletion becomes a danger to ecosystems and human health.

Question.11. Explain any four reasons why Super Powers encouraged alliances with smaller countries.
Answer. With their nuclear weapons and regular armies superpowers were so powerful that the combined power of most of the smaller states in Asia and Africa and even in Europe, was no match to them. Yet, the smaller states were helpful to the superpowers in gaining access to:

  • Territory-from where the superpowers could launch their weapons and troops.
  • Locations – from where the superpowers could spy on each other.
  • Economic support – in that many small allies together could help pay for military
    expenses.
  • Ideological reasons – They were also important for ideological reasons. Like the loyalty of allies suggested that the superpowers were winning the war of ideas as well.

Question.12. What role has been played by the European Union in solving the problems of the European countries ?
Answer. European Union is a group of European capitalist countries established in 1992. It was founded for a common foreign and security policy, cooperation injustice and home affairs.

  1. European Union functions as an important bloc in international organisations such as the World Trade Organisation. In this way, it is able to intervene in economic areas.
  2. The European Union has tried to expand areas of cooperation while acquiring new members especially from the erstwhile Soviet Bloc.
  3. At the same time European Union has a great influence in the world arena, and on some of the UN policies because its two members, Britain and France, hold permanent seats in the UN Security Council.
  4. European Union’s another important feature is its very effective influence in the arenas of diplomacy, economic investments and negotiations.

Question.13. Define globalisation. Describe any two of its characteristics.
Answer. The term globalisation means integration of our economy with world economy. It involves free flow of trade, capital, technology and human resources.
Characteristic features of globalisation:

  1. Globalisation is a complex process: Globalisation involves economic, political and cultural dimensions. All the three aspects are co-related and depend on each other in making policies and programmes. For example, crisis in a country may affect the developmental process of other country.
  2. Globalisation is associated with other two processes: Globalisation is linked with two processes namely liberation and privatisation. These two processes ensure smooth flow of globalisation process.

Question.14. What is meant by environment ? Suggest any two steps for the environmental improvement.
Answer. The term “Environment” refers to surrounding circumstances or regions of an area. Steps to improve the Environment are :

  1. Afforestation : Planting more trees helps in maintaining ecological balance, prevent soil erosion and enhance oxygen and the water cycle.
    Planting of trees should be encouraged in almost all festivals.
    Schools and colleges should have compulsory projects to create awareness and to be role model to the society.
  2. Location of Industries : Industries should be located away from towns and cities. Proper disposal of air / water / solid materials with scientific methods should be strictly implemented. Eco-friendly industries showed be preferred.

Question.15. “The Congress party proved its dominance in comparison to a number of other political parties while contesting the first three General Elections.” Justify this statement.
Answer. In the first three General elections the Congress Party maintained its dominance. In the first election Indian National Congress was expected to win and when the final results were declared the extent of the victory of the Congress did surprise many. The party won 364 of the 489 seats in the first Lok Sabha and finished way ahead of any other challenger.

  1. The Communist Party of India that came next in terms of seats won only 16 seats.
  2. In the state elections also the Congress scored big victories. It won a majority of seats in all the states except Travancore Cochin (Kerala). Madras and Odisha.
  3. So the party ruled all over the country at the national and the state level and Jawaharlal Nehru became the Prime Minister.
  4. In the second and third General Election held in 1957 and 1962 respectively, the Congress Party maintained the same position in the Lok Sabha by winning three-fourth of seats.
  5. None of the opposition parties could win even one-tenth of the number of seats captured
    by the Congress. *(Any four)

Question.16. Explain any four challenges faced by India at the time of independence.
Answer. Immediately after independence there were many challenges or problems in India that needed a solution. These challenges can be categorised as:
(i) A challenge to shape a nation.
(ii) A challenge to establish democracy and,
(iii) A challenge to ensure the development and well-being of the entire society.
(iv) A challenge to accommodate refugees.
(i) To Shape a Nation : The first and foremost challenge was the political unification and integration of the territory. India is a land of continental size and diversity. There were around 600 states of varying sizes and population. The partition of the country appeared to confirm every one’s worst fears. Hence, there was a serious question about the future of India, i.e., would India survive as a unified country? Sardar Vallabhbhai Patel took upon himself the task of integrating these princely states, which was sometimes completed in stages.
(ii) To Establish Democracy : Another challenge was to develop democratic practices in accordance with the Constitution, i.e., India opted for representative democracy, based on the parliamentary form of government.
(iii) To Ensure the Development and Well-being of the Society: The third challenge was to evolve effective policies for economic development and eradication of poverty and unemployment. The Indian Constitution set out in the Directive Principles of State Policy the welfare goals that democratic polities must achieve.
All these challenges required a deliberate effort which India put in accommodating social differences, establishing a welfare state and by democratising political institutions.
(iv) To Accomodate Refugees : The migration of people took place in two forms.

  1. large number of people migrated from East and West Pakistan.
  2. Migration of people from the princely states.

Question.17. Some of the statements below are incorrect. Identify the incorrect statements and rewrite those with necessary correction :
(a) Social movements are hampering the functioning of India’s democracy.
(b) The main strength df social movements lies in their mass base across social sections.
(c) Social movements in India emerged because there were many issues that political parties did not address.
Answer.
(a) Social movements are hampering the functioning of India’s democracy.
Rewritten with correction
(b)

  1.  Social movements involve a gradual process of coming together of people with
    similar problems.
  2. Social movements in India have been involved in educative task like making people aware of their rights.

(c) Social movements in India emerged because there were many issues that political parties did not address.
Rewritten with correction : Social Movements in India emerged to reduce the possibility of deep social conflict and disaffection of the groups from democracy.

Question.18. Read the passage the answer the questions below :
Indian democracy was never so dose-to a two-party system as it was during the 1977 elections. However, the next few years saw a complete change. Soon after its defeat, the Indian National Congress split into two groups…………….. The Janata Party also went through major convulsions………………… David Butler, Ashok Lahiri and Prannoy
Roy.—Partha Chatterjee.
(a) What made the party system in India look like a two-party system in 1977?
(b) Many more than two parties existed in 1977. Why then are the authors describing this period as close to a two-party system ?
(c) What caused splits in Congress and the Janata Parts ?
Answer.
(a) The imposition of emergency in 1977 and The crises among political parties (ruling party) made the party system in India look like a two-party system.
(b) Two parties which existed in 1977 were

  1. Congress and
  2. Non-Congress.
    The authors are describing this period as close to a two-party system because this period saw the end of dominance of a single party in India and emergence of Janata Party.

(c) Causes for the splits in Congress and Janata Party were:

  1. For Congress—On the issue of candidate for the presidential election in 1969.
  2. For Janata Party—Tension among three leaders
    Morarji Desai, Charan Singh, and Jagjivan Ram for the leadership in 1979.

Question.19. Read the following passage and answer the questions below :
“In the early years of Independence, two contradictory tendencies were already well advanced inside the Congress party. On the one hand, the national party executive endorsed socialist principles of state ownership, regulation and control over key sectors of the economy in order to improve productivity and at the same time curb economic concentration. On the other hand, the national Congress government pursued liberal economic policies and incentives to private investment that was justified in terms of the sole criterion of achieving maximum increase in production”.
                                                                                                                                                          —Francine Frankel
(a) What is the contradiction that the author is talking about ? What would be the political implications of a contradiction like this ?
(b) If the author is’correct, why is it that the Congress was pursuing this policy ? Was it related to the nature of the opposition parties ?
(c) Was there also a contradiction between the central leadership of the Congress party and its State level leaders ?
Answer.
(a) The author Francine Frankel was talking about the double-minded nature of the Congress that it advocated socialist policy on one hand and capitalist policy in some sectors where the government needed large investment in the industrial development.
(b) The government adopted ‘Mixed Economy’ where elements of both capitalist and socialist principles are included. This was criticised on the ground that the planners refused to provide the private sector with enough opportunities.
(c) As most of the states were ruled by the Congress, there was not much contradiction between the central leadership of the Congress and the state level leaders. The states of West Bengal and Kerala criticised the planning programme of the Congress.

Question.20. Identify the following six states created after 1956 marked as A, B, C, D, E and F on the given outline political map of India. Match the names of the states given below with the letter codes and write them in your Answer-Book.
Name of the states : Chhattisgarh, Gujarat, Meghalaya, Mizoram, Himachal Pradesh and Uttarakhand.
Also match the year of the creation of each state from amongst the years given below and write them against the names of the states in your Answer-Book.
Years of creation :
1987,1966, 2000,1960, 1972 and 2000.
solved-cbse-sample-papers-for-class-12-political-science-set-8-1
Answer.
A. Uttarakhand            – 2000
B. Gujarat                      – 1960
C. Chhatisgarh             – 2000
D. Himachal Pradesh – 1966
E. Mizoram                   – 1987
F. Maghalaya                – 1972

Question.21.
solved-cbse-sample-papers-for-class-12-political-science-set-8-2
Related to the end of the decade of 1960, study the cartoon given above and answer the following questions:
(a) The cartoon refefs to which event ?
(b) This event happened in which year ?
(c) Why is the lady in the cartoon so pleased ?
(d) Who is the man wearing the garland ?
(e) Who is the person on his knees ?
(f) Name the Congress leader lying on the ground.
Answer.
(a) Presidential Election, 1969
(b) 1969
(c) Because Indira Gandhi supporter V.V. Giri won the Presidential election.
(d) V.V. Giri
(e) S. Nijalingappa
(f) N. Sanjeeva Reddy

Question.22. What was ‘Shock-Therapy’c? Examine any four consequences of ‘Shock-Therapy’.
Or
“It is said that Non-Aligned Movement in the present scenario, has become irrelevant.” Do you agree with this statement ? Support your answer with any three arguments.
Answer. Shock-Therapy: The term shock-therapy refers to transition of authoritarian socialist system to a democratic capitalist system.

  1. ‘Shock-Therapy brought ruin to the economies and disaster upon the people of the entire region.
    Consequences of Shock-Therapy:
  2. In Russia, the large public controlled industrial complex almost collapsed because 90 per cent of its industries were put up for sale to private individuals and companies. Many industries lost their value and it was called as “the largest garage sale in history”.
  3. The value of the Ruble (the Russian currency) declined very rapidly and the rate of inflation was so high that people lost all their savings.
  4. The old system of social welfare was also systematically destroyed. The withdrawal of government subsidies pushed large sections of the people into poverty.
    Privatisation led to the new disparities.
    Russia was divided between rich and poor regions and there was great economic inequality between people.

Or
Non-alignment as a strategy evolved in the Cold War context, but the end of Cold War and a Unipolar world did not see the end of the non-alignment.
Non-alignment still contains some core values and end-using ideas.

  1. Its emphasis has shifted from “political issues to economic issues”.
    Liberalisation of third world economies for rapid development of the countries of south now remains the main concern of NAM.
  2. Issues like democracy, disarmament, human rights and neo-colonialism are as relevant today as earlier.
    With the disintegration of the erstwhile USSR, there is only one super power therefore, it is very essential for NAM to make sincere efforts to check USA from taking unilateral decisions.
  3. Now its emphasis has also shifted towards poverty alleviation, New International Economic Order based on equality, equity and justice and promotion of industrialisation.

Question.23. Examine the role played by India in the implementation of the UN policies and programmes.
Or
What is cooperative security ? Explain India’s security strategy.
Answer. Role played by India in the implementation of UN Policies and Programmes :

  1. India believes that a strengthened and revitalised UN is desirable in the changing world.
  2. It also supports an enhanced role for the UN in promoting development and cooperation among states.
  3. India believes that development should be central to the UN’s agenda as it is a vital precondition for the maintenance of international peace and security.
  4. One of India’s major concerns has been the composition of the Security Council which has remained largely static. India considers that this has harmed the representative character of the Security Council.
  5. It also argues that an expanded council, with more representatives, will enjoy greater support in the world community.
  6. India also supports that developing countries as the members of the General Assembly
    should also have a role in shaping the decisions in the Security Council which affect them. ‘

Or
Cooperative Security : The term cooperative security refers to international cooperation between the countries or organisations to protect the people from any kind of threats like terrorism, poverty, etc. –
India’s Security Strategy : In India both traditional and non-traditional notions of security have been given equal priority because India has faced traditional (military) and non- traditional threats to its security which have emerged from within as well as outside its borders.
Its security strategy has four broad components which have been used in a varying combinations from time to time like :
(a) Strengthening military capabilities : India tried to strengthen its military capabilities because it has been involved in conflicts with its neighbours and has fulfledged wars like with Pakistan in 1947-48,1965,1971 and 1999 and with China in 1962.
Apart from this, since it is surrounded by nuclear armed countries in the south Asian region India’s decision to conduct Nuclear tests in 1998 was justified in terms of safeguarding national security.
(b) Strengthening international Norms and institutions : The second area of India’s security strategy has been to strengthen international norms and institutions to protect its security interests.
We have some examples to prove the above statement:

  1. India’s first Prime Minister Jawaharlal Nehru supported the cause of Asian solidarity, declonisation, disarmament and the UN as a fora in which international conflicts could be settled.
  2. It argued for an equitable New International Economic Order (NIEO).
  3. India also joined Kyoto Protocol to check global warming. Besides, as a followup to cooperative security, India sends its troops abroad on UN peace keeping missions.

(c) Strengthening internal security: Indian security strategy is also geared towards meeting security challenges within the country.
In order to accommodate several militant groups from areas such as Nagaland, Mizoram, Punjab and Kashmir among others, India has tried to preserve national unity by adopting a democratic political system which allows different communities and groups of people to freely articulate their grievances and share political power.
(d) Developing India’s economy : In order to uplift its citizens out of poverty, misery, and huge economic inequality, India strategically planned to develop its economy.

Question.24. “The political competition between the Congress and the Muslim League and the British role led to the decision for the creation of Pakistan.” In the light of the above statement, assess any six consequences of the partition of India.
Or
The Shah Commission was appointed in 1977 by the Janata Party government. Why was it appointed and what were its findings.
Answer. The partition year 1947 was the year of one of the largest, most abrupt, unplanned and tragic transfers of population that human history has known.

  1. Communal Riots: In the name of religion people of one community ruthlessly killed and maimed people of the other community. There were killings and atrocities on both sides of the border. Cities like Lahore, Amritsar and Kolkata became divided into “communal zones”.
  2. Social Sufferings : People went through immense sufferings. They were forced to abandon their homes and move across borders. Minorities on both sides of the border fled their homes and often secured temporary shelter in “refugee camps”. Thousands of women were abducted on both sides of the border.
  3. In many cases women were killed by their own family members to preserve the “family honour”.
  4. Many children were separated from their parents. Those who did manage to cross the border found that they had no home. Hence, for lakhs of these “refugees” the country’s freedom meant life in ‘refugee camps’.
  5. It is estimated that the partition forced about 80 lakh people to migrate across the new border. Between five to ten lakh people were killed in partition related violence.
  6. Besides, the partition had also Created severe conflict between the two communities, i.e., Hindu and Muslims, keeping in view all these trauma, writers, poets and film makers in India and Pakistan have expressed the ruthlessness of the killings and the sufferings of displacement and violence in their novels, short stories, poems and films. They coined a phrase to describe partition – as a “division of hearts”.

Or
The Shah Commission was appointed in May 1977 by the Janata Party government. It was a commission of inquiry headed by Justice J.C. Shah, retired Chief Justice of the Supreme Court of India to inquire into :

  1. Several aspects of allegations of abuse of authority, excesses and malpractices committed and action taken in the wake of emergency, proclaimed on 25th June 1975.
  2. The commission examined various kinds of evidences and called scores of witnesses to give testimonies. This included Indira Gandhi who appeared before the commission but refused to answer any questions.

Findings of Shah Commission
The findings of Shah Commission in the form of reports were tabled in the two houses of parliament.

  1. The investigations by Shah Commission after the Emergency found out that there were many ‘excesses’ committed during the emergency.
  2. It estimated that nearly one lakh eleven thousand people were arrested under preventive detention laws.
  3. Several restrictions were put on the press sometimes without proper legal sanctions.
  4. The Shah Commission report also mentions that the general manager of the Delhi Power Supply Coorporation received verbal orders from the offices of the Lt. Governor of Delhi to cut electricity to all newspaper presses at 2 a.m. on 26 June 1975.

Question.25. What is meant by Emergency ? Was the Emergency declared on 25th June, 1975 on the request of the Prime Minister,Indira Gandhi, necessary ? Support your answer with any four arguments.
Answer. Emergency : It is an extraordinary condition in which normal democratic politics can not function and special powers are granted to the government.
Indira Gandhi had no option except to impose/declare emergency in 1975 :

  1. Indira Gandhi argued that in a democracy, the opposition parties should allow the elected ruling party to govern according to its policies.
  2. She felt that the opposition parties and their frequent agitations, protests and collective action led to political instability.
  3. She also held that opposition cannot continuously have extra-parliamentary politics targeting the government.
  4. She alleged that subversive forces were not allowing her government to implement her progressive programmes (including 20-Point Programme).
  5. She also felt that Judiciary was acting as a hurdle to her government.
  6. She believed that to control such subversive forces including some newspapers and leading businessmen emergency was necessary.

No need to impose emergency:

  1. Popular struggle occurs when there was a need and the opposition and others considered
    that it is a legacy of national struggle for freedom.
  2. The movements that occurred before emergency period in Bihar and Gujarat were non¬violent.
  3. Law and order situation was mostly normal and Emergency during such condition ‘reflected the overreaction’, on the part of the government.
  4. Indira Gandhi did not get the approval of the cabinet while declaring Emergency.
  5. It was also believed that she misused the extraordinary constitutional provisions to remain in power in the name of saving the nation.
  6. The Government should obey the Judiciary’s verdict. (Any four)

Question.26. How was the ‘one party dominant system’ in India different from similar systems elsewhere? Did the dominance of one party mean that India was not really a democracy? Give reasons to support your answer.
Or
What was the green revolution? Which areas did it affect most? Mention two positive and two negative Consequences of the green revolution.
Answer.
(a) One party dominant system means that a single party enjoys monopoly of power over a long period of time and exclusion of other parties to reach power.
(b) No. The dominance of one party did not mean that India was really not a democracy. Even though Congress had dominated the political scene for two decades, the Indian political system is still democratic for the following reasons:

  1. Open nature of the Indian political system, i.e., existence of multi-party system.
  2. At the time of independence, there was no strong political party except the Congress.
  3. The role of Congress during our freedom struggle could be remembered and people cast votes as their tribute to the Congress leaders.
  4. The regional parties gained importance at national levels and multi-party system had come into the picture since 1989.
  5. The advancement in the electoral system (electronic voting machine, common platform for political agenda) strengthened the democratic pattern.
  6. Active participation of women and improvement in Panchayat Raj system and its success testify the Indian political system to be democratic.

Or
The term ‘Green Revolution’ refers to the new methods adopted by the government in agriculture to increase foodgrain production during 1960s.
The modern method includes the use of high yielding variety of seeds, use of fertilisers and pesticides, better irrigation methods, use of modern tools and highly subsidised prices, etc. Positive aspects of Green Revolution :

  1. The region where it was introduced like Punjab, Haryana and Western UP became
    prosperous.
  2. Most of middle class peasants benefited from the Green Revolution.
  3. The use of modern technology made peasants skilled and they were able to use it in maximum areas for cultivation.
  4. It led to easy availability of latest technology, good equipments, loans and other facilities like Crop Insurance Scheme, etc.
  5. The food grain increased manifold and food security was ensured.
  6. Many new varieties of food grains, adaptable to our climatic conditions, increased the
    net sown area.

Negative aspects of Green Revolution :

  1. It increased polarisation between classes and regions. Some regions of North and Northwestern India like Punjab and Haryana became prosperous while rest of India remained backward.
  2. For small farmers it is still a dream because it requires large areas and investment for cultivation.
  3. Some of the traditional varieties of food grains lost their importance.
  4. There was a sharp contrast between the rich and the poor farmers.
  5. The left wing organisations used the poor peasants as their vote banks.
  6. The illiterate peasants could not undei stand the system behind the Green Revolution.

Question.27. Explain the increase and decrease in the role of the state activities in the developing countries due to globalisation.
Or
“Pursuing economic’development without causing further damage to the global environment is a major challenge before the States.” Suggest any three measures to overcome this problem.
Answer. The term globalisation refers to the interaction of our economy with foreign economy. It will generate greater economic growth and well being for larger sections of population. The impact of globalisation could be seen in India when the Congress government introduced new economic policy in 1991. It resulted in foreign investment through multi-national companies (MNC) which not only strengthened the economic growth but also changed the lifestyle of the people. It is a boost for the developing countries.
“Power of the state has increased due to enhanced technology. From India’s point of view this is an important impact because modern technology (computer and internet services) has brought many changes and increased our market contacts.
Globalisation led to flow of ideas across the world. We can say that global culture is due to the imposition of western culture on the rest of the world. It affects us at our home, in what we eat, drink, wear and indeed-in what we think. It shapes, what we think, are our preferences.
Events taking place in one part of the world leave an impact on other parts. Natural calamities like Tsunami, bird flue, etc. are no longer confined to any particular nation. The idea of transparency has left a mark on the developing countries.
The cultural impact of globalisation can be seen in the following examples. The ‘McDonaldisation’ of the world, with cultures seeking to buy into the dominant American dream.
Or
Pursuing economic development without damaging global environment:
In recent times the concept of sustainable development is widely spoken of. It means that the development should be at a faster rate without affecting environment and also the needs of future generations. It is to be kept in mind that pursuing economic development should not affect global environment.
Some of these measures can be as follows :

  1. Use of non-conventional resources such as water, wind, solar, natural gas could help to have clean environment. All public vehicles should use such fuels and the private
    individuals should also try to use clean fuels.
  2. Conventional sources like coal and petroleum are limited and cause damage to environment in the form of air pollution. This can be replaced by renewable resources, e.g.: CNG has reduced the level of pollution in Delhi.
  3. We should use environmentally sound technology which would help both development as well as to have clean environment, e.g., the new construction of buildings should be accompanied by natural parks and rainwater harvesting system.
  4. The harmful gases (Greenhouse Emissions) should be reduced because these increase the temperature which leads to global warming. It is believed that the developed nations should take the responsibility of increasing greenhouse gases (due to their large scale industrial activities).
  5. Afforestation should be increased because it helps to grow more trees at all possible levels. The forest policy should cover this as the main plank which would help both economic and environmental aspects.
  6. The government laws should be very strict in handling environment issues, e.g., there should be a ban on forest clearing and any activity which is related to deforestation. There should be a well planned system for the location of industries and the industrial waste should be disposed off through a safe process/method.

CBSE Previous Year Solved Papers Class 12 Computer Science Outside Delhi 2014

CBSE Previous Year Solved  Papers  Class 12 Computer Science Outside Delhi 2014

Time allowed : 3 hours                                                                                           Maximum Marks: 70

General Instructions :

  1.  There are a total of 26 questions and five sections in the question paper, All questions are compulsory.
  2. Section A contains question number 1 to 5, Very Short Answer type questions of one mark each.
  3.  Section B contains question number 6 to 10, Short Answer type I questions of two marks each.
  4.  Section C contains question number 11 to 22, Short Answer type II questions of three marks each.
  5.  Section D contains question number 23, Value Based Question of four marks.
  6. Section E contains question number 24 to 26, Long Answer type questions of five marks each.
  7. There is no overall choice in the question paper, however, an internal choice is provided in one question of two marks, one question of three marks and all three questions of five marks. An examined is to attempt any one of the questions out of two given in the question paper with the same question number.

SET I

Question.1.(a) What is the difference between call by reference and call by value with respect to memory allocation? Give a suitable example to illustrate using a C++ code. 
Answer:
cbse-previous-year-solved-papers-class-12-computer-science-outside-delhi-2014-1

(b) Observe the following C++ code and write the name(s) of the header file(s), which will be essentially required to run it in a C++ compiler:
cbse-previous-year-solved-papers-class-12-computer-science-outside-delhi-2014-2

(c) Rewrite the followup C++ code after removing all the syntax error(s), if present in the code. Make sure that you underline each correction done by you in the code. 
Important Note:
— Assume that all the required header files are already included, which are essential to run this code.
— The corrections made by you do not change the logic of the program.
typedef char [80] STR;
void main ()
{
cbse-previous-year-solved-papers-class-12-computer-science-outside-delhi-2014-3

(d) Obtain the output of the following C++ program as expected to appear on the screen after its execution.
Important Note:
— All the desired header files are already included in the code, which are required to run the code.
cbse-previous-year-solved-papers-class-12-computer-science-outside-delhi-2014-4

(e) Obtain the output of the following C++ program, which will appear on the screen after its execution.
Important Note:
— All the desired header files are already included in the code, which are required to run the code.
cbse-previous-year-solved-papers-class-12-computer-science-outside-delhi-2014-5
cbse-previous-year-solved-papers-class-12-computer-science-outside-delhi-2014-6
cbse-previous-year-solved-papers-class-12-computer-science-outside-delhi-2014-7

(f) Read the following C++ code carefully and find out, which out of the given option (i) to (iv) are the expected correct output(s) of it. Also, write the maximum and minimum value that can be assigned to the variable Start used in the code:
cbse-previous-year-solved-papers-class-12-computer-science-outside-delhi-2014-8

Question.2. (a) What is function overloading ? Write an example using C++ to illustrate the concept of function overloading.
Answer: Function overloading means two or more function having same name, but they have either different number of parameters or different data type of the parameters.
cbse-previous-year-solved-papers-class-12-computer-science-outside-delhi-2014-9

(b) Answer the question (i) and (ii) after going through the following class:
cbse-previous-year-solved-papers-class-12-computer-science-outside-delhi-2014-10
cbse-previous-year-solved-papers-class-12-computer-science-outside-delhi-2014-11
(i) Which of the function out of Function 1,2,3,4 or 5 will get executed when the Statement 1 is executed in the above code?
(ii) Write a statement to declare a new object G with reference to already existing object H using Function 3.
Answer:
(i) Function 1 will be executed when the Statement 1 is executed.
(ii) Hospital G(H);

(c) Define a class Tourist in C++ with the following g specification:
Data Members
• CNo — to store Cab No
• CType — to store a character ‘A’, ‘B’ or ‘C’ as City Type
• PerKM — to store per kilometre charges
• Distance — to store Distance Or avelled (in Km)
Member Functions
• A constructor function to initialize CType as ‘A’ and CNo as ‘0000’
• A function CityCharges () to assign PerKM as per the following table:
cbse-previous-year-solved-papers-class-12-computer-science-outside-delhi-2014-12
• A function RegisterCab( ) to allow administration to enter the values for CNo and CType. Also, this function should call CityChargesO to assign PerKM Charges.
• A function Display() to allow user to enter the value of Distance and display CNo, CType, PerKM, PerKM * Distance (as Amount) on screen.
cbse-previous-year-solved-papers-class-12-computer-science-outside-delhi-2014-13
cbse-previous-year-solved-papers-class-12-computer-science-outside-delhi-2014-14

(d) Consider the following C++ code and answer the question from (i) to (iv):
cbse-previous-year-solved-papers-class-12-computer-science-outside-delhi-2014-15
cbse-previous-year-solved-papers-class-12-computer-science-outside-delhi-2014-16
(i) Which type of Inheritance is shown in the above example ?
(ii) Write the names of those member functions, which are ditecdy accessed from the objects of class Student.
(iii) Write the name of those data members, which can be directly accessible from the member functions of class Student.
(iii) Is it possible to directly call function Display O of class University from an object of class Department ? (Answer as YES or NO).
Answer:
(i) Multi-level inheritance is shown in the above example.
(ii) Member functions:
void Enroll();
void View();
void Enter();’
void Show();
(iii) Data Members:
long Kollno;
char Name[20];
double Budget;
(iv) No, it is not possible because DisplayO function of Campus becomes private for the object of Department class.

Question.3. (a) Write a code for a function Even Odd(int T[ ], int C) in C++, to add 1 in all the odd values and 2 in all the even values of the array T.
cbse-previous-year-solved-papers-class-12-computer-science-outside-delhi-2014-17
cbse-previous-year-solved-papers-class-12-computer-science-outside-delhi-2014-18

(b) An array A[20] [30] is stored along the row in the memory with each element requiring 4 bytes of storage. If the base address of array A is 32000, find out the location of A [15] [10]. Also, find the total number of elements present in this array.
cbse-previous-year-solved-papers-class-12-computer-science-outside-delhi-2014-19

(c) Write user-defined function Add End 2(int A[ ] [4], int N, int M) in C++ to find and display the sum of all the values, which are ending with 2 (ie., units place is 2). For example if the content of array is:
cbse-previous-year-solved-papers-class-12-computer-science-outside-delhi-2014-20
The output should be 36.
cbse-previous-year-solved-papers-class-12-computer-science-outside-delhi-2014-21

(d) Evaluate the following post fix expression. Show the status of Stack after execution of each operation separately: T, F, NOT, AND, T, OR, F, AND
cbse-previous-year-solved-papers-class-12-computer-science-outside-delhi-2014-22

(e) Write a function PUSHBOOK/) in C++ to perform insert operation on a Dynamic Stack, which contains Book No and Book_Tide. Consider the following definition of NODE, while writing your C++ code.
cbse-previous-year-solved-papers-class-12-computer-science-outside-delhi-2014-23

Question.4. Feeling in the blanks marked as the Statement 1 and the Statement 2, in be program segment given below the appropriate functions for the required task.
cbse-previous-year-solved-papers-class-12-computer-science-outside-delhi-2014-24
cbse-previous-year-solved-papers-class-12-computer-science-outside-delhi-2014-25
cbse-previous-year-solved-papers-class-12-computer-science-outside-delhi-2014-26

(b) Write the function AECount/) in C++, which should read character of a text file NOTFS.txt, should count and display the occurrence of alphabets A and F (including small case a Sjfid e too)
EXAMPLE:
If the file content is as follows:
CBSE enhanced its CCE guidelines further.
The AE Count function should display the output as
A: 1
E: 7
cbse-previous-year-solved-papers-class-12-computer-science-outside-delhi-2014-27

(c) Assume the class TOYS as declared below, write a functions in C++ to read the objects TOYS from binary file TOYS. DAT and display those details of those TOYS, which are meant for children of Age Range “5 to 8”.
cbse-previous-year-solved-papers-class-12-computer-science-outside-delhi-2014-28
cbse-previous-year-solved-papers-class-12-computer-science-outside-delhi-2014-29

Question.5. (a) Explain the concept of Cartesian Product between two tables, with the help of appropriate example.
cbse-previous-year-solved-papers-class-12-computer-science-outside-delhi-2014-30
NOTE:
Answer the question (b) and (c) on the basis of the following tables SHOPPE and ACCESSORIES
Table: SHOPPE
cbse-previous-year-solved-papers-class-12-computer-science-outside-delhi-2014-31
cbse-previous-year-solved-papers-class-12-computer-science-outside-delhi-2014-32

(b) Write a SQL query (1 to 4)
1. To display Name and Price of all the Accessories in ascending order of their Price.
2. lo display Id and S Name of all Shopee located in Nehru Place.
3. To display Minimum and Maximum price of each Name of Accessories.
4. To display Name, Price of all the Accessories and their respective S Name where they are available,
Answer: b(l) select Name, Price from ACCESSORIES order by Price;
b(2) select ID, S Name from SHOPPE where Area=’Nehru Place’;
b(3) select MIN(Price), MAX(Pnce) from ACCESSORIES group by Name;
b(4) select Name, Price, S Name ‘ from SHOPPE, ACCESSORIES
where SHOPPE.ID= ACCESSORIES.ID;

(c) Write the output of the following SQL command (1 to 4)
1. SELECT DISTINCT NAME FROM ACCESSORIES
WHERE PRICE>=5000;
2. SELECT AREA, COUNT(*), FROM SHOPPE GROUP BY
AREA;
3. SELECT COUNT (DISTINCT AREA) FROM SHOPPE;
4. SELECT NAME, PRICE * 0.05 DISCOUNT FROM ACCESSORIES WHERE SNO IN (‘SO2’, ‘SO3’);
cbse-previous-year-solved-papers-class-12-computer-science-outside-delhi-2014-33
cbse-previous-year-solved-papers-class-12-computer-science-outside-delhi-2014-34

Question.6. (a) Name the law shown below and verily it using a truth table.
X+X’.Y=X+Y
cbse-previous-year-solved-papers-class-12-computer-science-outside-delhi-2014-35

(b) Obtain the Boolean Expression for the logic shown below:
cbse-previous-year-solved-papers-class-12-computer-science-outside-delhi-2014-36

(c) Write the Product of Sum form of the function F(X, Y, Z) for the following truth representation of F:
cbse-previous-year-solved-papers-class-12-computer-science-outside-delhi-2014-37

(d) Obtain the minimal form for the following Boolean expression using Karnaugh’s Map.
cbse-previous-year-solved-papers-class-12-computer-science-outside-delhi-2014-38

Question.7.(a) Write two characteristics Wi-Fi.
Answer: The characteristics of Wi-Fi are as follows:-
1. It allows the devices to connect with the network without any wire.
2. Group of devices can be connected with single internet connection.

(b) What is the difference between E-Mail and Chat?
Answer : In Email, it is not necessary that receiver should be present online when the receiver is sending the E-mail, whereas, in Chat, it is must that the communicators should be online at the time of communication.

(c) Expand the following:

  • GSM
  • GPRS

Answer:
GSM— Global System for Mobile Communication
GPRS— General Packet Radio Service

(d) Which type of network (out of LAN, PAN and MAN) is formed, when you connect two mobiles using Bluetooth to transfer a video. 
Answer : PAN (Personal Area Network)

(e) Tech Up Corporation (TUC) is a professional consultancy company. The company is planning to set up their new offices in India with its hub at Hyderabad. As a network adviser, you have to understand their requirement and suggest them the best available solutions. Their queries are mentioned as (i) to (iv) below. ‘
Physical locations of the blocks of TUC
cbse-previous-year-solved-papers-class-12-computer-science-outside-delhi-2014-39
cbse-previous-year-solved-papers-class-12-computer-science-outside-delhi-2014-40
(i) What will be the most appropriate block, where TUC should plan to install their services?
(ii) Draw a block to block cable layout to connect all the
buildings in the most appropriate manner for efficient communication.
(iii) What will be the possible connectivity out of the following,
you will suggest to connect the new set up of offices in Hyderabad with its London based office.

  • Infrared
  • Satellite Link
  • Ethernet Cable

(iv) Which of the following device will be suggested by you to connect each computer in each of the buildings?

  • Gateway
  • Switch
  • Modem

Answer:
(i) Human Resource Block is appropriate to install the server.
(ii) CABLE LAYOUT:
cbse-previous-year-solved-papers-class-12-computer-science-outside-delhi-2014-41
(iii) Satellite Link
(iv) Switch

(f) Write the name of any two popular Open Source Software, which are used as operating system.
Answer: Linux and Unix are two Open Source operating system.

(g) Write any two important characteristics of Cloud Computing.
Answer: Two characteristic of Cloud Computing are:-
(i) It is controlled by entity and restricted to their authorized user.
(ii) It is delivered through internet 24 x 7.

CBSE previous Year Solved Papers Class 12 Maths Outside Delhi 2010

CBSE previous Year Solved  Papers  Class 12 Maths Outside Delhi 2010

Time allowed : 3 hours                                                                                           Maximum Marks: 100

General Instructions:

  1.  All questions are compulsory.
  2.  Please check that this question paper contains 26 questions.
  3.  Questions 1-6 in Section A are very short-answer type questions carrying 1 mark each.
  4.  Questions 7-19 in Section B are long-answer I type questions carrying 4 marks each.
  5. Questions 20-26 in Section C are long-answer II type questions carrying 6 marks each.
  6.  Please write down the serial number of the question before attempting it.

SET I

SECTION – A

Question.1.
cbse-previous-year-solved-papers-class-12-maths-outside-delhi-2010-1
Solution.
cbse-previous-year-solved-papers-class-12-maths-outside-delhi-2010-2

Question.2.Write the principal value of sec-1 (-2).
Solution.
cbse-previous-year-solved-papers-class-12-maths-outside-delhi-2010-3

Question.3.What positive value of x makes the following pair of determinants equal ?
cbse-previous-year-solved-papers-class-12-maths-outside-delhi-2010-4
Solution.
cbse-previous-year-solved-papers-class-12-maths-outside-delhi-2010-5

Question.4.
cbse-previous-year-solved-papers-class-12-maths-outside-delhi-2010-6
Solution.
cbse-previous-year-solved-papers-class-12-maths-outside-delhi-2010-7

Question.5.Write the adjoint of the following matrix:
cbse-previous-year-solved-papers-class-12-maths-outside-delhi-2010-8
Solution.
cbse-previous-year-solved-papers-class-12-maths-outside-delhi-2010-9
cbse-previous-year-solved-papers-class-12-maths-outside-delhi-2010-10

Question.6.Write the value of the following integral:
cbse-previous-year-solved-papers-class-12-maths-outside-delhi-2010-11
Solution.
cbse-previous-year-solved-papers-class-12-maths-outside-delhi-2010-12

Question.7. A is a square matrix of order 3 and | A | =7. Write the value of | adj. A |.[1]
Solution.
cbse-previous-year-solved-papers-class-12-maths-outside-delhi-2010-13

Question.8.Write the distance of the following plane from the origin
:2x – y + 2z +1 = 0
Solution.Given plane is2x – y + 2z+1 = 0 .
Its distance from origin is
cbse-previous-year-solved-papers-class-12-maths-outside-delhi-2010-14

Question.9. Write a vector of magnitude 9 units in the direction of vector
cbse-previous-year-solved-papers-class-12-maths-outside-delhi-2010-15
Solution.
cbse-previous-year-solved-papers-class-12-maths-outside-delhi-2010-16

Question.10.
cbse-previous-year-solved-papers-class-12-maths-outside-delhi-2010-17
Solution.
cbse-previous-year-solved-papers-class-12-maths-outside-delhi-2010-18

Question.11. A family has 2 children. Find the probability that both are boys, if it is known that
(i) at least one of the children is a boy,
(ii)the elder child is a boy.
Solution: Let eldest boy and girl be represented by capital B and G respectively and youngest boy and girl be denoted by small letter b and g respectively.
cbse-previous-year-solved-papers-class-12-maths-outside-delhi-2010-19
cbse-previous-year-solved-papers-class-12-maths-outside-delhi-2010-20

Question.12. Show that relation Sin the set A={x ∈ Z:0 ≤ x ≤ 12} given
by S = {(a, b): a, b∈ Z, | a – b | is divisible by 4} is an equivalence relation. Find the set of all elements related to 1.
Solution: Reflexivity:
cbse-previous-year-solved-papers-class-12-maths-outside-delhi-2010-21
cbse-previous-year-solved-papers-class-12-maths-outside-delhi-2010-22

Question.13. Prove the following
cbse-previous-year-solved-papers-class-12-maths-outside-delhi-2010-23
Solution.
cbse-previous-year-solved-papers-class-12-maths-outside-delhi-2010-24
cbse-previous-year-solved-papers-class-12-maths-outside-delhi-2010-25

Question.14. Express the following matrix as the sum of a symmetric and skew symmetric matrix, and verify your result:
cbse-previous-year-solved-papers-class-12-maths-outside-delhi-2010-26
Solution.
cbse-previous-year-solved-papers-class-12-maths-outside-delhi-2010-27
cbse-previous-year-solved-papers-class-12-maths-outside-delhi-2010-28
cbse-previous-year-solved-papers-class-12-maths-outside-delhi-2010-29
cbse-previous-year-solved-papers-class-12-maths-outside-delhi-2010-30

Question.15.
cbse-previous-year-solved-papers-class-12-maths-outside-delhi-2010-31
Solution.
cbse-previous-year-solved-papers-class-12-maths-outside-delhi-2010-32
cbse-previous-year-solved-papers-class-12-maths-outside-delhi-2010-33
Solution. Given that
cbse-previous-year-solved-papers-class-12-maths-outside-delhi-2010-34
cbse-previous-year-solved-papers-class-12-maths-outside-delhi-2010-35

Question.16.
cbse-previous-year-solved-papers-class-12-maths-outside-delhi-2010-36
Solution.
cbse-previous-year-solved-papers-class-12-maths-outside-delhi-2010-37
Find the distance of the point P(6,5,9) from the plane determined by the points A(3, – 1, 2), B(5,2,4) and C(-1, -1,6).
Solution.
cbse-previous-year-solved-papers-class-12-maths-outside-delhi-2010-38
cbse-previous-year-solved-papers-class-12-maths-outside-delhi-2010-39

Question.17. Solve the following differential equation:
cbse-previous-year-solved-papers-class-12-maths-outside-delhi-2010-40
Solution.
cbse-previous-year-solved-papers-class-12-maths-outside-delhi-2010-41
cbse-previous-year-solved-papers-class-12-maths-outside-delhi-2010-42
Solution.
cbse-previous-year-solved-papers-class-12-maths-outside-delhi-2010-43
cbse-previous-year-solved-papers-class-12-maths-outside-delhi-2010-44
cbse-previous-year-solved-papers-class-12-maths-outside-delhi-2010-45

Question.18.Show that the differential equation is homogeneous and solve it.
cbse-previous-year-solved-papers-class-12-maths-outside-delhi-2010-46
Solution.
cbse-previous-year-solved-papers-class-12-maths-outside-delhi-2010-47
cbse-previous-year-solved-papers-class-12-maths-outside-delhi-2010-48
cbse-previous-year-solved-papers-class-12-maths-outside-delhi-2010-49
cbse-previous-year-solved-papers-class-12-maths-outside-delhi-2010-50
cbse-previous-year-solved-papers-class-12-maths-outside-delhi-2010-51
cbse-previous-year-solved-papers-class-12-maths-outside-delhi-2010-52

Question.19.
cbse-previous-year-solved-papers-class-12-maths-outside-delhi-2010-53
Solution.
cbse-previous-year-solved-papers-class-12-maths-outside-delhi-2010-54
cbse-previous-year-solved-papers-class-12-maths-outside-delhi-2010-55
cbse-previous-year-solved-papers-class-12-maths-outside-delhi-2010-56
cbse-previous-year-solved-papers-class-12-maths-outside-delhi-2010-58

Question.20. Evaluate the following:
cbse-previous-year-solved-papers-class-12-maths-outside-delhi-2010-129 (2)
Solution.
cbse-previous-year-solved-papers-class-12-maths-outside-delhi-2010-59
cbse-previous-year-solved-papers-class-12-maths-outside-delhi-2010-60

Question.21.
cbse-previous-year-solved-papers-class-12-maths-outside-delhi-2010-61
Solution.
cbse-previous-year-solved-papers-class-12-maths-outside-delhi-2010-62

Question.22.
cbse-previous-year-solved-papers-class-12-maths-outside-delhi-2010-63
Solution.
cbse-previous-year-solved-papers-class-12-maths-outside-delhi-2010-64

Question.23. Using properties of determinants, prove the following:
cbse-previous-year-solved-papers-class-12-maths-outside-delhi-2010-65
Solution.
cbse-previous-year-solved-papers-class-12-maths-outside-delhi-2010-66
cbse-previous-year-solved-papers-class-12-maths-outside-delhi-2010-67
cbse-previous-year-solved-papers-class-12-maths-outside-delhi-2010-68
cbse-previous-year-solved-papers-class-12-maths-outside-delhi-2010-69
Solution.
cbse-previous-year-solved-papers-class-12-maths-outside-delhi-2010-70
cbse-previous-year-solved-papers-class-12-maths-outside-delhi-2010-71

Question.24. A bag contains 4 balls. Two balls are drawn at random and are found to be white. What is the probability that all balls are white ?
Solution: Consider the following events:
E1 = The bag contains two white and two other colour balls.
E2 = The bag contains three white and one other coloured ball. –
E3 = The bag contains all white balls.
A = Two ball drawn are white.
Clearly,
cbse-previous-year-solved-papers-class-12-maths-outside-delhi-2010-72

Question.25.One kind of cake requires 300g of flour and 15g of fat, another kind of cake requires 150g of flour and 30g of fat. Find the maximum number of cakes which can be made from 7.5 kg of flour and 600g of fat, assuming that there is no shortage of the other ingredients used in making the cakes. Make it as an L.P.P. and solve it graphically.
Solution: Let x be the number of cakes of first kind and y is of second kind. We make the following table from the given data:
cbse-previous-year-solved-papers-class-12-maths-outside-delhi-2010-73
cbse-previous-year-solved-papers-class-12-maths-outside-delhi-2010-74
cbse-previous-year-solved-papers-class-12-maths-outside-delhi-2010-75
cbse-previous-year-solved-papers-class-12-maths-outside-delhi-2010-76

Question.26.Find the coordinates of the foot of the perpendicular
and the perpendicular distance of the point P(3, 2,1) from the plane 2x-y + z +1 = 0. Find also the image of the point in the plane.
Solution.Equation of the plane is given as
cbse-previous-year-solved-papers-class-12-maths-outside-delhi-2010-77
cbse-previous-year-solved-papers-class-12-maths-outside-delhi-2010-78

Question.27. Find the area of the circle 4x2+4y2=9 Which is interior to the parabola x2=4y
Solution.Given curves are
cbse-previous-year-solved-papers-class-12-maths-outside-delhi-2010-79
and
Clearly, the first one represents a circle with centre at origin and radius at 1.5 umts. The second one represents a parabola opening upwards.
cbse-previous-year-solved-papers-class-12-maths-outside-delhi-2010-80
cbse-previous-year-solved-papers-class-12-maths-outside-delhi-2010-81
cbse-previous-year-solved-papers-class-12-maths-outside-delhi-2010-82 OR
Using integration, find the area of the triangle ABC, coordinates of whose vertices are A(4,1), B(6,6) and C(8, 4).
Solution: First we find the equation of the sides of ΔABC
cbse-previous-year-solved-papers-class-12-maths-outside-delhi-2010-83
cbse-previous-year-solved-papers-class-12-maths-outside-delhi-2010-84
cbse-previous-year-solved-papers-class-12-maths-outside-delhi-2010-85
cbse-previous-year-solved-papers-class-12-maths-outside-delhi-2010-86

Question.28.If the length of three sides of a trapezium other than the base is 10 cm each, find the area of the trapezium, when it is maximum.
Solution: Let ABCD be the given trapezium.
cbse-previous-year-solved-papers-class-12-maths-outside-delhi-2010-87
cbse-previous-year-solved-papers-class-12-maths-outside-delhi-2010-88
cbse-previous-year-solved-papers-class-12-maths-outside-delhi-2010-89
cbse-previous-year-solved-papers-class-12-maths-outside-delhi-2010-90

Question.29. Find the intervals in which the following function f(x) = 20 – 9x + 6x2 – x3 is
(a) strictly increasing,
(b) strictly decreasing.
Solution.
cbse-previous-year-solved-papers-class-12-maths-outside-delhi-2010-91
cbse-previous-year-solved-papers-class-12-maths-outside-delhi-2010-92

SET II

Note: Except for the following questions, all the remaining questions have been asked in previous set.

SECTION – A

Question.10.
cbse-previous-year-solved-papers-class-12-maths-outside-delhi-2010-93
Solution.
cbse-previous-year-solved-papers-class-12-maths-outside-delhi-2010-94

SECTION – B

Question.11.Prove the following:
cbse-previous-year-solved-papers-class-12-maths-outside-delhi-2010-95
Solution.
cbse-previous-year-solved-papers-class-12-maths-outside-delhi-2010-96
cbse-previous-year-solved-papers-class-12-maths-outside-delhi-2010-131 (2)
Solution. We have,
cbse-previous-year-solved-papers-class-12-maths-outside-delhi-2010-98
cbse-previous-year-solved-papers-class-12-maths-outside-delhi-2010-99

Question.14.
cbse-previous-year-solved-papers-class-12-maths-outside-delhi-2010-100
Solution.
cbse-previous-year-solved-papers-class-12-maths-outside-delhi-2010-101 (2)
cbse-previous-year-solved-papers-class-12-maths-outside-delhi-2010-102

Question.18.
cbse-previous-year-solved-papers-class-12-maths-outside-delhi-2010-103
Solution.
cbse-previous-year-solved-papers-class-12-maths-outside-delhi-2010-104
cbse-previous-year-solved-papers-class-12-maths-outside-delhi-2010-105
cbse-previous-year-solved-papers-class-12-maths-outside-delhi-2010-106

Question.20. Show that the following differential equation is homogeneous, and then solve it:
cbse-previous-year-solved-papers-class-12-maths-outside-delhi-2010-107
Solution.
cbse-previous-year-solved-papers-class-12-maths-outside-delhi-2010-108
cbse-previous-year-solved-papers-class-12-maths-outside-delhi-2010-109
cbse-previous-year-solved-papers-class-12-maths-outside-delhi-2010-110
cbse-previous-year-solved-papers-class-12-maths-outside-delhi-2010-111
cbse-previous-year-solved-papers-class-12-maths-outside-delhi-2010-112

SECTION – C

Question.23. Find the equations of the tangent and the normal to the
cbse-previous-year-solved-papers-class-12-maths-outside-delhi-2010-113
Solution.
cbse-previous-year-solved-papers-class-12-maths-outside-delhi-2010-114
cbse-previous-year-solved-papers-class-12-maths-outside-delhi-2010-115
cbse-previous-year-solved-papers-class-12-maths-outside-delhi-2010-116

Question.24.Find the equation of the plane passing through the point P(1,1,1) and containing the line that the plane contains
cbse-previous-year-solved-papers-class-12-maths-outside-delhi-2010-117
Solution.
cbse-previous-year-solved-papers-class-12-maths-outside-delhi-2010-118
cbse-previous-year-solved-papers-class-12-maths-outside-delhi-2010-119
cbse-previous-year-solved-papers-class-12-maths-outside-delhi-2010-120

SET III

Note: Except for the following questions, all the remaining questions have been asked in previous sets.

SECTION – A

Question.6.
cbse-previous-year-solved-papers-class-12-maths-outside-delhi-2010-121
Solution.
cbse-previous-year-solved-papers-class-12-maths-outside-delhi-2010-122

Question.7.
cbse-previous-year-solved-papers-class-12-maths-outside-delhi-2010-123
Solution.
cbse-previous-year-solved-papers-class-12-maths-outside-delhi-2010-124

SECTION – B

Question.11. Show that the relation S defined on the set N x N by (a, b) S (c, d) ⇒ a + d = b + c is an equivalence relation.
Solution.
cbse-previous-year-solved-papers-class-12-maths-outside-delhi-2010-125

Question.15. For the following matrices A and B, verify that (AB)’ = B’A’ where
cbse-previous-year-solved-papers-class-12-maths-outside-delhi-2010-126
Solution. Givsen that,
cbse-previous-year-solved-papers-class-12-maths-outside-delhi-2010-127
cbse-previous-year-solved-papers-class-12-maths-outside-delhi-2010-128

Question.17.Solve the following differential equation:
cbse-previous-year-solved-papers-class-12-maths-outside-delhi-2010-129
Solution.
cbse-previous-year-solved-papers-class-12-maths-outside-delhi-2010-130
cbse-previous-year-solved-papers-class-12-maths-outside-delhi-2010-131
Solution.
cbse-previous-year-solved-papers-class-12-maths-outside-delhi-2010-132
cbse-previous-year-solved-papers-class-12-maths-outside-delhi-2010-133
cbse-previous-year-solved-papers-class-12-maths-outside-delhi-2010-134

Question.20.
cbse-previous-year-solved-papers-class-12-maths-outside-delhi-2010-135
Solution.
cbse-previous-year-solved-papers-class-12-maths-outside-delhi-2010-136
cbse-previous-year-solved-papers-class-12-maths-outside-delhi-2010-137

SECTION – C

Question.23. Using matrices, solve the following system of equations:
cbse-previous-year-solved-papers-class-12-maths-outside-delhi-2010-138
Solution. We have,
cbse-previous-year-solved-papers-class-12-maths-outside-delhi-2010-139
cbse-previous-year-solved-papers-class-12-maths-outside-delhi-2010-140
cbse-previous-year-solved-papers-class-12-maths-outside-delhi-2010-141
cbse-previous-year-solved-papers-class-12-maths-outside-delhi-2010-142
Solution.
cbse-previous-year-solved-papers-class-12-maths-outside-delhi-2010-143
cbse-previous-year-solved-papers-class-12-maths-outside-delhi-2010-144

Question.25. Show that the volume of the greatest cylinder that can be inscribed in a cone of height ‘h’ and semi-vertical
cbse-previous-year-solved-papers-class-12-maths-outside-delhi-2010-145
Solution.Let us consider height of cone = h
cbse-previous-year-solved-papers-class-12-maths-outside-delhi-2010-146
cbse-previous-year-solved-papers-class-12-maths-outside-delhi-2010-147
cbse-previous-year-solved-papers-class-12-maths-outside-delhi-2010-148